PDA

Archiv verlassen und diese Seite im Standarddesign anzeigen : Quantenlogik


Knut Hacker
14.09.11, 19:45
Quantenlogik


„Wahre Sätze sind immer daran zu erkennen, dass man sie alle umkehren kann und dass sie alsdann eben so wahr sind, und vielleicht noch mehr.“ Goethe, Wanderjahre

„ Jeder Aussage steht eine gleichwertige gegenüber“ Sextus Empiricus

„Es gibt triviale Wahrheiten und große Wahrheiten. Das Gegenteil einer trivialen Wahrheit ist schlichtweg falsch. Das Gegenteil einer großen Wahrheit ist ebenfalls wahr.“ (Niels Bohr)



Der Begriff der mehrwertigen Logik – im Gegensatz zur zweiwertigen aristotelischen Logik – wurde bekanntlich von Niels Bohr geprägt.Er hat hierzu nichts Schriftliches hinterlassen.Werner Heisenberg schreibt dazu in „Physik und Philosophie“:

„In der Quantentheorie muss offenbar dieses Gesetz `tertium non datur´ abgeändert werden.....von Weizsäcker schlägt vor, die klassische Logik in ähnlicher Weise als ´a priori´ zur Quantenlogik zu betrachten, wie die klassische Physik ein a priori für die Quantentheorie darstellt. Die klassische Logik würde dann als eine Art Grenzfall in der Quantenlogik enthalten sein, aber die letztere würde doch das allgemeinere logische Schema darstellen.“

Die zweiwertige aristotelische Logik vom ausgeschlossenen Dritten lässt sich allerdings nicht begründen, da dies diesen Satz vom ausgeschlossenen Dritten voraussetzen würde, also zu einem Zirkelschluss führen würde.

C.F.v.Weizsäcker schreibt hierzu in „Ein Blick auf Platon“:

„Hier werden wir daran erinnert, dass die Quelle aller logischen Grundlagenkrisen in unserem Jahrhundert am Begriff der Negation zu liegen scheint. Die logischen Paradoxien verwenden stets die Negation beziehungsweise die Falschheit, so der Kreter, der versichert, das, was er soeben sage, sei falsch, oder die Menge aller der Mengen, die sich selbst nicht als Element enthalten.Das von Brouwer kritisierte Tertium non datur versichert, dass ein Satz und seine Negation die Menge der Möglichkeiten ausschöpfen. J.v.Neumanns Quantenlogik behauptet, dass eine Eigenschaft einem physikalischen Objekt nicht mit Notwendigkeit entweder zukommt oder nicht zukommt.“

Der wohl bekannteste gegenwärtige Quantenphysiker, Anton Zeilinger, schreibt in „Spektrum der Wirtschaft, Dossier“ 2/10:

„ Während das klassische Bit nur entweder die Werte 0 oder 1 annehmen kann, existiert ein Quantenbit, Qubit genannt, in einer Superposition von 0 oder 1. Genauso, wie für ein Teilchen, das durch einen Doppelspalt geht, nicht festgelegt ist, welchen Spalt es passiert, nimmt ein Quantenbit quasi die beiden Zustände 0 und 1 gleichzeitig ein.“

Eben das ist mehrwertige Quantenlogik( insbes.Hans Reichenbaches und Lukasiewic´s) In ihr ist das Distributivgesetz ungültig.

Ich verweise auf die grundlegenden Werke von
G.Birkhoff / J.v.Neumann:“The Logic of Quantum Mechanics“und
R.L.G. Hughes: „Quantum Logic“

Die Entdeckung zahlreicher Quanteneffekte auch im Makrokosmos (eine Übersicht gibt Vladko Vedral http://www.spektrum.de/artikel/1116464 ) lässt die Quanten-Logik nicht – wie die aristotelische Logik – als bloßen Operator kognitiv-resultativer Erfassung von Problemen erscheinen, sondern als Wesenszug der als durch „Verschränkung“ vor-raumzeitlich disponierten Natur. Die Natur ist kein Algorithmus. Sie ist nicht „wohldefiniert“ sondern holistisch. Es gibt mangels scharfer Unterscheidbarkeit keine Identitäten, sodass letztlich Sein und Nichtsein identisch sind :

Hans-Peter Dürr (Quantenphysiker,*1929):
“Es gibt … gar nichts Seiendes, nichts, was existiert.“

Erwin Schrödinger (1882-1961; Quantenphysiker; Nobelpreisträger):

Die in Raum und Zeit ausgedehnte Welt existiert nur in unserer Vorstellung. Dass sie außerdem noch etwas anderes sei, dafür bietet jedenfalls die Erfahrung – wie schon Berkeley wusste – keinen Anhaltspunkt ( Dürr, 167 ).

Werner Heisenberg (1901-1976 ; Quantenphysiker-“Unschärferelation“-; Nobelpreisträger):

...dass nicht einmal die Eigenschaft des “ Seins“, wenn man hier überhaupt von Eigenschaft reden will, dem Elementarteilchen ohne Einschränkung zukommt. Es ist eine Möglichkeit oder eine Tendenz zum Sein (Physik und Philosophie).

Stephen Hawking (*1942):

Es gibt keinen abbild-oder theorieunabhängigen Realitätsbegriff (Der große Entwurf).



Darüber ist man sich in der Philosophie seit jeher einig:

Gorgias von Leontinoi (483-375 v. Chr.):

ταὐτόν ἐστι τῷ μὴ ὄντι τὸ ὄν
Dasselbe ist das Seiende wie das Nichtseiende

Heraklit von Ephesus (545-475 v, Chr.):

ταὐτὸν...εἶναι καὶ μὴ εἶναι
dasselbe...( ist ) Sein und Nichtsein ( A 7; Arist. Metaph.Γ 3. 1005b 23 )

Aristoteles ( 384/3 – 322 v. Chr. ):

τὸ δὲ εἶναι οὐκ οὐσία οὐδενί
das Sein gehört nicht zum Wesen von etwas ( Analytica posteriora, II. 7 )

Pyrrhon aus Elis ( 365/60-275 v.Chr. ):

περὶ ἑνὸς ἑκάστου λέγοντας ὅτι οὐ μᾶλλον ἔστιν ἢ οὐκ ἔστιν ἢ καὶ ἔστιν καὶ οὐκ ἔστιν ἢ οὔτε ἔστιν οὔτε οὐκ ἔστιν
über jeden einzelnen Gegenstand müsse man sagen, dass er nicht mehr “sei“ als „nicht sei“, oder: dass er sowohl „sei“ als „nicht sei“, oder: dass er weder „sei“ noch „nicht sei“ ( Aristokles bei Eusebius Praep. evang. 14.18.4 – Caizzi 53 )

Johann Gottlieb Fichte (1762-1814):

Ich weiß überall von keinem Sein und auch nicht von meinem eigenen.Es ist kein Sein. - Ich selbst weiß überhaupt nicht und bin nicht.

Georg Wilhelm Friedrich Hegel (1770-1831):

Die, welche auf dem Unterschiede von Sein und Nichts beharren wollen, mögen sich auffordern, anzugeben, worin er besteht (Wissenschaft der Logik).

Das Sein, das unbestimmte Unmittelbare, ist in der Tat Nichts und nicht mehr noch weniger als Nichts.

Das reine Sein und das reine Nichts ist also dasselbe.

Das reine Sein ist die reine Abstrakion, damit das Absolut-Negative, welches, gleichfalls unmittelbar genommen, das Nichts ist.

Das unbestimmte unmittelbar bestimmte „reine Sein“ ist zugleich „dasselbe“ wie „nichts“. Beide finden ihre „Wahrheit“ in „der Bewegung“ als „Werden“.

Das Nichts ist „dieselbe Bestimmung oder vielmehr Bestimmungslosigkeit und damit überhaupt dasselbe, was das reine Seyn ist.“

Das reine Sein als inhaltslose Leere muss als Produkt der vollständigen Abstraktion zugleich auch sein eigenes Gegenteil sein: sowohl auszudifferenzierende affimative Unmittelbarkeit, genannt Sein, als auch auszudifferenzierende reine Negativität, genannt Nichts.

Martin Heidegger (1889-1976):

Im Sein des Seienden geschieht das Nichten des Nichts.

Welt ist nie, sondern weltet.

Jogi
14.09.11, 20:09
Hallo Knut.

Nachdem dir auf AC niemand antwortet versuchst du's hier.:)

Beim Lesen deiner Postings drängt sich mir eine Frage auf:
So ganz alltägliche Dinge, wie z. B. einkaufen, Bahn fahren, etc...
Kriegst du das gebacken?


Gruß Jogi

Bauhof
15.09.11, 14:33
Quantenlogik
Hallo Knut Hacker,

dein Beitrag besteht fast nur aus Zitaten. Ich habe nichts gegen das Zitieren ─ das mache ich auch ─, aber ich vermisse bei dir eine eigene Quintessenz aus den vielen Zitaten. Was willst du uns nahe bringen? Aber bitte keinen Roman, sondern es genügen zunächst drei bis vier Sätze.

M.f.G. Eugen Bauhof

Knut Hacker
15.09.11, 17:35
ich vermisse bei dir eine eigene Quintessenz aus den vielen Zitaten. Was willst du uns nahe bringen?


Hallo Bauhof,

ich wollte nahe bringen, dass das, was in der spekulativen Philosophie (und im meditativen Buddhismus ) über Jahrtausende hinweg Ansichtssache ( beziehungsweise spirituelle Erfahrung) war, seit einem Jahrhundert naturwissenschaftlich-experimentelle Unumständlichkeit ist ( „ Das wichtigste philosophische Ereignis unseres Jahrhunderts ist die moderne Physik.“ Werner Heisenberg ):
Das begriffliche Denken, insbesondere das Denken in Gegensätzen ist nicht geeignet, die Natur in ihrer Elementarität zu erfassen.
Dies ist evolutionsbiologisch gut zu erklären: Dieses in der durch den Paradigmenwechsel in den Naturwissenschaften überholte Denken hat sich als Überlebensvorteil zur Orientierung und Behauptung im Alltag entwickelt, für Bereiche außerhalb der unmittelbaren Sinneserfahrung fehlte für eine Weiterentwicklung die Möglichkeit der Adaption und Selektion.

Ich habe dies deshalb zur Diskussion gestellt, weil heute in den Naturwissenschaften wieder eine positivistische Tendenz im Vordringen ist – siehe insbesondere Wigner und Deutsch, wonach die klassische Newtonsche Physik nicht ein Grenzfall der Quantenphysik sei ( wovon deren „Kopenhagener Deutung“ ausgeht), sondern diese in jene zurückgeführt werden könne.

Knut Hacker
15.09.11, 19:25
Beim Lesen deiner Postings drängt sich mir eine Frage auf:
So ganz alltägliche Dinge, wie z. B. einkaufen, Bahn fahren, etc...
Kriegst du das gebacken?



Verstehst du ein Quanten-Diskussionsforum als Chatroom für alltägliche Dinge?

Aber falls du dich auf Normalo-Trip befindest, lass dich nicht stören!

amc
15.09.11, 20:17
Georg Wilhelm Friedrich Hegel (1770-1831):

Die, welche auf dem Unterschiede von Sein und Nichts beharren wollen, mögen sich auffordern, anzugeben, worin er besteht (Wissenschaft der Logik).

Ich würde nicht von einem Unterschied sprechen, wie soll man auch etwas mit dem Nichts vergleichen, aber auf die Frage, "was kennzeichnet das Sein" wäre meine Antwort: Masse

Besitzt nicht alles was wir als physikalisch existent bezeichnen können Masse? Raum und Zeit könnte man als Beziehungsgeflecht bezeichnen (vielleicht nur Illusion?), welches untrennbar mit der Masse verbunden ist. Man müsste nur klären was genau Masse ist (eher ein Prozess und nichts starres?), wie sie sich zusammensetzt (atomar, wirkend, potentiell?), und warum gibt es sie überhaupt? (ganz schwer)

Einer Sache bin ich mir jedenfalls sicher: Masse ist nicht Nichts! Meine Meinung ...

Gibt es Dinge die man als physikalisch existent betrachten muss, die keine Masse besitzen?

PS:
Ich halte nichts von dem Nichts, denn würde das Nichts existieren, so wäre es nicht Nichts. Also reicht es meiner Meinung nach vollkommen aus, sich mit der Existenz zu beschäftigen, um alles zu erfassen.

EMI
15.09.11, 21:36
Gibt es Dinge die man als physikalisch existent betrachten muss, die keine Masse besitzen?Da gibt's ne Menge Dinge z.B. el.mag.Wellen, Photonen usw. amc,

ich würde es so formulieren:
Gibt es Dinge die man als physikalisch existent betrachten muss und die keine Materie sind?

Antwort: NEIN

Gruß EMI

amc
15.09.11, 23:28
Da gibt's ne Menge Dinge z.B. el.mag.Wellen, Photonen usw. amc,

Das weiß ich doch EMI. Allerdings besitzen diese Dinge ja lediglich keine Ruhemasse. Aber danke trotzdem für den Hinweis. Es ist sicher sehr richtig hier an dieser Stelle nochmal genau darauf einzugehen.

Vor kurzer Zeit nahm ich an, aufgrund lückenhafter Kenntnisse, dass Photonen keine Masse besitzen, somit nicht träge, und keine Quelle für Gravitation sind. Bis du mich hier aufgeklärt hast:

http://www.quanten.de/forum/showpost.php5?p=61773&postcount=13

ich würde es so formulieren:
Gibt es Dinge die man als physikalisch existent betrachten muss und die keine Materie sind? Antwort: NEIN


Um Missverständnissen vorzubeugen ist diese Ausdrucksweise möglicherweise etwas besser geeignet. Aber auch den Begriff "Materie" setzen viele (Laien) nicht in direkten Zusammenhang mit Strahlung. Ich mag den Begriff "Masse" lieber.

Ich kenne mich mit physikalischem Formalismus kaum aus - hier wollte ich eh mal nachfragen - unterscheidet man im physikalischen Formalismus zwischen Materie und Masse? Nein, oder doch? Der Begriff "Materie" wird meist eher im alltäglichen Sprachgebrauch verwendet, "Masse" eher im physikalischen Bereich. Kann man das so sagen?

Dann ist es doch eigentlich auch eine persönliche Geschmackssache ob man sagt: Alles ist Masse, Alles ist Materie, oder auch: Alles ist Energie. Ist das richtig?

Oder sollte man sagen: Alles ist Materie, und alles besitzt Masse/Energie ... ?

EMI
16.09.11, 01:21
Alles ist Masse, Alles ist Materie, oder auch: Alles ist Energie. Ist das richtig?Nein amc,

Materie ist alles (materielle Welt), zur Materie zählen Masse und Strahlung (Energie), egal was die Leut unbedacht so schwatzen.:)

In der Physik sollte man schon versuchen auf saubere Begrifflichkeiten zu achten (grad in einem Forum), sonst entstehen Missverständnisse IMHO.
Wenn ich z.B. schreibe Materie krümmt die Raumzeit, gehe ich davon aus, das Jeder weis das ich Alles damit meine (Masse, Impuls, Energie, Druck...).
Wenn man schreibt Masse krümmt die Raumzeit, fehlt da einiges und man muss dazu schreiben, Energie auch, Impuls auch, Druck auch usw..

"Alles ist Materie und besitzt Masse/Energie" ist da schon besser.

Gruß EMI

Nach PS: Inmaterielles gehört nicht zu unserer Welt und ist Esoterik. Daher stößt Gefasel darüber bei EMI immer sauer auf.
Das was wir nicht sehen/messen sei nicht, ist genau so ein Quark.
Ich frage mich da immer, wieso nur finden wir z.B. Saurierknochen? Ich kenne keinen Menschen der jemals einen Saurier gesehen hat.

amc
16.09.11, 10:52
Materie ist alles (materielle Welt), zur Materie zählen Masse und Strahlung (Energie), egal was die Leut unbedacht so schwatzen.:)

In der Physik sollte man schon versuchen auf saubere Begrifflichkeiten zu achten (grad in einem Forum), sonst entstehen Missverständnisse IMHO.

Kurz nachdem ich gestern den Beitrag schrieb wurde mir der Unterschied zwischen den Begriffen klarer. Masse und Energie sind messbare Eigenschaften, Materie ist das Objekt selbst. Daher hat "Materie" im Formalismus auch nichts verloren. Und daher ist es problematisch zu sagen "Alles ist Masse/Energie". Man sollte dann wirklich eher sagen "Alles besitzt Masse/Energie".

Allerdings kann man an dieser Stelle auch einen Bezug zum eigentlichen Thread-Thema herstellen. Durch die neuen Sichtweisen der QM wissen wir, dass der Begriff "Materie" als eine künstliche Worthülle verstanden werden kann. Unser Verstand benötigt offenbar diese Verobjektivierung, um die Welt zu begreifen. Nur existiert nichts materielles mehr als Objekt, wenn man es von dem Subjekt trennt. Nimmt man alle Eigenschaften der Materie weg, dann bleibt nichts übrig, zumindest gehen wir davon aus, und wissen nicht ob und wenn was dann noch übrig bleibt. Also das was in unserer Welt das wahrhaft existente ist ist nicht die Materie, sondern es sind die Eigenschaften dessen, was wir Materie nennen. Die Eigenschaften selbst sind offenbar das materielle.

So würde ich es jetzt ausdrücken: Alles was man als physikalisch existent betrachten kann, das muss sich durch prinzipiell messbare Eigenschaften bemerkbar machen können. Das Nichts kann dies nicht, daher liegt es nahe es als physikalisch nicht existent zu betrachten. Warum beschäftigt das soviele Denker, das ist doch Zeitverschwendung :)

Knut Hacker
16.09.11, 16:37
Zu der hier aufgeworfenen Frage, was Materie in der Naturwissenschaft sei und ob sie dort alles sei., möchte ich gerne noch einiges anmerken, obwohl es eigentlich nicht zum Thema gehört:

Der Begriff der Materie rührt noch aus der platonisch-cartesianischen Unterscheidung zwischen Materie und Geist her. Abgesehen davon, dass bis heute noch niemand diese Begriffe klar zu definieren vermochte, gibt es sie heute nur noch in der Philosophie. In den Naturwissenschaften ist diese Unterscheidung obsolet. Die Naturwissenschaften beschäftigen sich nach ihrem Selbstverständnis mit messbaren Erscheinungen. Aussagen über nicht Messbares überlässt sie der Philosophie und Theologie.
Bereits in der klassischen Newton´schen Physik ließ die Einführung der physikalischen Begriffe von Energie, Kraftfeldern usw. den Begriff der Materie als fragwürdig erscheinen, noch mehr die Umwandelbarkeit von Energie in Materie und umgekehrt nach der Relativitätstheorie,erst recht in der Quantenphysik die paradoxen Erscheinungen der Superposition und der Verschränkung sowie das Missproblem.

Aber man braucht gar nicht so tief in die Physik einzusteigen, um die Frage der Unterscheidung von Materie und Nicht-Materie („Geist“) und die Frage, ob „alles“ Materie oder Geist sei, als Scheinprobleme beurteilen zu können, welche sich lediglich aufgrund der philosophisch- theologischen Vorbelastung dieser unscharfen Begriffe ergeben:
Wer in der Naturwissenschaft am umfassenden Begriff der Materie festhalten will, muss sich insbesondere folgende Fragen gefallen lassen:

Sind auch Eigenschaften der Materie Materie ?
Ist die Raumzeit Materie?
Ist Bewegung Materie?
Sind die Naturgesetze und der Zufall Materie?
Sind Strukturen und Beziehungen Materie?
Ist Komplexität Materie (Bekanntlich wird Leben und Bewusstsein als Eigenschaft komplex-dynamischer Strukturen der Materie zu erklären versucht nach dem Grundsatz, dass jedes Ganze nicht lediglich die Summe seiner Teile ist, sondern qualitativ etwas Selbstständiges.)?
Sind physikalische Felder Materie, die imaginäre Einheit i, Pi, die Naturkonstanten?

Die heutigen Quantenphysiker wie Zeilinger und Dürr weigern sich, den Begriff „Elementarteilchen“ zu gebrauchen, sie sprechen stattdessen von Potentialitäten/Information.

Harti
16.09.11, 18:50
Das begriffliche Denken, insbesondere das Denken in Gegensätzen ist nicht geeignet, die Natur in ihrer Elementarität zu erfassen.

Hallo Knut Hacker,
dies gilt möglicherweise nicht nur für die "Natur in ihrer Elemetarität".
Der Gegensatz Ruhe/Bewegung wird in der SRT durch das Relativitätsprinzip aufgehoben, indem das Postulat formuliert wird, dass eine Unterscheidung zwischen Ruhe und geradlinig gleichförmiger Bewegung tatsächlich nicht möglich ist; mit anderen Worten: Es gibt keinen Unterschied, er wird im Alltagsleben aus Zweckmäßigkeitsgründen nur vorgestellt.

Ich habe allerdings Zweifel, ob wir eine andere Möglichkeit haben, die Natur anders als dadurch, dass wir Dinge vergleichen (zueinander in Beziehung setzen), zu erfassen. Auch eine mathematische Funktion ist z.B. nichts anderes als eine Regel für eine Beziehung zwischen zwei Dingen, die wir begrifflich vorab unterscheiden müssen.
Beispiel: Strecke (Raum) und Zeit. Beides wird im konkreten Fall mit Hilfe einer Funktion zueinander in Beziehung gesetzt und Geschwindigkeit (oder im kartesischen Koordinatensystem Steigung) genannt.

MfG
Harti

Knut Hacker
16.09.11, 19:34
Ich habe allerdings Zweifel, ob wir eine andere Möglichkeit haben, die Natur anders als dadurch, dass wir Dinge vergleichen (zueinander in Beziehung setzen), zu erfassen. Auch eine mathematische Funktion ist z.B. nichts anderes als eine Regel für eine Beziehung zwischen zwei Dingen, die wir begrifflich vorab unterscheiden müssen.



Da gebe ich dir grundsätzlich recht. Einstein und die alten Quantenphysiker haben ja immer auf das Problem hingewiesen, dass die Erkenntnisse der Relativitätstheorien und der Quantenphysik, obwohl sie unseren angeborenen und tradierten Vorstellungen widersprechen, mit diesen beschrieben werden müssen und sei es in der Sprache der Mathematik, die ja auch – wie die Naturgesetze – nicht der Natur eingeschrieben ist, sondern Produkt unseres ordnenden und abstrahierenden Denkens ist. Die Mathematik hat allerdings den Vorteil, dass sie auch Paradoxien zu beschreiben weiß, ohne dass dadurch – wie bei der vokalen Sprache – der Aussagegehalt verloren geht., da sie keine Semantik kennt.

amc
16.09.11, 22:52
Der Gegensatz Ruhe/Bewegung wird in der SRT durch das Relativitätsprinzip aufgehoben...

Hallo Harti,
der Gegensatz Ruhe/Bewegung ist nicht aufgehoben. So würde ich es nicht formulieren. Nur eine absolute Unterscheidung ist nicht möglich. Es kommt daruf an in welchem Bezugsystem sich ein Beobachter befindet. Ich denke aber es ist klar was du meinst.

Ein Relativitätsprinzip galt übrigens schon bei Newton:
Relativitätsprinzip - Klassische Mechanik (http://de.wikipedia.org/wiki/Relativit%C3%A4tsprinzip#Klassische_Mechanik)

Bauhof
17.09.11, 09:59
Hallo Harti, der Gegensatz Ruhe/Bewegung ist nicht aufgehoben. So würde ich es nicht formulieren. Nur eine absolute Unterscheidung ist nicht möglich. Es kommt daruf an in welchem Bezugsystem sich ein Beobachter befindet. Ich denke aber es ist klar was du meinst.
Hallo amc,

volle Zustimmung, dass eine absolute Unterscheidung zwischen Ruhe und Bewegung nicht möglich ist. Aber was Harti damit meint, versuche ich schon seit mehreren Jahren zu ergründen, leider bis jetzt erfolglos.

M.f.G. Eugen Bauhof

Knut Hacker
17.09.11, 13:06
Ich habe Harti bei wohlwollender Auslegung so verstanden, dass er ausdrücken wollte, dass es nach der Relativitätstheorie kein absolutes Bezugssystem mehr für Ruhe und Bewegung gebe.

Schon Newton, der von einem absoluten Raum und einer absoluten Zeit ausging, in Bezug auf die Bewegung von der Ruhelage zu unterscheiden sei, hatte ein für ihn unlösbares Problem damit. In seinen „Prinzipia“ bekennt er:
„Körper, welche in einem gegebenen Raum eingeschlossen sind, haben dieselbe Bewegung unter sich; dieser Raum mag ruhen oder sich gleichförmig geradlinig, nicht aber im Kreise fortbewegen.“

Albert Einstein schreibt in seinem zusammen mit Leopold Infeld herausgegebenem Buch „Die Evolution der Physik“:
„1)Im leeren Raum ist die Lichtgeschwindigkeit stets konstant. Sie hängt weder von der Bewegung der Lichtquelle noch von der des Beobachters ab
2).In zwei gleichförmig gegeneinander bewegten Systemen herrschen genau dieselben Naturgesetze. Es gibt keine Möglichkeit, eine absolute gleichförmige Bewegung zu konstatieren.“

Das Dilemma der klassischen Physik zeigt er wie folgt auf:

„In der klassischen Physik gibt es keine absolute gleichförmige Bewegung. Wenn zwei Systeme sich gleichförmig gegeneinander bewegen, dann hat es keinen Sinn zu sagen: „Dieses System ruht, und jenes bewegt sich“. Wenn jedoch zwei Systeme ungleichförmig gegeneinander bewegt werden, dann kann man sehr wohl sagen: „Dieser Körper bewegt sich, und jener ruht (beziehungsweise bewegt sich gleichförmig)“. Der Begriff „absolute Bewegung“ bedeutet dann nämlich etwas ganz Bestimmtes. Hier tut sich zwischen dem vom gesunden Menschenverstand geleiteten Empfinden und der klassischen Physik eine tiefe Kluft auf. Die erwähnten Schwierigkeiten, die des Inertialsystems und die der absoluten Bewegung, sind auf das engste miteinander verknüpft; denn eine absolute Bewegung kann es nur geben, wenn ein Inertialsystem existiert, für das die Naturgesetze gelten“.

Die Voraussetzung für Bewegung, die Raumzeit, wurde bereits durch die Relativitätstheorien infrage gestellt. So sind Photonen wie Elektronen punktförmig also unendlich klein, sie besitzen keine Ruhemasse und bewegen sich stets mit Lichtgeschwindigkeit, haben also keine Eigenzeit.
Einstein prägte daher dem Satz:
„Leute wie wir, die an die Physik glauben, wissen, dass die Unterscheidung zwischen Vergangenheit, Gegenwart und Zukunft nur eine hartnäckige, beharrliche Illusion ist.“

In der Quantenphysik lassen sich die Superposition und die Verschränkung als Raum-und Zeitlosigkeit interpretieren. In der Wheeler-De-Witt -Gleichung kommt überhaupt keine Zeitvariable vor! Sie wird ins Feld geführt gegen die Theorie einer Quantelung der Raumzeit.

fossilium
17.09.11, 16:44
Hallo Knut,

[QUOTE=Knut Hacker;63125]
Dies ist evolutionsbiologisch gut zu erklären: Dieses in der durch den Paradigmenwechsel in den Naturwissenschaften überholte Denken hat sich als Überlebensvorteil zur Orientierung und Behauptung im Alltag entwickelt, für Bereiche außerhalb der unmittelbaren Sinneserfahrung fehlte für eine Weiterentwicklung die Möglichkeit der Adaption und Selektion[.QUOTE]

Das Argument, wir kämen evolutionsbiologisch nicht über gewisse Grenzen hinaus, wird oft angeführt, dabei ist es auch ein evolutionsbiologisches Resultat, dass wir diese Grenzen erkennen, und süchtig danach sind, diese zu überwinden, womit sich die Evolution selbst ein Bein gestellt hat.

[QUOTE=Knut Hacker;63125]
Das begriffliche Denken, insbesondere das Denken in Gegensätzen ist nicht geeignet, die Natur in ihrer Elementarität zu erfassen.[.QUOTE]

Na, na. Ich denke: ohne Gegensätze nichts Verbindendes. Erst muss man die Unterschiede erkennen, dann kann man die Beziehungen erkennen, und dann daraus das Ganze. Eine andere Reihenfolge ist nicht möglich. Kann sein, dass es asiatische Tradition ist, erst das Ganze zu sehen, aber daraus ist in den vergangenen Jahrtausenden keine Blüte der Wissenschaft so wie im Westen hervorgegangen. Ausserdem würde sich ohne Gegensätze auch pysikalisch gar nichts entfalten. Immerhin sind alle elementaren Prozesse im Universum
aus Symmetiebrüchen hervorgegangen, und nicht aus der (fernöstlichen)universellen Harmonie.

Grüsse Fossilium

fossilium
17.09.11, 16:49
Hi Knut,
nochmal kurz zu diesem:


Der Begriff der Materie rührt noch aus der platonisch-cartesianischen Unterscheidung zwischen Materie und Geist her. Abgesehen davon, dass bis heute noch niemand diese Begriffe klar zu definieren vermochte, gibt es sie heute nur noch in der Philosophie.

In den Naturwissenschaften ist diese Unterscheidung obsolet.
Worin sieht Du den Unterschied zwischen der Wirklichkeit der Gedanken (Vorstellung) und der Wirklichkeit der uns umgebenden Natur ?

Grüsse Fossilum

Knut Hacker
17.09.11, 20:11
Worin sieht Du den Unterschied zwischen der Wirklichkeit der Gedanken (Vorstellung) und der Wirklichkeit der uns umgebenden Natur ?


Die Frage, ob alles durch Denken, Fühlen und Werten Wahrgenommene lediglich Konstrukt unseres Bewusstseins ist (und sogar auch die Prämisse eines vorhandenen Bewusstseins), oder ob das Bewusstsein eine Außenwelt widerspiegelt, wenn auch lediglich selektiv, abstrakt und adaptiv, ist müßig, da wir unser Bewusstsein nicht verlassen können und daher nur das als Realität wahrnehmen können, was uns unser Bewusstsein als solche ausgibt.

Knut Hacker
17.09.11, 20:18
Na, na. Ich denke: ohne Gegensätze nichts Verbindendes. Erst muss man die Unterschiede erkennen, dann kann man die Beziehungen erkennen, und dann daraus das Ganze. Eine andere Reihenfolge ist nicht möglich. Kann sein, dass es asiatische Tradition ist, erst das Ganze zu sehen, aber daraus ist in den vergangenen Jahrtausenden keine Blüte der Wissenschaft so wie im Westen hervorgegangen. Ausserdem würde sich ohne Gegensätze auch pysikalisch gar nichts entfalten. Immerhin sind alle elementaren Prozesse im Universum
aus Symmetiebrüchen hervorgegangen, und nicht aus der (fernöstlichen)universellen Harmonie.

Die Verdienste der westlichen reduktionistischen Denkweise - manifestiert in der heutigen Technik - sind unbestritten. Aber wir sind eben durch die Quantentheorie an die Grenzen dieses Erkenntnisweges geraten. Superposition und Verschränkung lassen sich nicht reduktionistisch erfassen.
Auch in der Chaosforschung hat man ja die Grunderkenntnis gewonnen, dass alles mit allem zusammenhängt. Die Logiker haben schon seit jeher argumentiert, dass das Ganze nicht lediglich aus der Summe seiner Teile bestehe.

EMI
17.09.11, 23:59
Die Frage, ob alles durch Denken, Fühlen und Werten Wahrgenommene lediglich Konstrukt unseres Bewusstseins ist (und sogar auch die Prämisse eines vorhandenen Bewusstseins), oder ob das Bewusstsein eine Außenwelt widerspiegelt, wenn auch lediglich selektiv, abstrakt und adaptiv, ist müßig, da wir unser Bewusstsein nicht verlassen können und daher nur das als Realität wahrnehmen können, was uns unser Bewusstsein als solche ausgibt.Das was ein Mensch denkt ist für ihn die Realität, unbestritten Knut,

aber, und da bin ich mir absolut sicher, die Realität/Wirklichkeit existiert unabhängig von uns und von unserem Bewustsein.

Davon bin ich fest überzeugt und diese Überzeugung nehme ich mit ins Grab, auch das Atome und Elementarteilchen real existieren und keine Fiktion sind!

Gruß EMI

Marco Polo
18.09.11, 01:06
Hallo EMI,

aber, und da bin ich mir absolut sicher, die Realität/Wirklichkeit existiert unabhängig von uns und von unserem Bewustsein.

Davon bin ich fest überzeugt und diese Überzeugung nehme ich mit ins Grab, auch das Atome und Elementarteilchen real existieren und keine Fiktion sind!

der Meinung bin ich zwar auch. Aber wie ist das z.B. mit der Gegenwart? Die ist für den Beobachter real. Aber zumindet laut Einstein ist sie eine Fiktion.

Ohne Beobachter gibt es demnach also keine Gegenwart.

Daran sieht man also, dass das mit der Wirklichkeit so eine Sache ist.

Gruss, Marco Polo

EMI
18.09.11, 02:13
Aber wie ist das z.B. mit der Gegenwart?Die Gegenwart ist meist Öde Marco,

die Vergangenheit erinnert mich immer an die süssen Mausies, sehr schön.
Die Zukunft lässt hoffen. auf die Mausies die man da noch so kennenlernt.

Eins ist gewiss (zumindest für mich) in 100 Jahren ist damit leider wohl Sicht im Schacht.

Gruß EMI

PS: Nur der Mensch denkt in die Vergangenheit und Zukunft, andere entwickelte Lebensformen leben NUR in der Gegenwart.

Marco Polo
18.09.11, 02:27
PS: Nur der Mensch denkt in die Vergangenheit und Zukunft, andere entwickelte Lebensformen leben NUR in der Gegenwart.

So stehts oft geschrieben, EMI. Aber wie verträgt sich das z.B. mit Problemlösungsstrategien von Affen? Die Lösung des Problems liegt ja aus deren Sicht in der Zukunft. Hmm...

Egal. Wir sollten das Thema hier nicht verwässern...

Gute Nacht, MP

EMI
18.09.11, 02:47
Egal. Wir sollten das Thema hier nicht verwässern...Wasser, Affen...da fällt mir gleich was zu ein...ein Moment Marco,

ah gefunden:

http://www.quanten.de/forum/showpost.php5?p=24745&postcount=17

Gruß EMI

Bauhof
18.09.11, 09:13
... der Meinung bin ich zwar auch. Aber wie ist das z.B. mit der Gegenwart? Die ist für den Beobachter real. Aber zumindet laut Einstein ist sie eine Fiktion. Ohne Beobachter gibt es demnach also keine Gegenwart. Daran sieht man also, dass das mit der Wirklichkeit so eine Sache ist.
Hallo Marc,

real ist nur das, was registriert wurde (laut John Archibald Wheeler (http://de.wikipedia.org/wiki/John_Archibald_Wheeler)). Ob von einem Messinstrument oder einem menschlichen Beobachter registriert wird, ist gleichgültig. Bleiben wir aber beim Beobachter.

Ein Beobachter kann nichts Gegenwärtiges registrieren, sondern immer nur Vergangenes. Denn eine Registrierung exakt scharf in der Gegenwart kann nicht stattfinden, weil das die quantenmechanische Energie-Zeit-Unschärfe nicht zulassen würde. Also kann der Beobachter für die mögliche Existenz oder Nicht-Existenz einer Gegenwart prinzipiell nicht verantwortlich sein.

M.f.G. Eugen Bauhof

P.S. Einstein hielt nicht speziell die Gegenwart, sondern allgemein die Zeit (oder den Zeitablauf) für eine Fiktion

Harti
18.09.11, 10:59
Ich habe Harti bei wohlwollender Auslegung so verstanden, dass er ausdrücken wollte, dass es nach der Relativitätstheorie kein absolutes Bezugssystem mehr für Ruhe und Bewegung gebe.

Genau

Hallo zusammen,
wir müssen allerdings zur Orientierung in der Welt (bei Betrachtung der Wirklichkeit) den Unterschied (Gegensatz) zwischen Ruhe und Bewegung machen, d.h. uns bei der Betrachtung von Bewegungsvorgängen festlegen, wer ruht und wer sich bewegt.
Beispiel: Kein Mensch würde verstehen, dass in Anwendung des Relativitätsprinzips der PKW-Fahrer als ruhend und der Polizeibeamte mit seinem Radargerät als bewegt angesehen werden kann, und der Polizeibeamte deshalb die Geldbuße wegen Geschwindigkeitsüberschreitung zu zahlen hat.

Der Grund für die erforderliche Festlegung (Orientierung) liegt letztlich darin, dass wir uns nur so verständigen können. Insoweit macht die Mathematik, auch wenn sie nicht mit begrifflichen Ungenauigkeiten belastet ist, bei Anwendung auf die Wirklichkeit keine Ausnahme.

Beispiel: Wenn wir uns den Zahlenstrahl bei Anwendung auf die Wirklichkeit als einen Stab vorstellen gehen wir so vor, dass wir einen Punkt festlegen (Null) und zwei entgegengesetzte Richtungen, rechts plus, links minus.
Wenn wir diesen Stab (Zahlenstrahl) aus einer entgegengesetzten Position, von der anderen Seite betrachten, kehren sich rechts und links, plus und minus um. Bei dieser Änderung der Perspektive ändert sich an dem Stab garnichts. Der Stab als solcher hat keinen Nullpunkt (das Universum hat keinen Mittelpunkt) und der Stab als solcher hat auch keine entgegen gesetzten Richtungen. Um uns orientieren und verständigen zu können, müssen wir sowohl ein Orientierungssystem (Null, Richtungen) schaffen wie auch unsere Perspektive festlegen. An dieser Notwendigkei ändert m.E. auch die Beschreibung der Natur mit Hilfe der Mathematik nichts.

MfG
Harti

Knut Hacker
18.09.11, 15:06
Die Zukunft lässt hoffen. auf die Mausies die man da noch so kennenlernt.


Aber nur, weil die Schrödingersche Katze damit beschäftigt ist, sich nicht zwischen Leben und Tod entscheiden zu können

Knut Hacker
18.09.11, 15:34
Ein Beobachter kann nichts Gegenwärtiges registrieren, sondern immer nur Vergangenes. Denn eine Registrierung exakt scharf in der Gegenwart kann nicht stattfinden, weil das die quantenmechanische Energie-Zeit-Unschärfe nicht zulassen würde. Also kann der Beobachter für die mögliche Existenz oder Nicht-Existenz einer Gegenwart prinzipiell nicht verantwortlich sein.



Hinzu kommt aus neurobiologischer Sicht, dass zwischen dem Sinnesreiz, der die Wahrnehmung auslöst, und dessen Bewusstwerdung und Verarbeitung (Interpretation) im Gehirn ein Sekundenbruchteil vergeht, um den wir also in der Vergangenheit leben.
Schon logisch erscheint jeder Zeitpunkt und damit die Zeit insgesamt paradox.Denn wenn man nicht von einer Quantelung der Zeit ausgeht,lässt sich jeder wenn auch noch so kurze Zeitraum unendlich teilen, zwischen Zukunft und Vergangeheit besteht daher keine erfassbare Grenze, die die Gegenwart ausmachen könnte.Die Zukunft mündet übergangslos in die Vergangenheit.
Paralleles haben wir beim Raum.Jeder Raumbereich ist unendlich teilbar. Ein Raumpunkt hat null Dimensionen.Keine Raumbereich lässt sich vom anderen durch eine räumliche Grenze abgrenzen.Die Grenzen sind zweidimensional.

fossilium
18.09.11, 15:48
Hi Knut,

Die Verdienste der westlichen reduktionistischen Denkweise - manifestiert in der heutigen Technik - sind unbestritten. Aber wir sind eben durch die Quantentheorie an die Grenzen dieses Erkenntnisweges geraten.

Wieso sind wir durch die Quantentheorie an die Grenzen dieses Erkenntnis w e g e s gelangt? Die Quantentheorie ist ein Modell, mit der wir bestimmte Ausschnitte der Wirklichkeit halbwegs gut beschreiben können. Nach dieser Theorie ist die Wirklichkeit offenbar so gestrickt, dass immer, wenn wir sie beobachten, etwas im Verborgenen bleibt. Nun gut, das ist vom Prinzip her nichts Beunruhigendes (wenn nicht etwas Alltägliches) - wie man das auch auffasst, es beeinträchtigt jedoch nicht die Herangehensweise an die Fragen, z.B. dass wir uns jetzt über das Verborgene keine Gedanken mehr machen können, z.B. wie das Beobachtete und das Verborgene in Einzelheiten zerlegt werden könnte, oder dass nun die Synthese der Analyse vorausgehen müsste. Warum sollte Deiner Meinung nach der Erkenntnis w e g davon berührt sein.

Superposition und Verschränkung lassen sich nicht reduktionistisch erfassen..
Superposition wird durch die Zusammensetzung von Einzelnem beschrieben.
Ein Musterbeispiel für Reduktionismus. Bezgl. Beschränkung ist Deine Behauptung eine Tautologie. Was Eins ist, kann von sich aus nicht in Einzelteile zerlegt werden, es geht definitionsgemäss nicht. Also: kein Argument.

Ich kann Dich verstehen, wenn Du in Sorge bist, dass der Trampelpfad des Reduktionismus in die Sackgasse führt - aber zwei Sachen stehen dagegen:
a) es ist nicht nicht so weit,
b) der Mensch ist so gestrickt, dass er es in denkerischen Sackgassen nicht lange aushält. Wie gesagt, die Evolution hat ihn mit Qualitäten ausgestattet, immer nach neuen Wegen zu suchen. Die wird er finden.
Wir müssen uns eben in Geduld üben.
Mahner wie Dich brauchen wir natürlich auch, um nicht zu viele Umwege zu gehen (wie die String- und Viele Welten Theoretiker). Aber das nur am Rande.
Grüsse Fossilium

fossilium
18.09.11, 15:59
Hi Knut,

Die Frage, ob alles durch Denken, Fühlen und Werten Wahrgenommene lediglich Konstrukt unseres Bewusstseins ist (und sogar auch die Prämisse eines vorhandenen Bewusstseins), oder ob das Bewusstsein eine Außenwelt widerspiegelt, wenn auch lediglich selektiv, abstrakt und adaptiv, ist müßig, da wir unser Bewusstsein nicht verlassen können und daher nur das als Realität wahrnehmen können, was uns unser Bewusstsein als solche ausgibt.

Es war nicht die Frage, was die Gedanken sind, sondern worin der Unterschied zwischen Gedanken und Aussenwelt besteht ? Ist das nicht ein ähnlicher Unterschied wie der zwischen Geist und Materie. Es war doch Deine zentrale Aussage:

"Das begriffliche Denken, insbesondere das Denken in Gegensätzen ist nicht geeignet, die Natur in ihrer Elementarität zu erfassen."

Hattest Du mit dem Denken in Gegensätzen auch den Unterschied zwischen Geist und Materie gemeint ? So als wäre Geist und Materie kein Unterschied mehr in der Physik ? Weil der Substanzbegriff nach dem Welle/Teilchen/Quanten-Begriffsdebakel seine Griffigkeit verloren hat ?

Daher sag uns, welchen Unterschied es früher vermeintlich gab und heute nicht mehr. Also da wäre auch begrifflich klar und eindeutig zu argumentieren.
Gruss Fossilium

fossilium
18.09.11, 16:15
Hi Knut, Hi Bauhof,
es gibt zwei Zeiten: die menschliche, die kennt Vergangenheit, Gegenwart und Zukunft (alle drei sind gehirnpysiologisch begründbar). Die physikalische: das ist eine Abfolge von Ereignissen ohne Gegenwart, Vergangenheit, und Zukunft. Gegenwart, Vergangenheit und Zukunft sind menschliche Wertungen, die in der Natur eins nach dem anderen stattfindenden Prozessakte kennen diese nicht. Und die Natur hat mit dem Wirkungsquntum dem unendlich Kleinen und zeitlich Kurzen einen Riegel vorgeschoben. Das ist nur für den Menschen mit seiner Drang zum Überwinden aller Grenzen ein Problem. Für die Natur nicht. Dafür gibt es da Zufälle, die könnten ein Problem sein, meine ich.
Grüsse Fossilium

Harti
18.09.11, 16:48
es gibt zwei Zeiten: die menschliche, die kennt Vergangenheit, Gegenwart und Zukunft (alle drei sind gehirnpysiologisch begründbar). Die physikalische: das ist eine Abfolge von Ereignissen ohne Gegenwart, Vergangenheit, und Zukunft.

Hallo fossilium,

das sehe ich auch so. Den physikalischen Zeitbegriff könnte man begrifflich als reine Dauer konkretisieren, wie sie durch den periodischen Takt von Uhren dargestellt wird. Für diese Dauer spielen die kausalen Abläufe keine Rolle. Es ist für die reine Dauer egal, ob ein Apfel vom Baum auf die Erde oder von der Erde an den Baum "fällt".
Mir ist klar, dass "Dauer" in diesem Zusammenhnag nur ein anderes Wort für Zeit ist und das Wesen von Zeit nicht näher beschreibt. Überlegungen dazu würden zu sehr vom Thema wegführen.
MfG
Harti

EMI
18.09.11, 16:57
...der Mensch ist so gestrickt, dass er es in denkerischen Sackgassen nicht lange aushält. Wie gesagt, die Evolution hat ihn mit Qualitäten ausgestattet, immer nach neuen Wegen zu suchen. Die wird er finden. Wir müssen uns eben in Geduld üben.Das sehe ich genau so fossi...

Gruß EMI

Knut Hacker
18.09.11, 18:14
Wieso sind wir durch die Quantentheorie an die Grenzen dieses Erkenntnis w e g e s gelangt? Warum sollte Deiner Meinung nach der Erkenntnis w e g davon berührt sein.


Der Erkenntnisweg bestimmt sich nach dem Erkenntnisziel. Wenn wir wissen wollen, „was die Welt im Innersten zusammenhält“, werden wir die Forschung reduktionistisch ausrichten.Das tun die Naturwissenschaften. Wenn wir wissen wollen, „was Welt ist“, werden wir alles, vor allem das uns selbstverständlich Erscheinende, hinterfragen. Das tun die Philosophen.
Beide Wege führen wegen der Selbstbezüglichkeit unseres Denkens in letzter Konsequenz zu Paradoxien. Außerdem ist unser Denken ein System, aus dem wir nicht ausbrechen können, so dass wir es auch nicht verifizieren können (siehe Gödel).

Ein historisches Beispiel reduktionistischer Erkenntnis::

Der alte Zenon hatte die Bewegung mit dem Argument zu widerlegen versucht, bei jeder Bewegung müssten unendlich viele Teilstrecken in unendlich vielen Zeitabschnitten zurückgelegt werden ( Achilles und die Schildkröte). Gegen dieses Paradoxon der unendlichen Teilbarkeit von Raum und Zeit hat sich Demokrit gewandt, indem er argumentierte, gerade das Paradoxon zeige,dass es keine unendliche Teilbarkeit geben könne. Es gebe Grenzen der Teilbarkeit, letzte Elemente seien der leere Raum und das Atom ( grch. To átomon= das Unteilbare). Aristoteles hat darauf hingewiesen, dass es sich um ein Scheinproblem handele. Man könne zwar unendlich oft teilen, aber man hinterlasse dabei immer endlich viele Teile. So wie man unendlich zählen könne, aber immer mit endlichen Zahlen.

Die Ouantenphysik hat nun* eine experimentell absolut abgesicherte, völlig überraschende Lösung des Problems gefunden. Es gibt keine unendliche Teilbarkeit der Materie, vielmehr löst sich diese letztlich in Potentialität, Information auf, und unterhalb der sogenannten Planck´schen Mindestgrößen ist es überhaupt sinnlos, noch von etwas zu sprechen.

Die modernen Naturwissenschaften rütteln also an unsere traditionellen Denkvorstellungen, mit denen bisher Philosophie betrieben worden ist. Der reduktionistische Erkenntnisweg der Quantenphysik führte zu holistischen Aspekten. Er ist an seine Grenzen gestoßen..

Umgekehrt ist auch die Gesamtschau der Philosophie an ihre Grenzen angelangt. Ihr liegt immer noch das begriffliche Denken zugrunde, das in den modernen Naturwissenschaften längst nicht mehr geeignet ist, die gewonnenen Erkenntnisse zu beschreiben. Die Selbstbezüglichkeit dieses Denkens wird offenbar bei der philosophischen Grundfrage, warum überhaupt etwas ist. Die Frage ist selbstbezüglich, weil Gründe als etwas bereits Seiendes das Sein bereits voraussetzen und daher für das Sein selbst nicht gelten können.



Superposition wird durch die Zusammensetzung von Einzelnem beschrieben.
Ein Musterbeispiel für Reduktionismus.

Wieso? Die Schrödingerschen Wellengleichungen sind doch nicht- lokal!


Bezgl. Beschränkung ist Deine Behauptung eine Tautologie. Was Eins ist, kann von sich aus nicht in Einzelteile zerlegt werden, es geht definitionsgemäss nicht.
Eben, auch hier die Nichtlokalität!
Wie hast du mich denn verstanden?



Ich kann Dich verstehen, wenn Du in Sorge bist........
Mahner wie Dich...

Entschuldige bitte, ich mag ein *****loch( gestirnt durch das Forum) sein, aber auf keinen Fall ein Besorgter oder Mahner ( was die Naturwissenschaften betrifft)!!!!!!!!!!!

Die Quantenphysik ist ein riesiger Erkenntnisgewinn im Sinne eines über sich hinausweisenden Denkens. Zu Recht spricht man von einem Paradigmenwechsel.Es wäre schlimm, wenn die Welt so primitiv wäre, dass sie in unsere Gehirne passte. Gäbe es die Weltformel X., müsste immer noch gefragt werden, warum es eine Weltformel gibt und warum gerade diese und keine andere. Gott sei Dank stecken wir nicht in einer Zwangsjacke letzter Antworten und in einem Sinngefängnis, wie dies die klassische Physik glaubte ( Laplace´scher Dämon ) und die Religionen heute noch glauben (Gott als Hampelmann unsrer Vorstellungen).

EMI
18.09.11, 18:33
...und unterhalb der sogenannten Planck´schen Mindestgrößen ist es überhaupt sinnlos, noch von etwas zu sprechen.Das ist ein Postulat Knut,

dies folgt aus keinem einzigen Gesetz der Physik und aus keiner einzigen Messung/Experiment.
Meiner Ansicht nach ein adhoc, aus der Luft gegriffenes Postulat.

Gruß EMI

Knut Hacker
18.09.11, 18:41
Hi Knut,

Es war nicht die Frage, was die Gedanken sind, sondern worin der Unterschied zwischen Gedanken und Aussenwelt besteht ? Ist das nicht ein ähnlicher Unterschied wie der zwischen Geist und Materie. Es war doch Deine zentrale Aussage:

"Das begriffliche Denken, insbesondere das Denken in Gegensätzen ist nicht geeignet, die Natur in ihrer Elementarität zu erfassen."

Hattest Du mit dem Denken in Gegensätzen auch den Unterschied zwischen Geist und Materie gemeint ? So als wäre Geist und Materie kein Unterschied mehr in der Physik ? Weil der Substanzbegriff nach dem Welle/Teilchen/Quanten-Begriffsdebakel seine Griffigkeit verloren hat ?

Daher sag uns, welchen Unterschied es früher vermeintlich gab und heute nicht mehr. Also da wäre auch begrifflich klar und eindeutig zu argumentieren.
Gruss Fossilium

Ich weiß wirklich nicht recht, worauf du hinaus willst.

Descartes unterschied doch zwischen „res cogitans“ und „res extensa“. Diese Unterscheidung muss spätestens nach dem Mess- Problem in der Quantenphysik aufgegeben werden. Zuvor war sie schon im philosophischen Idealismus mega out ( von Berkeley über Kant und Schopenhauer bis zu Hegel, Fichte und Schelling).

Der Dualismus Geist-Materie ist auf Platon und Aristoteles zurückzuführen.Auch er ist spätestens durch die Quantenphysik in dem Sinne überholt, als die Grenzen zwischen Materie (die ja, wie ausgeführt, noch nie brauchbar definiert werden konnte), und Nicht- Materie im Elementaren zerfließen, indem das Quantenfeld nur noch als Potentialität oder Information verstanden werden kann.(„Geist“ war noch nie definiert, man verstand alles darunter, was nicht „Materie“, das heißt „nach Maß, Zahl und Gewicht bestimmbar“ - so Newton- oder „raumzeitlich wohldefiniert“ - so Max Planck -war.)

Nicht nur das Denken in Gegensätzen, sondern überhaupt das begriffliche Denken ist doch überholt.
Daher werden die Erkenntnisse der modernen Naturwissenschaften in der Sprache der Mathematik ausgedrückt.

Muss leider aus Zeitgründen für heute schließen.

Knut Hacker
18.09.11, 18:54
Das ist ein Postulat Knut,

dies folgt aus keinem einzigen Gesetz der Physik und aus keiner einzigen Messung/Experiment.
Meiner Ansicht nach ein adhoc, aus der Luft gegriffenes Postulat.

Gruß EMI

Diese Aussage stammt von Werner Heisenberg in seiner "Physik und Philosophie" und wird auch zitiert vo Stephen Hawking in seinem neuen Buch:

Wörtlich heißt das Zitat:"..... die Planckschen Mindestgrößen markieren die Grenze, jenseits derer sinnvolle Aussagen nicht mehr möglich sind."

sinngemäß auch bei Albert Einstein/Leopold im Feld, die Evolution der der Physik im Kapitel über das Raum-Zeit-Kontinuum.

fossilium
18.09.11, 21:34
Hi Knut,

den reduktionistischen Weg als erfolglos zu bezeichnen, ist Dein gutes Recht. Aber dann verstehe ich nicht, wieso die Quantenphysik da eine Lösung gefunden hat, mit Deinen Worten:

"Die Ouantenphysik hat nun* eine experimentell absolut abgesicherte, völlig überraschende Lösung des Problems gefunden. Es gibt keine unendliche Teilbarkeit der Materie, vielmehr löst sich diese letztlich in Potentialität, Information auf, und unterhalb der sogenannten Planck´schen Mindestgrößen ist es überhaupt sinnlos, noch von etwas zu sprechen."

Es stellt sich doch sofort die Frage, warum sind die Plankschen Grössen so wie sie sind. Was ist der Grund dafür, das die Materie nicht teilbar ist? Den letzten Grund zu hinterfragen hat die Quantenphysik ja gar nicht abgeschafft.
Auch frage ich mich, worin sich die Materie auflösen soll. Zu behaupten, das sei etwas Substratloses, wodurch der Gegensatz zw. Geist und Materie sich quasi aufheben würde, setzt schon ein Verständnis dieses Substratlosen voraus, welches doch mit der überraschenden Lösung der Quantenphysik gar nicht mehr benötigt wird (oder ist doch noch mal ein allerletzter reduktionistische Erklärungsversuch erlaubt) ?

Aber das Wichtigste ist doch: was schlägst Du vor ? Welchen anderen Weg - statt den des Reduktionismus - willst Du gehen ? Wie willst Du die passenden Begriffe für das Substratlose finden ? Wie willst Du vermeiden, dass wir immer wieder in die Falle der Paradoxien tapsen ?

Grüsse Fossilium

fossilium
18.09.11, 21:56
Hi Knut,

man kann der Meinung sein, dass es keinen Sinn macht, Physik bei Größenordnungen unterhalb der Plank-Grössen zu betreiben. Aber was folgt daraus ?

Dass es keinen Sinn macht, darüber zu reden ? Wir reden doch super sinnvoll darüber. Und dann denk mal an das Mathematisch-Unendliche. Macht es keinen Sinn über Unendliches in der Mathematik zu reden. Da war Castor ganz anderer Ansicht. Also das Unendliche war da auch keine Grenze, hinter der sich nichts Sinnvolles mehr offenbart hat.
Knut, es ist eher sinnlos, zu behaupten, sinnvolle Physik höre da und da auf. Kein Mensch wird aufhören, an einer solchen Grenze weiter zu denken, wo es angeblich keinen Sinn macht, nur die, die dem Verkünder solcher Grenzen ohne weiter nachzudenken glauben.

Der Mensch (sein Verstand) kann nicht anders als immer weiter zu fragen, das ist stärker als alle Grenzen. Es mag also im Moment keinen Sinn machen, hinter die Plank-Grenzen weiter zu fragen, aber glaubst Du, dass das so bleibt für alle Zeiten ? Ich nicht.

Grüsse Fossilium

amc
18.09.11, 22:29
Es gibt keine unendliche Teilbarkeit der Materie ... unterhalb der sogenannten Planck´schen Mindestgrößen ist es überhaupt sinnlos, noch von etwas zu sprechen.

Hallo Knut,

ich bin geneigt zu sagen: Es hackt auf dich ein ;) - Hier ein paar Anmerkungen meinerseits dazu. Ich weiß nicht ob EMI darauf hinauswollte - ich denke es ist ein feiner, aber nicht unbedeutender Unterschied zu sagen:

unterhalb der sogenannten Planck´schen Mindestgrößen ist es überhaupt sinnlos, noch von etwas zu sprechen
oder
die Planckschen Mindestgrößen markieren die Grenze, jenseits derer sinnvolle Aussagen nicht mehr möglich sind

Ob es sinnlos ist, noch von etwas zu sprechen, wissen wir nicht, aber wir können keine sinnvollen Aussagen darüber machen.

Es gibt keine unendliche Teilbarkeit der Materie, vielmehr löst sich diese letztlich in Potentialität, Information auf ...

Hier würde ich gerne einwerfen, dass ich den Begriff der Information anders verstanden habe. Nicht der potentielle Aspekt unserer Welt, sondern der wahrhaftige, der gemessene bzw. prinzipiell messbare Teil ist Träger von Information. Sobald es konkrete Informationen über die Zustände eines Systems gibt, lösen sich die Superspositionszustände auf. Ich meine von Zeilinger z.B. nie gehört zu haben Alles ist nur Information. Der Begriff der Information ist auf untrennbarste Weise mit dem Begriff der Wirklichkeit verknüpft. Das Wesen der Potentialität, und die Frage ob, und inwiefern, sie Teil unserer Wirklichkeit ist, ist nach meinem (Halb)Wissen noch nicht hinreichend geklärt.

Ich für meinen Teil gebe dir absolut recht, der Reduktionismus ist IMHO an seine Grenzen gestoßen ist. Letzendlich bleibt von der Materie nach klassischer Definition nichts wirklich greifbares übrig, wenn man nur weit genug ins Detail schaut. Das heißt ja noch lange nicht, dass hier nur etwas geistiges übrig bleibt, ich bin mir sicher, dass du dies auch gar nicht meintest.

Auch etwas, was man als geistig bezeichnen könnte, würde sich IMHO auch durch gewisse materielle Eigenschaften erkennbar machen. So sehe ich es. Wenn es etwas gäbe, was sich nicht bemerkbar macht, dann kann uns das völlig kalt lassen. Wie wir es ja bereits kennen - Kräfte sind materiell, Energie ist materiell, Wirkungen, Bindungen, Zustände - alles besitzt Eigenschaften, ist somit materiell. Ich denke man kann statt materiellem auch einfach wesenhaftes, oder beschreibbares Dings sagen. Ein gutes Beispiel ist das Proton, es wiegt mehr als die Teile aus denen es besteht, die Bindungen der Teile besitzen Masse.


Freundlichst,
AMC

Knut Hacker
19.09.11, 13:15
den reduktionistischen Weg als erfolglos zu bezeichnen, ist Dein gutes Recht.

Wo habe ich dies denn um Himmels willen getan? Ich bin schon ziemlich down, dass ich mich offensichtlich so ausdrücke, dass das Gegenteil von dem bei dir rüberkommt, was ich meine.

Naturwissenschaften sind nach ihrem Selbstverständnis (und ihrer kognitiven Selbstbescheidung), die Natur so abstrakt zu beschreiben, dass Grundzüge der Naturvorgänge hervorgehen, die in ihrer Grundsätzlichkeit auch Entwicklungsprognosen erlauben (Naturgesetze), reduktionistisch. Diese Forschungsmethode hatte in der Quantenphysik ( wie für den makroskopischen Bereich auch in der Chaosforschung) zu dem unübertroffenen Erfolg führt, dass die Natur nicht reduktionistisch aufgebaut ist. Der Quantenphysiker Hans-Peter Dürr bringt dies so auf den Nenner: „Materie ist nicht aus Materie zusammengesetzt. Atome und ihre Bausteine haben nicht mehr die Eigenschaften von Materie. Es sind reine Gestaltwesen. Das heißt: wir haben auf einmal im Vergleich zur klassischen Vorstellung eine Umkehrung der Rangordnung.“ Der Quantenphysiker Carl Friedrich von Weizsäcker fasste es noch knapper zusammen:Das Fundamentale an der Quantenphysik sei, dass „sie Bedingungen der Möglichkeit der Erfahrung formuliert“

Nun wirst du sofort auf den Widerspruch zur allgemeinen Relativitätstheorie hinweisen. Hierzu führt der Quantenphysiker Vlatko Vedral in „Spektrum der Wissenschaft“, September 2011, Seite 38, aus: „...gehören Raum und Zeit zu den fundamentalsten klassischen Begriffen, aber gemäß der Quantenmechanik sind sie sekundär. Primär sind die Verschränkungen; sie verknüpfen Quantensysteme ohne Bezug auf Raum und Zeit.....Die allgemeine Relativitätstheorie unterstellt, dass Objekte stets eindeutig definierte Orte einnehmen und sich niemals an zwei Stellen gleichzeitig aufhalten – im direktem Widerspruch zur Quantenphysik. Stephen Hawking... und viele andere Physiker glauben, dass die Relativitätstheorie einer fundamentalen Theorie weichen muss, in der es weder Raum noch Zeit gibt. Die klassische Raumzeit geht demnach durch den Vorgang der Dekohärenz aus quantenmechanischen Verschränkungen hervor. Noch interessanter ist die Möglichkeit, dass die Gravitation gar keine eigenständige Kraft ist, sondern durch Quantenfluktuationen der übrigen Naturkräfte entsteht...“

Der Reduktionismus auch in der Quantenphysik hat also zu einem Holismus in der Wissenschaft geführt, der geeignet erscheint , dem großen Ziel einer vereinigten Feldtheorie näher zu kommen.




Den letzten Grund zu hinterfragen hat die Quantenphysik ja gar nicht abgeschafft.



Das ist nicht Thema von Wissenschaft. Es ist eine philosophische-theologische Frage. Ich habe doch ausdrücklich geschrieben: Einen letzten Grund kann es schon logisch nicht geben, weil er etwas Seiendes ist und daher für das Sein selbst nicht gelten kann.Außerdem habe ich geschrieben: Selbst wenn es eine Weltformel, also eine Formel, die den letzten Grund beschreibt, gäbe, könnte sie sich nicht selbst begründen, weil die Fragen offen blieben, warum es einen letzten Grund gibt und warum gerade diesen und nicht einen anderen.

Warum gehst du darauf nicht ein?




Wie willst Du vermeiden, dass wir immer wieder in die Falle der Paradoxien tapsen ?



Wieder hast du überlesen, dass ich das doch gerade nicht will. Ich will keine so primitive Welt, dass sie in unsere Köpfe passt. Die Erkenntnis, dass unser Denken immer in Paradoxien führt, ist doch gerade das wichtigste Verdienst in der Quantenphysik. Sie will die Paradoxien doch gerade nicht überwinden und hat daher die Quantenlogik, das heißt die Komplementarität von Gegensätzen entwickelt.

Das hat übrigens schon vorher die Philosophie geschafft, von den Vertretern des philosophischen Skeptizismus im Altertum bis hin zu Hans Albert mit seinem Münchhausen Trilemma.




man kann der Meinung sein, dass es keinen Sinn macht, Physik bei Größenordnungen unterhalb der Plank-Grössen zu betreiben. Aber was folgt daraus ?

es ist eher sinnlos, zu behaupten, sinnvolle Physik höre da und da auf.

Die Planckschen Mindestgrößen sind aus dem Planckschen Wirkungsquantum h abgeleitet und daher so definiert, dass es sich um letzte (physikalische) Aussagen über diese Größen handelt. Beispiel: Die Planck´sche Mindestlänge besagt, dass jenseits von ihr keine räumlich messbare Identität mehr besteht. Messobjekte „verschwimmen“.

fossilium
19.09.11, 13:16
Hi Knut,


Nicht nur das Denken in Gegensätzen, sondern überhaupt das begriffliche Denken ist doch überholt.

Ich glaube nicht, dass das Denken in Gegensätzen überholt ist, denn nur aus Gegensätzen entfaltet sich das ganze Universum. Aber möglicherweise meinst Du hier die Gegensätze aus der zweiwertigen aristotelischen ja/nein-Logik, die überholt ist, seitem wir wissen, dass physikalische Systeme in mehrere überlagerten Zustanden existieren können. Dieses Argument wird ja häufig vorgebracht und ich meine auch Du hast es an anderer Stelle angeführt.

Mal unterstellt, dass das so ist, möchte gerne dazu ein paar Anmerkungen machen, um die zweiwertige Logik für uns zu retten:

Im Alltag kennen wir jede Menge Zustände, die sozusagen unscharf sind, denen wir aber dennoch eine eindeutige Bestimmtheit zuweisen: ab wann ist ein Glas voll ? Ab wann ist ein Auto alt ? Ab wann kommt jemand zu spät ? Ab wann ist das Wetter schön ? Keine dieser Fragen lässt sich präzise beantworten. Das ist auch nicht nötig, denn es kommt gar nicht auf die Bestimmtheit einer Eigenschaft oder Ereignisses, oder die Bestimmtheit des zu beurteilenden Zustandes, an. Wir sind in der Lage, trotz dieser Unbestimmtheiten eine eindeutige Antwort zu geben, indem wir beim Antwortgeben die Vielzahl der erfahrenen Möglichkeiten und Wahrscheinlichkeiten zu einer asymptischen Sicherheit gedanklich reduzieren. Wir fällen eine zweiwertige logische ja/nein-Entscheidung trotz gegenständlicher Unbestimmtheit - und antworten so auf alle diese Frage klar und eindeutig.

Was zählt ist die operationale Brauchbarkeit der ja/nein-Entscheidung, nicht die Frage, ob etwas aktuell so oder so ist. Überträgt man diese Gedanken auf das Problem der Zustandsüberlagerung, so sieht man, dass die Unbestimmtheit überlagerter Zustände und die zweiwertigen Logik sich nicht ausschliessen. Die Alltagserfahrung sagt uns doch: wir müssen es nicht genau wissen. Es reicht doch, wenn wir aus einer Vielzahl von Beobachtungen über die Unbestimmtheit einen Schluss ziehen, der verstanden wird, also zu phsikalischen Randbedingungen führt, mit denen weitergearbeitet werden kann. Warum sollen wir uns mit mehrwertiger Logik abplagen, um eine Bestimmteit zu erreichen, die unnötig ist - ja, den Blick verstellt.

Man muss dabei auch berücksichtigen, dass unsere phsikalischen Modelle pragmatisch ausgerichtet sind. Sie sollen nicht die Wirklichkeit erklären, sondern nur Vorhersagen machen.
Nützlichkeit geht vor Wahrheit.
Grüsse Fossilium

Knut Hacker
19.09.11, 13:25
Ich meine von Zeilinger z.B. nie gehört zu haben Alles ist nur Information.



„Anton Zeilinger ist Professor am Institut für Experimentalphysik der Universität Wien, zählt zu den international bedeutendsten Quantenphysikern der Gegenwart. Seine „Teleportationsexperimente“... haben ihn auch einer breiten Öffentlichkeit bekannt gemacht und das allgemeine Interesse an der Quantenphysik enorm gesteigert“.Dieser Satz stammt aus dem Klappentext seines neuesten Buches: „Einsteins Schleier. Die neue Welt der Quantenphysik“, erschienen im Verlag C. H. Beck Auf Seite 221 führt er aus: „Unsere Grundannahme für die Quantenphysik ist also: 'Das elementarste System entspricht einem Bit an Information.' Es erhebt sich daher nun die Frage, ob aus dieser einfachen Grundannahme irgend etwas Interessantes folgt. Wir werden gleich sehen, dass die drei wichtigsten Eigenschaften der Quantenphysik, die wir bereits vorgestellt haben – nämlich der objektive Zufall, die Komplementarität und die quantenmechanische Verschränkung -, genau auf diese Weise ihre ganz natürliche Begründung finden. Wir werden sehen, dass wir den Zufall im quantenmechanischen Einzelprozess nach wie vor nicht erklären können, aber doch immerhin ein Verständnis dafür bekommen, dass der Zufall nicht weiter erklärbar ist. Insbesondere werden wir nun verstehen, warum es also für das quantenmechanische Einzelereignis in der Regel keinen Grund dafür gibt, warum es so stattfindet, wie es stattfindet.“
Und am angegebenen Ort auf Seite 217: "Information ist der Urstoff des Universums."

Meintest du etwas anderes oder habe ich mich missverständlich ausgedrückt?

Zur Potentialität:

Werner Heisenberg in "Physik und Philosophie", Kapitel X , am Ende:
" In den Experimenten über Atomvorgänge haben wir mit Dingen und Tatsachen zu tun, mit Erscheinungen, die ebenso wirklich sind wie irgendwelche Erscheinungen im täglichen Leben. Aber die Atome oder die Elementarteilchen sind nicht ebenso wirklich. Sie bilden eher eine Welt von Tendenzen oder Möglichkeiten als eine von Dingen und Tatsachen."
Kap. IV: "...dass nicht einmal die Eigenschaft des “ Seins“, wenn man hier überhaupt von Eigenschaft reden will, dem Elementarteilchen ohne Einschränkung zukommt. Es ist eine Möglichkeit oder eine Tendenz zum Sein ."

F.S.C. Northrop, Sterling Professor of Philosophy and Law, Yale Univesity, schreibt in seiner „Einführung in die Probleme der Naturphilosophie“:
„ Die Quantenmechanik, besonders Heisenbergs Prinzip der Unestimmbarkeit (Unschärferelation), ist für die Veränderungen bekannt, zu denen sie in der Erkenntnistheorie des Physikers über die Beziehungen zwischen dem Experimentator und dem Objekt seiner wissenschaftlichen Kenntnisse geführt hat........ dass die Quantenmechanik den Begriff der Potentialität in die Physik wieder eingeführt hat. Dadurch wird die Quantentheorie für die Ontologie und die Erkenntnistheorie wichtig.“

EMI
19.09.11, 15:07
Man muss dabei auch berücksichtigen, dass unsere physikalischen Modelle pragmatisch ausgerichtet sind.
Sie sollen nicht die Wirklichkeit erklären, sondern nur Vorhersagen machen. Nützlichkeit geht vor Wahrheit.Das sehe ich nicht so fossi...,

die letzten Elemente der Physik sind nicht die subjektiven Erlebnisse.
Die ganze Geschichte der Physik führt von subjektiven Erlebnissen, Akustik, Wärmelehre, Optik usw., die zunächst auf unseren Sinnesorganen fussten, immer weiter vom Subjektiven weg. Das Entfernen des Subjektiven führt zu immer umfassenderen Wahrheiten, zu immer allgemeineren Naturgesetzen.
Je abstrakter diese werden um so allgemeingültiger werden sie auch, um so allgemeiner ist der Wahrheitsgehalt.

Die Annahme, es gäbe verschiedene physikalische Weltbilder, das mechanische, das elektromagnetische, das quantenmechanische, die man sozusagen zur Auswahl hätte ist falsch und widerspricht der Einheit der Physik.

"Die Mechanik beschäftigt sich mit Raum und Zeit und der Bewegung von Substanzen in Raum und Zeit, etwas anderes kann die Elektrodynamik und die Quantenmechanik auch nicht tun." So sah es einst ein MAX PLANCK und dem schliesst EMI sich vollumfänglich an, auch wenn das der Stand von vor 100 Jahren ist!

Die Allgemeingültigkeit der Gesetze der Physik schliesst nach PLANCK nicht aus, dass in verschiedenen Gebieten der Physik zu den universellen Bewegungsgesetzen weitere Gesetze hinzutreten, die das spezielle Gebiet charakterisieren. PLANCK kam damals zu folgenden Schlüssen, die ich nur thesenartig angeben kann.

Er zeigte Erstens:
Es gibt nicht eine Beliebigkeit von ambivalenten Weltbildern, sondern es gibt nur eine einzige objektive Wahrheit, der sich unsere Bilder asymptotisch annähern.
Die Behauptung, dass die "Wahl der Bilder" eine Frage der Nützlichkeit und des Geschmacks sei, ist einfach falsch.
Man könnte sonst behaupten, dass es verschiedene konkurrierende Bilder real gibt.
PLANCK sagte dagegen, wenn es verschiedene konkurrierende Bilder gibt, ist bestenfalls eines davon wahr. Es kann passieren, dass alle falsch sind, aber bestenfalls ist eines zutreffend.

Zweitens hob PLANCK hervor, dass bei jeder Revision des Bildes, die notwendig wurde um sich der Realität weiter zu nähern, ein absoluter Wahrheitsgehalt bestehen bleibt.
Die Geschichte der Physik, besteht nicht in einem wilden Wechsel von Bildern, auch nicht in der kontinuierlichen Ablösung der Bilder, sondern es bleibt ein Wahrheitsbestand, der sich ständig vermehrt. Dieser Wahrheitsbestand ist es, den wir gerade dadurch gewinnen, dass wir zu immer allgemeineren und damit auch immer mehr vom Subjektiven abgelösten Naturgesetzen gelangen.

Drittens vertrat PLANCK die Ansicht, wenn dem nicht so wäre, wäre die Physik nichts weiter als die Beschreibung der privaten Erlebnisse jedes einzelnen Physikers.
Alle großen Physiker haben nicht irgend eine zweckmäßige Beschreibung ihrer Erlebnisse gesucht, sondern sie suchten nach objektiven Naturgesetzen, sie sprachen nicht einmal vom Weltbild, sondern vom Bild der Welt.

Gruß EMI

Knut Hacker
19.09.11, 15:11
Im Alltag

Was zählt ist die operationale Brauchbarkeit der ja/nein-Entscheidung Die Alltagserfahrung sagt uns doch: wir müssen es nicht genau wissen.
Man muss dabei auch berücksichtigen, dass unsere phsikalischen Modelle pragmatisch ausgerichtet sind. Sie sollen nicht die Wirklichkeit erklären, sondern nur Vorhersagen machen.


Ich habe doch ausdrücklich betont, dass sich unser begriffliches Denken, insbesondere das Gegensatzdenken, in der Evolution als Überlebensvorteil entwickelt hat. Selbstverständlich hat dieses Denken also Orientierungs- und Lebenserhaltungsfunktion.
Aber die Darwinschen Gesetze gelten ja nur für den Bereich der Lebensbewältigung und Arterhaltung. Das menschliche Bewusstsein ist mit der Besonderheit ausgestattet, sich seiner selbst bewusst zu sein und daher auch das eigene Denken zu hinterfragen. Daraus ist die Philosophie und Naturwissenschaft hervorgegangen. Letztere beschränkt sich definitionsgemäß auf das Wissen, also die rein praktische Erkenntnis. Auch die Quantenphysik erhebt keinen philosophischen Anspruch. Aber die Erkenntnisse drängen zu erkenntnistheoretischen Fragen. Dem sind die Quantenphysiker nicht ausgewichen, trennen aber strikt zwischen ihrem Metier und den philosophischen Schlussfolgerungen.
So hat die Quantenlogik bestimmungsgemäß allein praktische Bedeutung bei der Beschreibung der mit herkömmlichen Begriffen nicht erfassbaren Quantenphänomene. Darüber hinaus kommt ihr aber auch philosophische Bedeutung in der Erkenntnistheorie zu. Schon immer hat man ja in der Philosophie gefragt: „Wieso soll es bloß Ja und Nein geben, das führt doch letztlich immer zu unendlichen Regressen oder Denkzirkeln.“ Alle philosophischen Werke befassen sich ja letztlich damit, dass sich das Denken ab einem bestimmten Punkt der Abstraktion in den eigenen Schwanz beißt.

Es ist bestritten, ob dieses über sich hinausweisende Denken ein Produkt der Evolution ist, oder ob kein erkennbarer Überlebensvorteil damit verbunden ist. Die meisten körperlichen Funktionen werden ja unbewusst gesteuert.Und das Bewusstsein erfasst lediglich einen winzigen Seinsbereich und ist auch da äußerst störungsanfällig. Welchen Nutzen hat es also, dass wir nicht bloße perfekte Automaten sind? Die mit unseren Überbewusstsein verbundene Freiheit macht uns über die Natur überlegen.Aber Freiheit wovon? Überlegenheit wozu?

Knut Hacker
19.09.11, 18:20
um die zweiwertige Logik für uns zu retten:


Die bekannteste Argumentation, die zweiwertige Logik, also das Denken in Gegensätzen. ad absurdum zu führen,stammt von Gorgias von Leontinoi (483-375 vor Christi Geburt). Er hat sich hierfür den abstraktesten Gegensatz, den zwischen Sein und Nichtsein, ausgewählt.

Seitdem ist es noch keinem Denker gelungen, die zweiwertige Logik zu retten.

Hier also der Beweis des Gorgias dafür , dass es überhaupt nichts gibt ( auch nicht seinen Beweis).
Alle diese Beweisführungen sind möglich, weil nach der im Volk immer noch herrschenden, in der modernen Physik aber längst aufgegebenen aristotelischen Logik alles bewiesen werden kann einschließlich jeweils des Gegenteilsteils ( so insbesondere Protagoras, Hegel).

Die griechische Originalfassung der Argumente des Gorgias findet sich in der Reclam-Ausgabe „Die Sophisten“.Überliefert sind sie durch Sextus Empiricus in seinem Werk „Adversus mathematicos“ VII 65ff. Ich bringe sie hier auszugsweise und gegliedert nach Ralf Ludwig:

1) Es kann kein Nichtseiendes geben.

Wenn es das Nichtseiende gäbe, würde es zugleich sein und nicht sein; wenn es als nichtseiend gedacht wird, ist es nicht; wenn es nicht seiend ist, wird es sein. Da es widersinnig ist, dass etwas ist und nicht ist, gibt es das Nichtseiende nicht. Ein weiterer Beweis: Gibt es das Nichtseiende, kann es das Seiende nicht geben, da beide entgegengesetzt sind. Und wenn dem Nichtseinden Existenz zukommt, muss dem Seienden Nichtexistenz zukommen. Da aber das Nichtsein des Seienden ausgeschlossen ist, kann das Nichtseiende nicht existieren.

2) Es kann kein Seiendes gegeben.

Wenn das Seiende existiert, ist es entweder ewig oder geworden oder zugleich ewig und geworden. Aber keine der drei Möglichkeiten ist denkbar.

a) Wenn das Seiende ewig ist, hat es keinen Anfang. In diesem Fall ist es unbegrenzt. Wenn es aber unbegrenzt ist, ist es nirgendwo. Wenn es aber irgendwo sein soll, dann ist es von dem Raum, in dem es ist, verschieden. Dann aber kann das Seiende nicht mehr unbegrenzt sein, da es von einem Anderen umschlossen ist. Denn das Umschließende muss größer sein als das Umschlossene. Wenn aber nichts größer sein kann als das Unbegrenzte, gibt es nirgends ein Unbegrenztes..

b) Das Seiende kann aber auch nicht geworden sein. In diesem Fall wäre es entweder aus dem Seienden oder aus dem Nichtseienden geworden. Aber aus dem Seienden kann es nicht geworden sein, denn wenn es seiend ist, ist es bereits. Auch aus dem Nichtseienden kann es nicht geworden sein, da dieses nichts aus sich hervorbringen kann, weil notwendigerweise ein Hervorbringendes existieren muss. Somit ist das Seiende auch nicht geworden.

c) Es kann aber auch nicht beides sein, zugleich ewig und geworden, da beide Begriffe sich gegenseitig aufheben: denn wenn das Seiende ewig ist, ist es nicht geworden; und wenn es geworden ist, ist es nicht ewig.
Also, wenn das Seiende weder ewig noch geworden noch beides ist, gibt es das Seiende nicht..

3) Es gibt überhaupt nichts.

Wenn das Nichtseiende und das Seiende exisitiert, dann ist das Nichtseiende – hinsichtlich seiner Existenz – dasselbe wie das Seiende, deshalb existiert keines von beiden. Denn dass das Nichtseiende nicht existiert, ist allgemein anerkannt. Da aber bewiesen wurde, dass das Seiende mit dem Nichtseienden identisch ist, existiert es auch nicht. Es kann aber das Seiende, wenn es mit dem Nichtseinenden identisch ist, unmöglich beides sein. Wäre es beides, wäre es nicht identisch; wäre es identisch, wäre es nicht beides. Daraus folgt, dass es nichts gibt.
Denn wenn weder das Seiende noch das Nichtseiende noch beides existiert und außer diesen drei Möglichkeiten keine denkbar ist, dann gibt es überhaupt nichts...


Gorgias hat sich für unsere Zeit etwas unbeholfen ausgedrückt. Ich bin vermessen genug, etwas schärfer zu formulieren:

Das Sein kann nicht sein, weil es:

1) keine Identität besitzt. Denn es umfasst alles und unterscheidet sich daher von nichts, auch nicht vom Nichts, da dies ebenfalls alles umfasst und sich daher von nichts unterscheidet, so dass es ebenfalls keine Identität besitzt.

2) sich sonst selbst voraussetzen würde. Denn es müsste Gegenstand eines Seins sein, das wiederum Gegenstand eines Seins sein müsste usw. (unendlicher Regress).

3) alles und daher auch das Nichts umfassen müsste,das aber wiederum das Sein umfassen müsste usw. (Denkzirkel).

4) das Nichts als sein Gegenteil ausschließen müsste, das Nichts aber nicht möglich sein kann, da es sonst ja etwas Seiendes wäre, das vom Sein nicht ausgeschlossen wäre.

5) die Menge alles Seienden wäre. Diese kann es aber nicht geben, da sie sich auch selbst miteinschließen müsste, dann aber nur Teilmenge wäre, die wiederum von der Menge alles Seienden umfasst werden müsste,die sich aber wiederum selbst mit umfassen müsste usw.

6) die Unterscheidung zwischen Sein und Nichtsein erst im Sein gibt. Daher kann das Sein als solches weder sein noch nicht sein.

fossilium
19.09.11, 22:56
Hi Knut,

ich hab noch mal Deine Kernthesen zusammengestellt (hoffentlich richtig):

Diese Forschungsmethode hatte in der Quantenphysik (…) zu dem unübertroffenen Erfolg geführt, dass die Natur nicht reduktionistisch aufgebaut ist.

Die Erkenntnis, dass unser Denken immer in Paradoxien führt, ist doch gerade das wichtigste Verdienst in der Quantenphysik. Sie will die Paradoxien doch gerade nicht überwinden und hat daher die Quantenlogik, das heißt die Komplementarität von Gegensätzen entwickelt.

Aber wir sind eben durch die Quantentheorie an die Grenzen dieses Erkenntnisweges geraten. Superposition und Verschränkung lassen sich nicht reduktionistisch erfassen.

So hat die Quantenlogik bestimmungsgemäß allein praktische Bedeutung bei der Beschreibung der mit herkömmlichen Begriffen nicht erfassbaren Quantenphänomene. Darüber hinaus kommt ihr aber auch philosophische Bedeutung in der Erkenntnistheorie zu.

Hm, hm, Du mutest der Quantenphysik ganz schön viel zu. Eine besondere erkenntnistheoretische Bedeutung der Quantenphysik wird vielfach behauptet. Ich persönlich schätze diese aber nicht ganz so hoch ein: mit Unschärfen und Überlagerungszuständen konnten wir (auch im Alltag) schon immer gut umgehen, die Grenzen der Logik und alle Paradoxien waren auch vorher bekannt (sie haben Erkenntnisdrang und -fähigkeit nie behindert), Dualismen und Sprünge gab es auch schon vorher (z.B. elektr. Ladungen, magn. Pole, stehende Wellen), und vor grundsätzlich Neuem standen auch Faraday und Hertz, nicht erst die alten Quantenphysiker und Zeilinger. Das soll die Leistung der Quantenphysiker überhaupt nicht schmälern, versteh mich bitte nicht falsch. Aber letztendlich ist es Ansichtssache, ob die Erkenntnisse der Quantenphysik (die grossartig sind) zum Paradigmenwechsel taugen.

So ganz vollzogen scheint mir der auch nicht zu sein.

Entscheident ist doch: was ist denn jetzt zu tun ? Was ist die Alternative zum Reduktionismus, zur Anwendung der zweiwertigen Logik, zu all dem alten Zeug, nachdem wir nun zum x-ten Male in der Menschheitsgeschichte (diesmal durch die Quantenphsik) an unsere Grenzen gestossen sind ?

Ist es die Quantenlogik ? Wie sieht die denn aus ?

Es ist doch so, dass hierzu alle möglichen Diskussionen stattfinden, auch hier im Forum. Das ist auch wunderbar, das belebt den Geist. Also her mit neuen Ideen.

Lass uns Deine wissen.

Grüsse Fossilium

fossilium
19.09.11, 23:13
Hi Knut !

Welchen Nutzen hat es also, dass wir nicht bloße perfekte Automaten sind? Die mit unseren Überbewusstsein verbundene Freiheit macht uns über die Natur überlegen.Aber Freiheit wovon? Überlegenheit wozu?

Gut gefragt ! Ich sag ja, die Evolution hat sich ein Bein gestellt, oder der liebe Gott hat vergessen, uns beim Vertreiben aus dem Paradies den Verstand zu stutzen. Jetzt führt dieser sein Eigenleben und wir quälen uns masochistisch mit all den sinnlosen (oder doch sinnhaften ? ) Fragen und Widersprüchen unserer angesichts der Ewigkeit jämmerlichen Existenz.
Wotan Wahnsinn.
Grüsse Fosslium.

amc
20.09.11, 00:36
Meintest du etwas anderes oder habe ich mich missverständlich ausgedrückt?

Hi,
mir ging es darum, den fundamentalen Zusammenhang zwischen dem Begriff der Information und dem der Wiklichkeit hervorzuheben. Da du hier ...

Es gibt keine unendliche Teilbarkeit der Materie, vielmehr löst sich diese letztlich in Potentialität, Information auf ...

... die Begriffe Information und Potential in direkten Zusammenhang stellst. Das Merkmal der Potentialität ist aber gerade ein Fehlen an Information. Und Information ist immer an definiertem Zustand, einem Träger gebunden, und definierter Zustand ist das Gegenteil von Potential. Darum gilt beim Doppelspalt:

1. : Vorhandene Information über den Weg des Photons = keine Interferenz.
2. : Fehlt jegliche Information über den genommenen Weg, so kann dieser nicht mehr als tatsächlich angesehen werden, sondern nur noch als potentiell. Also erscheint das Interferenzmuster.

Materie löst sich in Potential auf, dem kann ich folgen, Materie löst sich in Information auf, ist dagegen kritisch, und verfehlt IMHO den Kern des Phänomens. Zumindest müsste geklärt werden, welche Rolle der Begriff der Information tatsächlich spielt, und wieviel Information noch in der Potentialität steckt.

Jedenfalls wirst du überall lesen, das Information primär mit Wirklichkeit (tatsächlichen Ereignissen), und nicht (wenn dann nur sekundär) mit Potentialität einhergeht. Und ich denke (bin mir nicht sicher), dass Zeilinger dies auch so sieht. Information ist der Urstoff des Universums, dem würde ich voll zustimmen. Das ist das, was ich die ganze Zeit versuche auszudrücken, nur in anderen Worten - alles, was man als seiend ansehen kann, muss Eigenschaften besitzen. Diese Eigenschaften sind mit dem Begriff der Information gleichzusetzen bzw. einhergehend. Erst durch die Eigenschaften der Dinge (Information) wird die Welt zur Welt.

Hier, aus einem Interview (http://www.psychophysik.com/html/re025-zeilinger-anton.html), ein Zitat von Zeilinger, welches meiner Sichtweise entgegen kommt, nämlich dass das, was wir als Information/Zustände/Eigenschaften wahrnehmen (dabei meine ich ausdrücklich nicht auschließlich den bewussten Beobachter), in Wahrheit, in Aneinanderreihung betrachtet, dem Wesen der Wirklichkeit an sich entspricht:

»Quantendinge« haben keine unabhängige Existenz mehr für sich allein ... Letztlich sprechen wir über Eigenschaften des Apparates. Wir haben hier diesen Kristall, dort diesen Laser und sehen, dass jener Detektor das und das macht. Wenn wir hingegen auf der Quantenebene davon sprechen, dass ein Photon auf einen Kristall auftrifft, dann ist das nur ein Bild. Ein »Photon« ist sozusagen ein Ausdruck für den Umstand, dass wir diesen experimentellen Aufbau haben und hinten am Detektor eine bestimmte Beobachtung machen. Sich jetzt ein Teilchen vorzustellen, das durch den Apparat wandert, hilft manchmal beim Nachdenken, ist aber mit größter Vorsicht zu genießen.

So, jetzt nochmal kurz ...

Zur Potentialität ...

Bitte versteh mich nicht falsch. Ich stelle in keiner Weise die herausragende Rolle der Potentialität in Frage, welche durch die Quantenphysik eingebracht wird. Die entscheidende Frage dabei ist nur, wie real, wie sehr Teil unserer Wirklichkeit, ist dieser potentielle Charakter. Diese Frage ist letztlich noch nicht geklärt, und ist sicher eine der interessantesten Fragen überhaupt.

In Anlehnung an Wittgenstein hier dazu auch noch ein Zitat von Zeilinger aus seinem Buch (Einsteins Schleier. Die neue Welt der Quantenphysik), welches den Schlusssatz darstellt:

Die Welt ist alles, was der Fall ist, und auch alles, was der Fall sein kann

Ich folge ihm dabei, dies gilt es aber IMHO zu beweisen.

Bis dann


Freundlichst,
AMC

Knut Hacker
20.09.11, 18:49
ich hab noch mal Deine Kernthesen zusammengestellt

Bitte nicht „meine“!!!
Ich traue mir zwar in der Philosophie eigene Thesen zu, haben doch Platon und Aristoteles die Ansicht vertreten, dass jeder Mensch ein Philosoph sei, sofern er Kind geblieben ist und daher noch staunen kann, also die Welt nicht als selbstverständlich hinnimmt
Aber in der Quantenphysik beschränke ich mich auf die Wiedergabe dessen, was die Quantenphysiker dazu zu sagen haben. Denn ich habe Quantenphysik lediglich im Rahmen eines studium generale studiert.



mit Unschärfen und Überlagerungszuständen konnten wir (auch im Alltag) schon immer gut umgehen

Der fundamentale Unterschied ist doch der, dass es sich dabei um epistemologische Unklarheiten handelt, nicht um ontologische. Das Zufallsprinzip, die lediglich statistische Kausalität, die Nicht-Lokalität, das Messproblem usw. beruhen nicht auf unvollkommenem Wissen, sondern stellen Wesenszüge der Natur dar.

Ich gebe dir recht, dass Parallelen zu Erscheinungen in der klassischen Physik bestehen. Doch erst durch die Quantenphysik ist klar geworden, dass die aus unserem Alltag hervorgegangenen Vorstellungen auf absolute Grenzen stoßen, dass – wie es in den Kognitionswissenschaften heißt – unser Denken strukturdeterminiert ist, während dagegen – wie in der Chaosforschung herausgestellt wird - die Welt, deren Teil wir selbst sind, in ihrer dynamisch- strukturellen Komplexität über uns als ihr Teil hinausgeht. Gödel mit seinen Unvollständigkeitssätzen lässt grüßen: Mangels Metaebene
der Erkenntnis können wir nicht über uns hinausdenken; Wir müssten aber über uns und über der Natur stehen, um diese erfassen zu können.




Entscheident ist doch: was ist denn jetzt zu tun ? Was ist die Alternative zum Reduktionismus, zur Anwendung der zweiwertigen Logik,
Ist es die Quantenlogik ? Wie sieht die denn aus ?



Die Naturwissenschaft ist reduktionistisch definiert, wie ausgeführt, und wird es, um Naturwissenschaft zu bleiben, auch künftig sein. Die Grenzen dieses Denkens kann sie nur hinnehmen, was ja auch in der Quantenphysik mit der Quantenlogik geschieht, denn diese ist komplementär, das heißt es gibt nicht das eine oder das andere von sich Widersprechendem, sondern beides zusammen, das Widersprüchliche sind lediglich Erscheinungsformen von etwas, das dahinter steckt, aber mit unserem Verstand nicht zu erfassen ist.
Die große Aufgabe der Physik ist die Vereinigung der Relativitätstheorien mit der Quantentheorie, die sogenannte allgemeine Feldtheorie. Auch dabei geht man nach wie vor reduktionistisch vor, indem man, um die Singularität des Urknalls (unendliche Gravitation) zu vermeiden, beispielsweise an eine Qantelung der Raumzeit und damit auch der Gravitation als deren geometrische Erscheinung denkt.

Ich komme leider mit den Antworten auf eure Beiträge nicht mehr nach. Ich bin in Zeitnot und kann mich erst übermorgen wieder melden.

Knut Hacker
20.09.11, 19:02
Hallo amc,

wegen Zeitnot komme ich leider vor übermorgen nicht dazu, näher zu antworten. Unsere Differenzen erscheinenmir aber behebbar.
Potenzialität ist mögliche Information, Information ist Voraussetzung für Möglichkeiten. Das Quantenfeld als "physikalisches Nichts" intendiert noch keine bestimmte Information. Diese kommt erst durch die Anreicherung zustande.In der Logik spricht man davon, dass aus Möglichkeiten Wahrscheinlichkeit und aus Wahrscheinlichkeiten Sicherheit wird. In der Quantenphysik spricht man nicht mehr von Sicherheit, sondern negativ von extremen Unwahrscheinlichkeiten, zu denen sich die Unscharfen des Mikrokosmos im Makrokosmos auswachsen ( es ist nicht sicher, dass ein Glas Wasser nicht von sich aus zu sprudeln beginnt, aber extrem unwahrscheinlich).

Harti
20.09.11, 19:46
Die bekannteste Argumentation, die zweiwertige Logik, also das Denken in Gegensätzen. ad absurdum zu führen,stammt von Gorgias von Leontinoi (483-375 vor Christi Geburt). Er hat sich hierfür den abstraktesten Gegensatz, den zwischen Sein und Nichtsein, ausgewählt.

Hallo zusammen,
ich verfolge Eure Diskussion mit großem Interesse, versuche mir aber etwas einfachere Gedanken zu machen.

Könnte man auch sagen, das Denken in Gegensätzen ist ein undifferenziertes Schwarz/Weiß-Denken und deswegen nur eingeschränkt geeignet, Erkenntnisse über eine komplexe Wirklichkeit und eine komplexe Ideenwelt zu gewinnen ?

Ist es nicht auch eine Bewertungsfrage und hängt von dem Kriterium ab, das ich meiner Bewertung zugrunde lege, ob ich z.B. zwei Ideen (z.B. Theorien) als gegensätzlich betrachte ?
Beispiel: Das Verhältnis von SRT zu ART.
Wenn ich die Gravitation als wesentlich ansehe, stehen SRT und ART in einem Gegensatz, weil die eine Theorie die Gravitation einbezieht, die andere aber nicht.
Wenn ich vorrangig das Verhältnis von Raum und Zeit betrachte, ist die SRT als Spezialfall in der ART enthalten.

Die Argumentation, dass ein gedachtes Gegensatzpaar nicht gleichzeitig vorgestellt werden kann, ist für mich trivial. Dies liegt einfach daran, dass ich es mir als Gegensatzpaar denke.

Weil in einem der Beiträge das Paradoxon des Zenon angesprochen wurde, äußere ich mich noch mal dazu.
Ein Gegensatz (Widerspruch) ist bei Paradoxien in aller Regel in den Annahmen verdeckt enthalten.
Auch das Paradoxon des Zenon läßt sich so aufklären. Wenn ich Achilles und die Schildkröte beide als bewegt betrachte, brauche ich ein drittes als ruhend vorgestelltes Bezugssystem, weil mir nur dieses die Geschwindigkeiten der beiden liefert. Mathematisch ist dies in einem kartesischen Koordinatensystem darstellbar, in dem die Bewegungen der beiden als als Geraden mit einer bestimmten Steigung erscheinen und im Schnittpunkt der Geraden überholt Achilles die Schildkröte.
Wenn ich in dauerndem Wechsel zunächst die Schildkröte als bewegt und Achilles als ruhend ansehe und dann Achilles als bewegt und die Schildkröte als ruhend betrachte u.s.w., wechsele ich dauernd das Bezugssystem und konstruiere auf diese Weise einen verdeckten Widerspruch in den Annahmen.
Dass Zenon die Infinitesimalrechnung noch nicht kannte, löst das Paradoxon, dass Achilles die Schildkröte nie erreicht, nicht auf.

MfG
Harti

amc
20.09.11, 22:45
Unsere Differenzen erscheinen mir aber behebbar.

Hi,
sehe ich auch so. Mir war es an dieser Stelle wichtig daruf hinzuweisen, dass die Aussage Materie löst sich in Information auf, nach aktuellem Stand, soweit ich das beurteilen kann, keine physikalische, sondern eine philosophische ist. Aus physikalischer Sicht können wir noch nicht sicher beurteilen, ob der Zustand der Potentialität ein realer Zustand ist, oder lediglich ein konstrukt unseres Verstandes, um zu verstehen, warum die Dinge so geschehen, wie sie geschehen. Ist der Zustand der Potentialität real, so müsste IMHO auch Information in ihm stecken. Andernfalls wären nur tatsächliche Erignisse als real und informationsbehaftet zu bezeichnen.


Freundlichst,
AMC

richy
21.09.11, 20:17
Hi Harti

wechsele ich dauernd das Bezugssystem und konstruiere auf diese Weise einen verdeckten Widerspruch in den Annahmen.So ist es (in etwa) Es wird aber nicht staendig das Bezugssystem gewechselt, sondern die Aufgabe ist so konstruert, dass die betrachteten Zeitintervalle und Ortsintervalle gegen 0 streben. Die Zeit wird ueber die Aufgabenstellung eingefroren, so das sie beim Ueberholvorgang stehen bleibt. Man kann dennoch den Zeitpunkt ueber eine geometrische Reihe berechnen, indem man alle Zeitintervalle aufaddiert. Es snd unendlich viele Intervalle, aber diese werden auch unendlich klein und daher konvergiert die Reihe. Es ist kein Paradoxon und das habe ich mit Knut vor einiger Zeit schon ausgiebig diskutiert. Es wundert mich daher ein wenig, dass er dies nun erneut in seiner Sammlung anfuehrt. Das Gedankenexperiment enthaelt vielleicht den interessanten Aspekt, dass es verschiedene Formen von Unendlichkeit gibt, wenn man dieses konstruierte Koordinatensystem der Schildkroete als deren Realitaet betrachten wuerde.
So ergaebe sich auch auf einem Kreis mit unendlich grossem Radius eine andere Form zu unserer "Vorstellung" der Unendlichkeit. Und dies kann man sogar in der Mathematik fuer die Berechnung unendlicher Integrale praktisch ausnuetzen.
Gruesse

fossilium
21.09.11, 22:18
Hi amc,
Du schreibst:
Hi,
aus physikalischer Sicht können wir noch nicht sicher beurteilen, ob der Zustand der Potentialität ein realer Zustand ist, oder lediglich ein konstrukt unseres Verstandes.
Beschreibt die potentielle Energie eines Systems nicht die Potentialität (für eine Wirkung) ? Oder genauer gefragt: ist die potentielle Energie nicht
d i e Messgrösse für Potentialität - (Energie als Mass für das Potential, etwas zu bewirken) ? Potentielle Energie ist aber eine Strukturgrösse, ähnlich wie "Gleichgewicht", "Raumzeit", "Feld", also alles Grössen, die von Abständen oder Beziehungen der Teile untereinander abhängen. Potentialität wäre demnach eben die generelle Bezeichnung für die Tendenz zu struktureller (im einfachsten Fall abstandsabhängiger)Veränderung, das Mass dafür ist die Energie, die Einheit der Veränderung das Plancksche Quantum. Könnte man das so sehen ?

Grüsse Fossilium

richy
22.09.11, 00:56
Dieser Potentialitaetsbegriff klingt zwar plausibel, anschaulich und vernuenftig, ist aber lediglich eine weitere leere Worthuelse, die aus physikalischer Sicht keinen Schritt weiter fuehrt. Jedliches physikalische Potential ist mit einem physikalischen Feld verbunden. Jedliches physikalisches Feld hat ein Gewicht. Jedliches physikalische Feld ist waegbar. Und damit ist aus physikalischer Sicht auch jede Potentialitaet waegbar und damit physikalisch und im Rahmen einer QM Interpretation damit den realistischen Interpretationen zuzuordnen.

Mit Wortspielerein und umdefinieren (wie es gerade bei WIKI geschieht) kann man weder mathematische noch physikalische Problematiken loesen. Ich betrachte das Dekohaerenzprogramm von u.a. Prof Zeh als einen der groessten Fortschritte der Quantenmechanik in den letzen Jahrzehnten ueberhaupt. Neben den Bellschen Ungleichungen und natuerlich den Arbeiten und Intuitionen der Gruendungsvaeter. Prof Zeh wird bei Wiki ja auch dementsprechend gewuerdigt :-)
http://de.wikipedia.org/wiki/Dieter_Zeh
In der Kuerze liegt die Wurze. Herrlich skuril.

Theoretische Physik ist schoen und gut aber ohne Experimentalphysik nunmal auch heute noch wertlos. Den aktuellen Zustand mit Prof Zeilinger als Experimentalphysiker, Philosophen empfinde ich als eine recht gute Ausgangsbasis um das Dekohaerenzprogramm weiter zu erforschen. Dazu haben wir noch David Deutsch und etliche Experimentalphysiker fuer den Quantenrechner. Ich bin hier recht optimistisch. Und natuerlich auch ausserordentlich gespannt wie man diese Rueckbesinnung zum Realismus, der durch das Dekohaerenzprogramm eingeleitet wurde, letzendlich handeln wird. Die Gedanken der KI aus dem abstrakten Bereich wird man dabei wohl kaum voellig aufgeben und so wird uns eventuell schon Prof. Zeilinger zu einer Welt fuehren, die abstrakte Elemente wie Information und physikalische Elemente wie Gravitation und Energie, EM Welle nicht mehr strikt voneinander trennt.

Wobei ich als eher mathematisch interessierter Mensch diese Dinge dennoch weiter strikt getrennt klassifizieren werde. Und dass beide Klassen existieren wussten ja schon die Urvoelker. So viel neues wird es also doch nicht geben. Nur dass beide Klassen zweifelsohne existieren wird Prof. Zeilinger mit Sicherheit belegen. Also insbesonders die Existenz geistiger, abstrakter Elemente nicht widerrufen sondern bestaetigen.

Gruesse

Knut Hacker
22.09.11, 13:19
Auch das Paradoxon des Zenon läßt sich so aufklären. Wenn ich Achilles und die Schildkröte beide als bewegt betrachte, brauche ich ein drittes als ruhend vorgestelltes Bezugssystem, weil mir nur dieses die Geschwindigkeiten der beiden liefert. Mathematisch ist dies in einem kartesischen Koordinatensystem darstellbar, in dem die Bewegungen der beiden als als Geraden mit einer bestimmten Steigung erscheinen und im Schnittpunkt der Geraden überholt Achilles die Schildkröte.
Wenn ich in dauerndem Wechsel zunächst die Schildkröte als bewegt und Achilles als ruhend ansehe und dann Achilles als bewegt und die Schildkröte als ruhend betrachte u.s.w., wechsele ich dauernd das Bezugssystem und konstruiere auf diese Weise einen verdeckten Widerspruch in den Annahmen.
Dass Zenon die Infinitesimalrechnung noch nicht kannte, löst das Paradoxon, dass Achilles die Schildkröte nie erreicht, nicht auf.



Hallo Harti,

Zenon hat vier Paradoxien vorgestellt.Das Grundprobleme dabei ist die Bewegung. Dieses Problem stellt sich auch bei relativer Bewegung ( im "Stadionparadoxon" hat Zenon dies ganz ohne Relativitätstheorie erkannt;abgesehen davon, dass die Relativität der Bewegung auch schon in der klassischen Physik Grundkonsens war).
Das Grundproblem der Bewegung wiederum lässt sich auf das Grundproblem des Kontinuums zurückführen, das seit Aristoteles als unendlich teilbar definiert ist.
Die Infinitesimalrechnung erklärt nichts, sondern beschreibt das Problem lediglich, wie ja überhaupt die Mathematik lediglich deskriptiver Natur ist

Ich habe mir einmal eine Menge Gedanken über dieses Paradoxon (wie auch über andere Paradoxa) gemacht und wage es, sie hier Im nächsten Beitrag (wegen der Begrenzung auf 10.000 Zeichen) vorzustellen:

Knut Hacker
22.09.11, 13:24
Fortsetzung:

Zenons Bewegungsparadoxien(Teilungsparadoxien) lassen sich abstrahierend auf das Problem zurückführen, ob das Ganze aus unendlich vielen Teilen besteht.Ist dies der Fall, dann ist eine Bewegung unmöglich, weil unendlich viele Teilstrecken überwunden werden müssen. Der Vorsprung des Läufers Achilles vor der Schildkröte ist nie aufholbar, weil das Tier in der Zeit, in der der Läufer den ursprünglichen Vorsprung aufholt, schon wieder einen neuen gewinnt, den der Läufer wieder aufholen muss usw..Der Vorsprung wird zwar immer kleiner, entsteht aber unendlich oft immer neu.

Der unendliche Regress ergibt sich dadurch, dass ein abstraktes Reduktionsverhältnis (Ganzes-Teil) auf die Reduktion (Teil) übertragen wird, so dass eine Selbstbezüglichkeit entsteht, die (im Gegensatz zum Zirkel) offen bleibt, weil sie nur ein Element (Teil) des auf sich selbst angewandten Begriffspaares (Ganzes-Teil) betrifft.

Zenons Paradox entsteht nicht, wenn man die Sicht der Schildkröte zugrundelegt. Denn in der Zeit, in der sie ihren ursprünglichen Vorsprung ausbaut, legt der Läufer längst eine darüber hinausreichende Strecke zurück. Der Läufer legt also nicht nur die hinter der Kröte liegenden Teilstrecken zurück, sondern auch! Das Paradox schlägt aus dieser zeitbezogenen Sicht fehl, da sich dabei das Reduktionsverhältnis Zielstrecke/Teilstrecke nicht ergibt.

Das Bewegungsparadoxon Zenons beruht auf zwei abstrakten Prämissen, die den Raum für eine „passende“ Lösung des Problems entsprechend einengen und in den heutigen Naturwissenschaften nicht mehr anerkannt sind, nämlich, dass das Ganze (Laufstrecke) lediglich die Summe seiner Teile sei, und dass Raum und Zeit absolute Größen seien, statische Gegebenheiten, voneinander unabhängig, wobei die Zeit den Raum linear überlagere.
Die Chaos – und Komplexitätsforschung hat die reduktionistische Sicht Zenons insofern widerlegt, als nunmehr anerkannt ist, dass das Ganze gegenüber seinen Teilen qualitativ etwas anderes darstellt, nicht nur quantitativ als deren Summe bestimmt wird.Auf eine Laufstrecke angewandt, bedeutet dies die Selbstverständlichkeit, dass Teilstrecken erst durch Teilung entstehen (Das Ganze besteht nicht aus Teilen, sondern kann in solche zerstört werden), die real nicht unendlich fortgesetzt werden kann (worauf schon Aristoteles hingewiesen hat),sondern nur unbegrenzt oft, wobei aber immer endlich viele Teilstrecken zurückbleiben.Ohne die Teilung ist die Laufstrecke Struktur, nämlich die ausgerichtete Aneinanderreihung von real möglichen und daher endlich vielen Teilstrecken ( also nicht etwa die Nebeneinander -, Übereinander – oder Querlegung von Teilstrecken, die aber ebenfalls nur gedanklich – reduktionistisch – unendlich klein und daher unendlich viele sein können).
Nach der Quantenphysik ist eine Strecke nicht unendlich teilbar. Die kleinstmögliche Strecke ist die Planck– Länge.Alles was darunter liegt, ist nicht mehr differenzierbar.In der Quantenphysik verschwimmt der Mikrokosmos zwischen Sein und Nichtsein. Das Elementarteilchen ist (unbeobachtet) sowohl nirgends als auch überall (Heisenberg´sche Unschärferelation).

Die Teilstrecken, die bei einer Bewegung zurückgelegt werden müssen, sind nicht unendlich klein, sondern unbegrenzt.Alles ist ungenau und alle Grenzen verschwimmen. Gehörten diese zum einen oder zum anderen voneinander Abgegrenzten oder zu beidem, wären sie keine Grenzen, weil sie im Abgegrenzten aufgingen..Gehörten sie weder zum einen noch zum anderen,wären sie ebenfalls keine Grenzen, sondern etwas – wenngleich unendlich teilbares - Drittes zwischen dem Getrennten.Es gibt sie also überhaupt nicht, ebenso wenig wie die Grenzen zwischen Vergangenheit, Gegenwart und Zukunft,wo dies besonders offenkundig wird. Daher ist alles Eines und dieses Eine mangels Identität (es kann ja nicht von etwas anderem unterschieden werden) nichts. Dieses Nichts kann es aber nicht geben, da es sonst ja doch etwas wäre.
Alle Begriffe sind Abstraktionen und daher im Konkreten bei genauer Betrachtung unscharf.

Zenons Laufstrecke besteht daher weder aus unendlich vielen Teilstrecken, noch ist sie in solche teilbar! Vielmehr kann man die Strecke unbegrenzt oft teilen, jedes dabei entstehende Teil ist aber begrenzt.So wie man unbegrenzt zählen kann, dabei aber immer eine endliche Zahl nennt.Die Zahl der Teilstrecken ist – wie der Weltraum - unbegrenzt, aber endlich.Eine Kugeloberfläche ist ebenfalls unbegrenzt aber endlich.Ein weiteres Beispiel zur Veranschaulichung: Gegeben sei ein Quadrat mit der Seitenlänge von einem Meter. Der Inhalt beträgt dann einen Quadratmeter,der Umfang vier Meter. Halbiert man zwei gegenüberliegende Seiten und verdoppelt man dafür die beiden anderen Seiten ,bleibt der Inhalt von einem Quadratmeter, der Umfang wächst jedoch auf fünf Meter. Teilt man die Seiten von einem halben Meter wiederum auf einen Viertelmeter, während man die beiden anderen Seiten von zwei Metern auf vier Meter verdoppelt, bleibt der Rauminhalt wiederum gleich, während sich der Umfang weiter vergrößert. So kann man den Umfang unbegrenzt weiter vergrößern, der Rauminhalt bleibt jedoch immer endlich.

Nach der Relativitätstheorie löst sich das Bewegungsparadoxon auf folgende Weise:
Jeder sich selbst überlassene Körper befindet sich in gleichförmiger geradliniger Bewegung.Raum und Zeit sind Eigenschaften der Materie, sie verschwinden mit dieser. Demnach ist Bewegung ein Grundzustand, der nur relativ zum Beobachter wahrnehmbar ist, der nicht „parallel gleichschnell“ bewegt ist.Die Bewegung erfolgt nicht in einem vorgegebenen Raum-Zeit-Kontinuum relativ zu diesem, sondern dieses ist mit dem bewegten Körper verbunden (Eigenraum, Eigenzeit).Zenons Läufer legt also keine Strecken im Raum innerhalb einer Zeitspanne zurück (unendlich viele Teilstrecken und Zeitabschnitte), sondern ändert nur seinen Grundzustand der Bewegung - in Bezug auf einen nicht gleichermaßen bewegten Beobachter- durch Beschleunigung und damit seine Raumzeit.

Mathematisch gesehen, ergibt sich das Zenon´sche Paradoxon aus einer unzulässigen Koordinationstransformation.Bewegung wird in einem Koordinatensystem mit den Koordinaten Strecke und Zeit abgebildet.Sie ergibt sich in der Relation auf den Nullpunkt. Verschiebt man diesen mit der Position des bewegten Körpers , lässt sich keine Bewegung auf dem mitbewegten Koordinatensystem ablesen.Die Verschiebung ist natürlich unbegrenzt möglich – worauf Zenon allein abstellt - , Bewegung ergibt sich jedoch nur in Bezug auf das ursprüngliche Koordinatensystem. Bewegung ist definiert als Strecke / Zeit. Strecke ist etwas Eindimensionales. Verkürzt man sie so, dass sie auf die Dimension Null zustrebt, kann von einer Bewegung daher nicht mehr gesprochen werden.Damit sind wir wieder bei der Speziellen Relativitätstheorie, die sich insoweit von der Newton´schen Physik gar nicht unterscheidet: Bewegung ergibt sich nur im Hinblick auf einen Beobachter, der nicht mit dem Bewegungsobjekt in gleicher Weise mitbewegt wird. Für den Läufer Zenons ergibt sich seine Bewegung erst in Relation auf die Umwelt, zum Beispiel die vorbeiziehenden Bäume und die überholte Schildkröte,für diese in Relation auf sich selbst.
Die Infinitesimalrechnung liefert nur eine mathematische Beschreibung des Paradoxons, keine Erklärung.

Fortsetzung folgt

Knut Hacker
22.09.11, 13:25
Letzte Fortsetzung:

Eine moderne Variante des Zenon´schen Paradoxons ist das Thompson´sche Lampen –Paradoxon:
Knipse eine Lampe an und aus: zunächst im Abstand von einer Minute, dann von einer halben Minute, dann von einer Viertelminute usw.! Ist die Lampe nach zwei Minuten an oder aus? Wie oft musstest du knipsen?
(Thompson ruinierte so viele Lampen, dass die Hersteller mit den Nachlieferungen nicht mehr nachkamen, und verursachte so viele Kurzschlüsse, dass das gesamte Stromnetz des Landes immer wieder lahmgelegt wurde. Schließlich starb er an einem gewaltigen Stromschlag.
Zu seiner Beerdigung erschien Meister Lampe und strahlte flackernd zwei Minuten lang über das gesamte Gesicht, was in der Presse als taktlos kritisiert wurde.Die Lampenhändler boykottierten - trotz ihrer glänzenden Einnahmen - die Beerdigung, weil sie sich von Thompson dadurch ver*****t fühlten, dass dieser vor der Lösung des Paradoxons auffälligerweise gerade beim Versuch der Lösung verstorben war. „Als ob sie an dieser ein Interesse gehabt hätten!" höhnte die Presse.
Mathematiker, die der Lösung des Paradoxon entgegengefiebert hatten, formierten sich zu einem Protestzug gegen die Praxisuntauglichkeit der Lampen und gegen die theoretische Lösungsfeindlichkeit von Paradoxien.
Aus dem Grab soll heute noch täglich um Mitternacht das klickende Geräusch von Lampenschaltern zwei Minuten lang dringen- und anschließend ein tiefer Seufzer.
Soweit eine kleine Auflockerung des Themas durch mich.)

Eine weitere Variante des Teilungsparadoxons – es gibt eine Vielzahl wie z.B. das Serides(Haufen)-Paradoxon - habe ich mir einfallen lassen (Zeitparadoxon):
Es ist vollkommen ausgeschlossen, geboren worden zu sein.Denn wäre man geboren worden, müsste es einen letzten Augenblick gegeben haben, in dem man noch nicht geboren war, und einen ersten Augenblick, in dem man bereits geboren war. Beide Augenblicke müssten sich unterschieden haben und daher voneinander getrennt gewesen sein.Sie können aber nicht getrennt gewesen sein, da jeder noch so kleine Zwischenraum entweder zum letzten Augenblick des Noch-nicht-geboren-Seins oder zum ersten Augenblick des Bereits-geboren-Seins gehört haben müsste. Denn während dieses Zwischenraumes kann man nicht gleichzeitig ungeboren und geboren und auch nicht weder ungeboren noch geboren gewesen sein.
Ebenso ist es vollkommen ausgeschlossen, zu sterben. Denn stürbe man, müsste es einen letzten Augenblick geben, in dem man noch lebt, und einen ersten Augenblick, in dem man bereits tot ist. Beide Augenblicke müssten sich unterscheiden und daher voneinander getrennt sein. Sie können aber nicht getrennt sein, da jeder noch so kleine Zwischenraum entweder zum letzten Augenblick des Noch-Lebens oder zum ersten Augenblick des Bereits-tot-Seins gehören müsste. Denn während dieses Zwischenraumes könnte man nicht gleichzeitig lebendig und tot und auch nicht weder lebendig noch tot sein.

Knut Hacker
22.09.11, 13:43
Hallo ihr,

Entschuldigt bitte meinen langen Exkurs in eines meiner Lieblingsthemen.

Zur Strafe will ich wieder auf das Ausgangsthema zurückkommen:

Es ist hier offenbar der falsche Eindruck entstanden, als könne der Ersatz der zweiwertigen durch die mehrwertige Logik das Grundproblem der Selbstbezüglichkeit unseres Denkens lösen.

Hierzu möchte ich Michael Guillen zitieren:

"....dass kein logisches System seine eigene logische Stimmigkeit nachweisen kann. Der Glaube an die Logik ist in anderen Worten nicht weniger subjektiv (als) der Glaube an ein säkuläres oder mystisches Erklärungsprinzip, weil die Logik selbst weder logisch noch objektiv verifiziert werden kann."

Oder etwas seriöser: Martin Heidegger schreibt in "Was ist Metaphysik?" :
"Die Idee der ´Logik` selbst löst sich auf im Wirbel eines ursprünglicheren Fragens."

Ähnliches besagen die Gödel´schen Unvollständigkeitssätze für den Bereich der Mathematik (verallgemeinerungsfähig: Kein System kann sich aus sich heraus erklären wegen fehlender Metaebene der Betrachtung und Beurteilung; die Verallgemeinerungsfähigkeit hat Gödel selbst in seinem - unter seinem Niveau liegenden - Gottesbeweis postuliert).

Jeder Frage liegen Prämissen zugrunde, die den Bereich der Antworten eingrenzen, die mit diesen Prämissen vereinbar sind.
Je abstrakter die Prämissen, desto weniger Raum für adäquate Antworten. Bei den abstraktesten Prämissen wie Sein, Wahrheit usw. kommt es daher zu Rückkopplungen und damit zur Selbstbezüglichkeit in Form eines Denkzirkels oder einer Iteration.

Was die mehrwertige Quantenlogik betrifft, so gibt es eine erstaunliche Parallele zur Theologie. Der Atheist sagt: Gott gibt es nicht. Der Theologe antwortet:eben, sonst wäre er nicht Gott, sondern durch das Sein überbestimmt, Gott ist über Sein und Nichtsein erhaben, er hat diesen Gegensatz erst erschaffen. Bonhoeffer: "Einen Gott, den es gibt, gibt es nicht!"

In der Quantenphysik geht es natürlich nicht um Sein und Nichtsein, sondern um die physikalischen Begriffe "raumzeitlich wohldefiniert" oder "unbestimmt" oder "unabhängig". Aber wenn man die Raumzeit als "Sein" bezeichnet dann führt die Quantenlogik in dieselbe Richtung wie die theologische: Paradoxien sind keine Aporien sondern ein Ausweg aus ihnen zu einer Denkgrenze, die hinzunehmen befriedigender erscheint als gegen eine Wand zu rennen.

Aristoteles hatte die Grenzen seiner zweiwertigen Logik übrigens selbst aufgezeigt am Beispiel der Inkommensurabilität der Quadratdiagonale zur Seite ( ἡ τῆς διαμέτρου ἀσυμμετρία ):
Es gibt kein gemeinsames Maß, kein durch ganze Zahlen ausdrückbares Verhältnis. Die Länge der Diagonale "Wurzel aus zwei" ist eine irrationale Zahl, ein unendlicher, nicht periodischer Dezimalbruch (Metaphysik 983 a, 15 sq 20).
Schon in der klassischen Physik war die zweiwertige Logik am Dreikörperproblem gescheitert.

Knut Hacker
22.09.11, 16:35
Worin sieht Du den Unterschied zwischen der Wirklichkeit der Gedanken (Vorstellung) und der Wirklichkeit der uns umgebenden Natur ?



Nachtrag zu meiner bereits abgegebenen Antwort:

Von Robert Gernhardt stammt folgendes Gedicht:

Philosophische Geschichte

Die Innen-und die Außenwelt,
die warn mal eine Einheit.
Das sah ein Philosoph, der drang
erregt auf Klar-und Reinheit.

Die Innenwelt,
dadurch erschreckt,
versteckte sich in dem Subjekt.

Als dies die Außenwelt entdeckte,
verkroch sie sich in dem Objekte.

Der Philosoph sah dies erfreut:
indem er diesen Zwiespalt schuf,
erwarb er sich für alle Zeit
den Daseinszweck und den Beruf.

richy
22.09.11, 19:21
Hi Knut

Die Diskussion zu Zeilinger und der KI fand ich weitaus interessanter als das Zenonsche Paradoxon. Daher kurz dessen Loesung :


1)
Der Hase sei c mal schneller als die Schildkroete :
vH=c*vS => vS/vH=1/c

Der Abstand fuer t=0 sei x0
Wir sollten wissen wie lange der Hase fuer diese Strecke braucht, weil wir daraus "berechnen" koennen wie weit die Schildkroete dann kam.
Hase t=x0/vH
Die Schildkroete kommt in dieser Zeit wie weit ?
Schildkroete x=t*vS=x0/vH*vS=x0*1/c. Das war logisch zu erwarten.

x0=x0*1/c ist somit der Abstand, Ausgangspunkt fuer die zweite Betrachtung. Und da sich lediglich X0 veraendert hat und wir nur diese als Ausgangsgroesse verwendet haben, wird unser naechster Abstand sein x0*1/c^2
Der Hase ist dann insgesamt weitergekommen : x0*(1+1/c)
Im k_ten Schritt: x0*(1+ 1/c + 1/c^2 + 1/c^3 .... +1/c^k)

Er hat die Kroete nach unendlich vielen Schritten k->00 erreicht.
Das ist eine geometrische Reihe und diese konvergiert im vorliegenden Fall fuer c>1
http://de.wikipedia.org/wiki/Geometrische_Reihe
Der Hase hat somit nach x0/(1-1/c) Metern,
xH=x0 *c/(c-1) Meter die Schildkroete erreicht.
************
Nichts paradoxes wenn man die geometrische Reihe kennt.


Vergleich mit ueblicher Rechnung Im s(t) Diagramm :
***************************************
vH*t=x0+vS*t
Sie treffen sich zum Zeitpunkt t=x0/(vH-Vs)
In dieser Zeit hat der Hase welchen Weg zurueckgelegt ?
xH=x0*vH/(vH-Vs)
xH=x0*c/(c-1)
***********

2)
Welche Zeitabschnitte betrachtet der Hase im k ten Schritt ?
***********************************************
t(k)=t0/c^k
Die Zeitabschnitte streben wie die Wegabschnitte gegen Null.
Aus Sicht des Hasen dreht sich nach dem Ueberholvorgang die Zeit um. Wenn er rueckwaerts schaut entfernt er sich nun von der Schildkroete.

Interessante Sache. Waehlt man beschleunigte Hasen lassen sich sicherlich auch andere Reihensummen ueber Polynome berechnen.

http://www.quanten.de/forum/showthread.php5?t=1598&page=6&highlight=Zenon

richy
22.09.11, 19:34
Aus selbigem damaligen Thread:

Loesung der Lampenscherzaufgabe :
***************************
Wir betrachten das erste "An und Ausschalten." Die Lampe soll c/2 Zeiteinheiten brennen und c/2 Zeiteineiten nicht. Der erste Vorgang ist somit nach c Zeiteinheiten abgeschlossen :

1111111100000000
0.........c/2.........c.........> Zeit

Nun der zweite Vorgang. Er dauert c/2 Zeiteinheiten.
Er startet somit bei c und endet bei c/2

111111110000000011110000
0.........c/2.........c........c+c/2

Nun der dritte Vorgang. Er dauert c/4 Zeiteinheiten.
Er startet somit bei c+c/2 und endet bei c+c/2+c/4

1111111100000000111100001100
0.........c/2.........c........c+c/2...c+c/4

Den ersten An und Ausschaltvorgang t=0-c betrachtet man getrennt :
Gilt t<c ist die Aufgebenstellung soundso trivial

Nun prueft man das naechste Intervall :
Gilt c < t < c+c/2 ?

und das uebernaechste
Gilt c*(1+1/2) < t < c*(1+1/2+1/4) ?

Es ergeben sich somit k Intervalle
summe c*(1+1/2+1/2^2+1/2^3 ..1/2^(k-1)) < t < summe c*(1+1/2+1/2^2+1/2^3 ..1/2^(k))

Wir druecken die geometrische Reihe von 1/2 geschlossen aus :
Loesung :
2*c*(1-0.5^k) < t < 2*c*(1-0.5^(k+1))
*******************************
Die Losung koennte man noch geschickter ausdruecken. Auch logarithmisch darstellen.

Jetzt sehen wir : Hat der gute Tompson im Mathe Unterricht geschlafen ?
Denn die Summe konvergiert fuer k-> unendlich gegen 1/(1-1/2)=2

D.h.
Er kann diesen seltsamen Versuch gar nicht praktisch fuer unbegrenzte Zeiten durchfuehren !
Weil seine Aufgabenstellung von Anfang an zum scheitern verurteil ist !
Das ist widerum kein Paradoxon sondern eine unsinnige Aufgabenstellung.

Gilt C=1 Minute, so ist das Spiel nach 2 Minuten aus !
Denn er muesste mit unendlich hoher Frequenz an und ausschalten.
Es werden ihm aber schon vorher die Finger abfallen.
Kein Paradoxon sondern eine Scherzaufgabe

Knut Hacker
22.09.11, 19:54
Hi richy,

die "Lösung" kenne ich. Wie oben in meinem ellenlangen Beitrag ausgeführt, sind mathematische Lösungen aber lediglich abstrakte Beschreibungen, bleiben also bei reduktionistischem Denken weiterhin rätselhaft.

Ganz abgesehen davon, dass Albert Einstein die bloße Idealität der Mathematik wie folgt aphorisiert hat: „Insofern sich die Sätze der Mathematik auf die Wirklichkeit beziehen, sind sie nicht sicher, und insofern sie sicher sind, beziehen sie sich nicht auf die Wirklichkeit.“ „... evident, dass die Mathematik als solche weder über Gegenstände der anschaulichen Vorstellung noch über Gegenstände der Wirklichkeit etwas auszusagen vermag.“ („Mein Weltbild“),

hat sich der Mathematiker und Logiker

Bertrand Russel nicht nur erfolglos den Kopf zerbrochen über das Paradoxon der Menge aller Mengen, die sich nicht selbst enthalten, sondern zu mathematischen "Lösungen" grundsätzlich angemerkt:

„Mathematische Erkenntnis wird ... nicht durch Induktion aus Erfahrung gewonnen: Dass 2 + 2 = 4 ist gleich vier ist, glauben wir nicht, weil wir so oft die Beobachtung gemacht haben, dass ein Paar und noch ein Paar zusammen ein Quartett ergeben. In diesem Sinne ist die mathematische Erkenntnis nicht empirisch. Sie ist aber auch keine apriorische Erkenntnis der Welt. In Wirklichkeit handelt es sich bei ihr nur um eine verbale Erkenntnis. „3“ bedeutet „2 + 1“, und 4 bedeutet „3 +1“. Es ergibt sich also (wenn der Beweis auch lang ist), dass „4“ das gleiche bedeutet wie „2 + 2“.
“ (Philosophie des Abendlandes, 31. Kapitel).

Ich will gar nicht mehr auf andere Mathematiker eingehen:

„.... dass sowohl die Mathematik als auch die Religion als Grundlage eines Glaubens bedürfen, den das Individuum selbst mitbringen muss.“
So die Mathematiker Davis und Hersh, 1990, In: „Descartes Traum. Über die Mathematik von Zeit und Raum“, Seite 304).

„..dass kein logisches System seine eigene logische Stimmigkeit nachweisen kann. Der Glaube an die Logik ist in anderen Worten nicht weniger subjektiv (als) der Glaube an ein säkulares oder mystisches Erklärungsprinzip, weil die Logik selber weder logisch noch subjektiv objektiv verifiziert werden kann.“
So der Mathematiker und Logiker Gödel, Unvollständigkeitssätze

Die einleuchtendste Llösung scheint mir der Quantenphysiker Carl Friedrich von Weizsäcker zu liefern:

Bewegung ist definiert als Weg durch Zeit. Daher gibt es keine Bewegung in einem bloßen Punkt des Weges oder der Zeit! Achilles muss also nicht unendlich viele Punkte seines Weges in unendlich vielen Zeitpunkten zurücklegen.

Slash
26.09.11, 08:03
Oder sollte man sagen: Alles ist Materie, und alles besitzt Masse/Energie ... ?

Und das Universum als Ganzes die Energie Null hat? :) :o

Hawkwind
26.09.11, 10:44
Und das Universum als Ganzes die Energie Null hat? :) :o

Ja, unser Universum ist nichts als eine Quantenfluktuation, die sich dank Hr. Heisenberg ganz kurz nur Energie ausgeborgen hat. Bald sind wir alle wieder weg. :)

Slash
26.09.11, 11:32
Lässt sich Fluktuation eigentlich durch irgendetwas beeinflussen?

Bzw. hat diese Fluktation so etwas wie eine mittlere Energie (Leistung) .- vielleicht ist das aber gerade eben der Casimireffekt :confused: kenn mich nicht so aus in Physik.

Hawkwind
26.09.11, 12:31
Ach, ich hab doch nur rumgesponnen. :)

Nick Rymer
26.09.11, 13:25
Hi,
Und das Universum als Ganzes die Energie Null hat? :) :o
Davon gehe ich aus.

Gruß, Nick

EMI
26.09.11, 13:53
Davon gehe ich aus.Kannst Du Nick,

ist aber trotzdem grottenfalsch.

Gruß EMI

Nick Rymer
26.09.11, 18:11
[...]ist aber trotzdem krottenfalsch.

Nabend EMI,
hab nochmal nachgedacht und bin zu dem Schluss gekommen, falsch interpretiert zu haben.

Alles nicht so einfach. Krottenfalsch schreibt man übrigens mit g.

Gruß, Nick

Knut Hacker
27.09.11, 13:57
Interessant erscheint mir, wie bereits Aristoteles (384-322 v. Chr.) die Materie (ὕλη) sehr nahe dem Verständnis der Quantenphysik definiert hat. Materie, rein für sich genommen, sei bloße Möglichkeit (δυνάμει ὄν). Es gebe kein Sein, das nicht Struktur aufzuweisen habe. Materie sei ein in sich widerspruchsvolles Gebilde, ein Zwischengebilde zwischen Sein und Nichtsein, eine Hinneigung, Entelechie (ἐντελέχεια).
Eine verblüffende Parallele der Formulierungen zu den Quantenphysikern, wie ich sie in diesem Thread zitiert habe.

Nick Rymer
27.09.11, 14:22
„Insofern sich die Sätze der Mathematik auf die Wirklichkeit beziehen, sind sie nicht sicher, und insofern sie sicher sind, beziehen sie sich nicht auf die Wirklichkeit.“ „... evident, dass die Mathematik als solche weder über Gegenstände der anschaulichen Vorstellung noch über Gegenstände der Wirklichkeit etwas auszusagen vermag.“ („Mein Weltbild“),

Hallo Knut,
aber es ist schon erstaunlich, dass wir glauben, über die Mathematik urteilen zu können. Das bedeutet letztlich, dass wir der Vernunft einen gößeren Wert zumessen als ihrem Kind, der Mathematik.
Es stellt sich somit die Frage, worauf basiert unser Geist, denn wenn es nicht das Kind der Vernunft, die Mathematik ist, auf der er basiert, kann es auch die Vernunft nicht sein. Dies führt mich zu deinem Beitrag
Es gebe kein Sein, das nicht Struktur aufzuweisen habe. Materie sei ein in sich widerspruchsvolles Gebilde, ein Zwischengebilde zwischen Sein und Nichtsein, eine Hinneigung, Entelechie (ἐντελέχεια).
der in meinen Augen vermuten lässt, das Aristoteles nicht-Lebendiges oder einfach Totes nicht zum Substantiellen zählt. Das aber lässt, nimmt man ihn ernst, den Schluss zu, das a) Substanz als Leben definiert ist, womit b) zusammen mit meinen Schlüssen zur Mathematik die einzig ernstzunehmende Substanz in unserem Kosmos der Geist ist.
Über den Geist bin ich leider wenig belesen, so dass ich nur vermuten könnte, dennoch halte ich ihn für das größte und wichtigste Geheimnis, dass man erforschen muss, bevor man glaubt, die Wahrheit erkennen zu können. Schön an dieser Aufgabe ist, dass man sogar allein und für sich daran arbeiten kann.

Für uns Physiker ist es wohl wahrscheinlich ganz und gar unannehmbar, dass es die Theologen sind, die zu dem Thema mehrheitlich etwas zu sagen haben. Letztlich ist es doch aber ganz wunderbar, dass überhaupt jemand befragt werden kann.

Gruß, Nick

P.S.: Ich denke, das Wesen des Geistes manifestiert sich uns geistbehafteten Menschen im Studium seiner Reaktionen auf weltliche Erfahrungen. Wie sollte es sonst gehen? Grübelei?
Diese Vorgehensweise, so leuchtet leicht ein, fordert eine kultivierende Wechselwirkung heraus. Mönchen sagt man dergleichen nach. So weit würde ich jedoch nicht gehen. Der Motor der Erkenntnis sind m.E. Erlebnisse. Erlebnisse, die zum Reflektieren zwingen. Man nennt so jemand dann ugs. 'erfahren'.

Knut Hacker
27.09.11, 17:46
Hallo Nick Rymer,

auf deine so vielen interessanten Fragen und Gedanken lässt sich leider schriftlich nicht befriedigend antworten, nicht nur wegen der Fülle, sondern auch wegen der Komplexität und der unendlich vielen Meinungen von Vordenkern seit Menschen Gedenken.

Ich möchte daher möglichst kurz nur im Grundsätzlichen antworten, soweit ich mir selbst diese Fragen beantwortet habe beziehungsweise dabei in Denkzirkel und Iterationen geraten bin.

Der „Geist“, was auch immer man darunter verstehen mag, ist zwar nicht Gegenstand der Physik und dieses Diskussionsforums. Aber die Quantenphysik führt uns eben in Bereiche, in denen die traditionellen Vorstellungen von Materie versagen.

Begreift man den „Geist“ im engeren Sinn als dasjenige, was zum Beispiel der Mathematik als Konstrukt des menschlichen Denkens zugrunde liegt –
denn die Natur selbst kennt ja keine Mathematik ( Krümmungen als ihre Hauptstrukturen lassen sich mathematisch nur approximativ durch π und die Integral-und Infinitesimalrechnung erfassen, die Quadratsdiagonale ist im Verhältnis zu den Quadratseiten inkommensurabel, das Dreikörperproblem nur approximativ lösbar; wir müssen lückenfüllend mit Variablen und Parametern arbeiten; Symmetriebrüche treten auf, die imaginäre Einheit i. ; Naturkonstanten wie die Lichtgeschwindigkeit lassen sich nicht ganzzahlig messen; nicht Linearität, sondern Nichtlinearität und Komplexität sind die Regel)-
dann stößt man auf das Problem, das Gödel für die Mathematik in seinen beiden Unvollständigkeitssätzen formuliert hat, nämlich dass kein System sich aus sich selbst heraus erfassen und erklären kann, weil es dann an der nötigen Metaebene der Betrachtung fehlt. Der Mensch kann nicht aus seinem Geist heraustreten und sich daneben stellen und sagen: Aha, das also ist der Geist.

Man stelle sich auch vor, dass das Universum ja fast 13 Milliarden Jahre ohne Leben und damit Geist ausgekommen ist, somit also aus der Perspektive des Geistes völlig überflüssig, weil niemandem bewusst, gewesen ist.

Wenn man am Begriff des Geistes festhält, sollte man ihn jedoch im weiteren Sinne begreifen, nicht nur als menschliches Wahrnehmen, Denken, Fühlen, Werten, sondern im Sinne Heideggers als „das Wunder aller Wunder“ – als Physiker würde man sagen: Singularität aller Singularität - , nämlich „dass Seiendes ist“- zu ergänzen, dass auch Nichtseiendes „ist“, dass letztlich alles kontingent ist, weil es eine letzte Ursache schon deshalb nicht geben kann, weil sie bereits etwas Seiendes wäre und daher das Sein voraussetzen würde, für das sie doch Ursache sein sollte!

Interessant ist ja der Zufall in der Quantenphysik und in der Evolutionslehre. Es handelt sich um eine selbst ursachenlose Ursache für weitere Entwicklungen. Allerdings geschieht der Zufall immer in einem System, so dass dieses System als Ursache angesehen werden kann.Doch auch diese Ursache hätte keine Ursache mehr.

Könnte man nicht diese Kontingenz als Geist bezeichnen?

Ohne den Zufall wäre die Zeit überflüssig. Denn es wäre zu fragen, warum etwas nicht gleich ist, was sich erst entwickelt. Der Zufall setzt einen objektiv der Natur innewohnenden Prognosehorizont.

Man könnte Geist als das nicht weiter Hinterfragbare bezeichnen.

Nick Rymer
28.09.11, 08:28
dann stößt man auf das Problem, das Gödel für die Mathematik in seinen beiden Unvollständigkeitssätzen formuliert hat, nämlich dass kein System sich aus sich selbst heraus erfassen und erklären kann, weil es dann an der nötigen Metaebene der Betrachtung fehlt. Der Mensch kann nicht aus seinem Geist heraustreten und sich daneben stellen und sagen: Aha, das also ist der Geist.[...]
Man könnte Geist als das nicht weiter Hinterfragbare bezeichnen.
Hallo Knut Hacker,
leider hast du mir wenig Definierendes über den Geist genannt. Du bietest lediglich einen persönlichen Vorschlag dazu an. Somit habe ich die Freiheit, es dir gleichzutun:

Vorraussetzung jeder Definierungsarbeit ist wohl in der Tat die Metaebene. M.E. gibt es die aber durchaus und dies sogar in unserem beschränkten 3+1-dim-Raum: Erstaunlicherweise gibt es allem voran das Bewusstsein (ich würde mich sehr freuen, dazu von dir Definierendes zu hören). Dieses Bewusstsein möchte ich nicht mit dem Geist verwechselt sehen. Das Bewusstsein ist selbst dazu in der Lage, über den Geist, in meinem Sinne, zu urteilen, und damit in der Lage, ihn zu definieren. Wie sollte zB Meditation sonst den Geist in einer Wechselwirkung beeinflussen können? Es scheint, als stünden Geist und Bewusstsein in stetem Kampf miteinander, der sie stärkt.

Was wir erfahren, im Leben, in Versuchen, sind in der Regel kleine Eisberge. Das meiste sehen wir nicht. Aber tragen wir zumindest das ab, was wir sehen bzw. verstehen, was wir sehen, so kommt wieder ein Stück des Eisbergs aus dem Meer hervor - mit diesem Stück gehen wir ebenso um. Es ergibt sich an diesem Bilde wie von selbst, dass wir nie den ganzen Eiseberg, das Wissen, den Geist zu Gesicht kriegen werden, denn wieder und wieder lugt etwas hervor. Ob der Eisberg jedoch begrenzt ist, das Wissen, der Geist, können wir allein daraus schließen, dass er nur einen Teil von sich preisgibt.

Wir vermuten, glaub ich, dass das Universum insgesamt ein System ist. Wir suchen Regeln, die dem ganzen System und jeder kleinsten Einheit des Systems zugrunde liegen. Somit ist klar, dass man ebensogut die kleinste Einheit erforschen kann, um eine oder zwei Regeln des Ganzen zu finden.
Du sagtest bereits, man möge doch auch Stoffliches ins Kalkül des Geistes miteinbeziehen, denn wenn wir festhielten, das eigentlich einzig Geistiges sustantiell ist, aber auch Stoffliches in seinem Verhalten einen lebendigen bzw. unberechenbaren Charakter aufweist, dann ist das legitim.
Ebensogut kann man dann aber am Studium des Lebens Regeln des Ganzen finden, denn vermutlich ist dann Geist und Leben nur die andere Seite der gleichen Medaillie, deren vordergründige Seite das Stoffliche ist. Wir alle bzw. ich sage lieber ich - halte es so. Gewisse Abstraktionsfähigkeiten seien vorausgesetzt.

Man nehme bspw. den Begriff Geld. Vor langer Zeit als Zahlungsmittel erdacht entwickelt es mehr und mehr Eigendynamik, ja , man kann ihm fast einen Geist unterstellen, doch dieser Geist sind immer noch wir Menschen. So zeigt sich am Bilde des Geldes ein Teil des Geistes, da er sich auch in uns Menschen manifestiert.
Ich könnte jetzt weit ausholen und es ist mir auch sehr danach, aber ich möchte es deinem Bewusstsein unbeeinflusst von meiner Sicht ermöglichen, eine andere Facette des über allem stehenden Geist zu entwerfen. Wir Menschen sind bzgl. des Geistes wie ein Kaleidoskop: Die Elemente sind immer dieselben, aber die Gestalt ist voller Vielfalt. Die Definition der Elemente des Kaleidoskops ist die eigentliche und spannende Aufgabe der Wissenschaft.

Gruß, Nick

Knut Hacker
28.09.11, 18:19
leider hast du mir wenig Definierendes über den Geist genannt.
Hallo Nick,
Das geschah bewusst, weil das Thema ja unerschöpflich ist. Ich will in weiteren Beiträgen in mehreren Fortsetzungen versuchen, Näheres dazu darzulegen.


Das Bewusstsein ist selbst dazu in der Lage, über den Geist, in meinem Sinne, zu urteilen, und damit in der Lage, ihn zu definieren. Wie sollte zB Meditation sonst den Geist in einer Wechselwirkung beeinflussen können?

Das Bewusstsein vom eigenen Bewusstsein, das sogenannte Überbewusstsein,
macht uns lediglich unser Bewusstsein als solches bewusst, aber nicht, worum es sich dabei handelt, denn auch das Überbewusstsein ist Bewusstsein, so dass sich die fehlende Metaebene nur verschiebt.
Außerdem stellt sich ja das Problem des Subjektivismus.Alles, auch die Unterscheidung zwischen Subjektiv und Objektiv und der Begriff des Bewusstseins sind lediglich Konstrukte des Bewusstseins, auch diese Aussage. Wir drehen uns immer im Kreis, da wir aus unserem Bewusstsein nicht heraustreten können, wobei auch diese Vorstellung lediglich bewusstseinsbedingt ist. In den Kognitionswissenschaften spricht man von Strukturdeterminiertheit unseres Bewusstseins, dazu später.
Auch die meditativen Zustände sind Zustände des Bewusstseins, können also über dieses nichts besagen, abgesehen davon, dass sie heute durch hirnneurologische Reizungen hervorgerufen werden können. Ich empfehle einen Besuch des Nürnberger "Turms der Sinne".


Ob der Eisberg jedoch begrenzt ist, das Wissen, der Geist, können wir allein daraus schließen, dass er nur einen Teil von sich preisgibt.
Das Problem liegt wohl tiefer. Ist denn der Geist ein Beobachtungsobjekt? Das Messproblem in der Quantenphysik bestätigt die Ergebnisse der Kognitionswissenschaft, dass die Trennung zwischen Subjekt und Objekt lediglich ein geistiges Konstrukt ist. Schon die so genannten Qualia beweisen doch, dass es kein "Wissen" im Sinne eines Erfassens von etwas außerhalb des Bewusstseins Gelegenem gibt. Man stößt immer wieder auf Denkzirkel und Iterationen oder willkürliche Abbrüche (sogenanntes "Münchhausen- Trillemma",wie es der Philosoph Hans Albert nennt).


Wir vermuten, glaub ich, dass das Universum insgesamt ein System ist. Wir suchen Regeln, die dem ganzen System und jeder kleinsten Einheit des Systems zugrunde liegen. Somit ist klar, dass man ebensogut die kleinste Einheit erforschen kann, um eine oder zwei Regeln des Ganzen zu finden.
Du sagtest bereits, man möge doch auch Stoffliches ins Kalkül des Geistes miteinbeziehen, denn wenn wir festhielten, das eigentlich einzig Geistiges sustantiell ist, aber auch Stoffliches in seinem Verhalten einen lebendigen bzw. unberechenbaren Charakter aufweist, dann ist das legitim.
Ebensogut kann man dann aber am Studium des Lebens Regeln des Ganzen finden, denn vermutlich ist dann Geist und Leben nur die andere Seite der gleichen Medaillie, deren vordergründige Seite das Stoffliche ist. Wir alle bzw. ich sage lieber ich - halte es so. Gewisse Abstraktionsfähigkeiten seien vorausgesetzt.

Du sprichst hier Probleme der Systemtheorie an. Die Chaosforschung hat auch naturwissenschaftlich ( wie bereits die Logik ) bewiesen, dass auch die Unterscheidung zwischen Ganzem und Teil lediglich ein Bewusstseinskonstrukt ist.Jedes Ganze ist Teil einer umfassenderen Einheit, und jedes Teil ist wiederum Ganzes von weiteren Teilen. Jedes Ganze ist qualitativ etwas anderes als die Summe seiner Teile (es weist dynamische Strukturen auf, die den Teilen nicht eigen sind, sondern sich "autopoietisch" entwickeln). Ilias Prigogine hat den wissenschaftlichen Terminus des "dissipativen Systems" entwickelt.Es gibt keine abgeschlossenen Systeme - es handelt sich dabei lediglich um Idealisierung, also auch beim zweiten Hauptsatz der Wärmedynamik -, sondern alles hängt mit allem zusammen. Insbesondere ist auch Leben Teil der Umwelt (Stoffwechsel usw.).

Der Begriff der Stofflichkeit dürfte wohl schwerlich weiterführen.Er verlagert nur das Problem. Die Unterscheidung zwischen Geist und Stofflichkeit bleibt undefinierbar. Alle abstrakten Begriffe können bekanntlich nicht beschrieben werden, in höchster Abstraktion der Begriff des Seins, darunter die Begriffe von Raum und Zeit, Ganzem und Teil, Ursache und Wirkung usw. insbesondere auch die Begriffe von Geist und Materie.


Wir Menschen sind bzgl. des Geistes wie ein Kaleidoskop: Die Elemente sind immer dieselben, aber die Gestalt ist voller Vielfalt. Die Definition der Elemente des Kaleidoskops ist die eigentliche und spannende Aufgabe der Wissenschaft.


So sahen es die alten Griechen, insbesondere Platon, der als Elemente nicht wie Demokrit die Atome, sondern (geistige)Strukturen (Dreiecke) in verschiedenen Kombinationen (Gestalten) ansah. Viele Quantenphysiker greifen auf dieses Bild zurück.
Der Vorstellung von Elementen liegt jedoch das oben angeführte geistige Konstrukt eines Gegensatzes oder einer Unterscheidung zwischen Ganzem und Teil zugrunde. Dagegen spricht nicht nur die Komplexitätsforschung (Chaosforschung), sondern auch die nichtlokalen Erscheinungen der Quantenphysik (Superposition, Verschränkung).

Knut Hacker
28.09.11, 18:55
Hallo nick,
Hier die angedrohte ausführlichere Darsellung des Problemkomplexes:

Es kommt wohl auf die Definition des Bewusstseins an.
Die positivistische Anschauung in der Kognitionswissenschaft und in der Neurobiologie sieht es als bloße Eigenschaft hochkomplexer materieller Systeme an.
In der Quantenphysik dagegen spielt das Bewusstsein beim Paradox des Kollapses der Wellenfunktion von Elementarteilchen ( sog. Superposition) durch die Beobachtung (sogenanntes Messproblem; makrokosmisch parabolisiert durch „Schrödingers Katze“) doch eine zentrale Rolle!

Die behavioristische, materialistische wissenschaftliche Auffassung, wonach Bewusstsein lediglich eine emergente Eigenschaft der Komplexität ist, stützt sich gerne u.a. auf das Gleichnis vom „chinesischen Zimmer“: In einem abgeschlossenen, uneinsehbaren Zimmer befindet sich ein Mensch, der nur Deutsch versteht.und Regale, in denen, auf Papierbögen geschrieben, alle chinesischen Sätze wohl sortiert archiviert sind.Ein Katalog enthält alle deutschen Sätze und verweist auf die entsprechenden chinesischen. Das Zimmer ist mit der Außenwelt durch einen Schlitz verbunden, durch den Papierbögen ein- und ausgeschoben werden können.Den Außenstehenden wird folgende Information erteilt: Im Zimmer befinde sich ein chinesischer Dolmetscher und sonst überhaupt nichts.Ein Außenstehender will einen Satz übersetzen lassen. Er schreibt ihn auf einen Papierbogen und reicht ihn in das Zimmer.Der angebliche Dolmetscher sucht anhand seines Kataloges den entsprechenden chinesischen Satz und reicht ihn hinaus.Der Außenstehende wird dem angeblichen Dolmetscher Kenntnisse des Chinesischen zuschreiben, obwohl sich die Beherrschung einer Sprache nicht in der Kenntnis von Wörtern und Grammatik erschöpft, sondern Bewusstseinstrukturen wie Abstraktion, Kombination, Intuition u.s.w.voraussetzt.
Dieses Gedankenexperiment ist natürlich schon aus zeitlichen Gründen irrreal, aber angeblich nicht zu ersetzen durch das Bild eines Übersetzungscomputers, da dieser ja durch Geist programmiert werden musste.
Ich halte das nicht für schlüssig, da im Gedankenexperiment der Geist eben auf die Erstellung des Kataloges verwendet worden ist.
Die Vertreter dieser positivistischen Anschauung schreiben allerdings bereits einer Türe, die sich durch eine Lichtschranke öffnen lässt, “Geist“ zu. Zwar wirke sowohl beim Treten durch die Lichtschranke wie auch beim Öffnen der Türe durch Bedienung der Klinke lediglich der mechanistische Grundsatz von Ursache und Wirkung. Aber im ersten Fall entstehe der Eindruck eines „Geistes“ der Türe, der die Annäherung bemerke.

Mir erscheint da zum Beispiel die Auffassung von Dietfurth überzeugender, wonach zwar das Leben durch die Evolution erklärt werden könne, aber dem Bewusstsein alle Merkmale einer solchen Entwicklung fehlten, weshalb es als Erscheinung außerhalb von Raum und Zeit zu begreifen sei.

Das entspricht zum Beispiel auch den Ergebnissen der Chaosforschung. Sie kennt sogenannte autopoietische (sich selbst ursachenlos bildende ) Systeme.Führt man beispielsweise dem brodelnden Wasser in einem Kochtopf weitere Wärmeenergie zu, bildet sich ein Muster sechseckiger Waben auf der Wasseroberfläche, wobei die Größe der Waben nicht vorausberechenbar ist und ihre Anzahl wegen des kreisförmigen Topfrandes gegen Unendlich strebt.
Gibt man verschiedene farblich markierte chemische Flüssigkeiten in eine Pfanne, so bilden sich ebenfalls unvorhersehbar geometrische Formen wie zum Beispiel Spiralen.
Hier ist auch auf die Bildung sogenannter dissipativer Systeme zu verweisen.Geschlossene Systeme unterliegen zwar insgesamt betrachtet dem zweiten Hauptsatz der Wärmedynamik, wonach die Entropie (Unordnung) zunimmt, wenn sich diese Systeme selbst überlassen bleiben. Fernab dieses sogenannten Gleichgewichtes können sich aber durch ursachenlose Abgabe von Entropie an die Umgebung hochkomplexe Systeme bilden, die zwar stabil, aber extrem empfindlich gegenüber ihren Anfangsbedingungen sind. So nimmt die Entropie des Universums zwar insgesamt zu, Doch bilden sich immer mehr geordnete Systeme wie in bisher höchster Komplexität der Mensch.

Ähnliche “Zufälle“ kennen wir ja bei den Genmutationen und bei den Quantensprüngen.

Mathematisch gesehen handelt es sich dabei um Erscheinungen unendlich vieler Dimensionen, wie Prigogine hervorhebt. Es drängt sich hier die Parallelität zu den Auffassungen der christlichen Mystiker des Mittelalters auf, wonach sich im Zufall in seiner Unerklärlichkeit Gott offenbare.

Die Beobachtungsabhängigkeit der Erscheinungen im Mikrokosmos nach der Quantenphysik, weist im Elementaren darauf hin, dass erst das Bewusstsein die „Realität“ schafft.

Anton Zeilinger ( Quantenphysiker, * 1945 ) :

„Die Natur selbst ist immer nur unsere geistige Konstruktion.“

Niels Bohr (1885-1962; Quantenphysiker, Nobelpreisträger):

„Realität wird durch Beobachtung geschaffen!“

Erwin Schrödinger (1882-1961; Quantenphysiker; Nobelpreisträger):

„Die in Raum und Zeit ausgedehnte Welt existiert nur in unserer Vorstellung.“.



Die Erforschung des Bewusstseins ist nicht nur Gegenstand der Kognitionswissenschaften und der Neurobiologie, insbesondere der Hirnforschung, sondern auch der Informatik im Fachbereich AI ( artifizielle Intelligenz ).
Das Bewusstsein ist aber wohl nicht erforschbar, da es sich selbst zwar (beim Menschen) als solches erkennen, aber nicht erklären kann. Denn hierzu fehlt es an einer Metaebene der Betrachtung. Es gilt der Gödel´sche Satz, dass sich kein System aus sich heraus erklären kann.
Außerdem ist Erkennen eine Funktion des Bewusstseins, setzt dieses also bereits voraus und kann daher nicht auf das Bewusstsein selbst gerichtet sein.
Im Gegensatz zu der in den angelsächsischen Ländern herrschenden Meinung ( vgl. insbes. Tipler ) ist daher in Europa die Ansicht im Vordringen, dass Bewusstsein mehr als ein Algorithmus sei.


„Woher weißt du, dass das, was du haben möchtest, ein Apfel ist?“ Bertrand Russel

„ Wer weiß schon, ob das Leben nicht eigentlich Sterben und Sterben nicht eigentlich das Leben ist?“ Euripides

„Gehirn: ein Organ, mit dem wir denken, dass wird denken.“ Ambrose Bierce

„Ich denke, also bin ich nicht.“ Jean Cocteau

""Der Verstand sagt: "Scheinbar ist Farbe, scheinbar Süßigkeit, scheinbar Bitterkeit, in Wirklichkeit nur Atome und Leeres"; worauf die Sinne entgegen:"Du armer Verstand, von uns nimmst du deine Beweisstücke und willst uns damit besiegen? Dein Sieg ist dein Fall. "" Demokrit

Fortsetzung folgt!

Knut Hacker
28.09.11, 18:56
Fortsetzung:

„ David Hume sagte: 'Es gibt kein ICH. Das ICH ist ein gefühlter und kein bewusster Zustand.'

Ego-Seele-Selbst-Sein, letztendlich ein- und dasselbe? Alles bloss gefühltes Gefühl? Ein Konstrukt, gebildet im Gehirn. Gebildet, anhand von Bildern, eingebildet.“

Das zeigt sich auch beim Problem der Willensfreiheit. Selbstverständlich ist der Wille nicht frei (wovon denn?), sondern Produkt der ihn bildenden Faktoren (Anlage, Erfahrung usw.). Trotzdem fühlen wir uns frei in der Entscheidung, da wir sie „uns“ zurechnen.Wir identifizieren uns also mit den willensbildenden Faktoren, obwohl wir auf diese nur zu einem geringen Teil bewusstseinsmäßig Einfluss nehmen können.Genauso, wie wir uns mit unserem Leben identifizieren, obwohl wir bei unserer Zeugung ja nicht gefragt worden waren und gefragt werden konnten, ob wir ein solches auf uns nehmen wollen ( Die Natur hat zum Ausgleich für dieses „ Diktat der Geburt“ die Möglichkeit der Selbsttötung geschaffen, die jedoch keine Erlösung bringt, da man sie ja dann nicht mehr spürt).

„Aristoteles: Die Seele ist die Form des Leibes, die mit dem Tod des Leibes zugrunde geht. [Seele, Ego, Selbst-Leid usw. - Alles]“. Platon war anderer Ansicht. Und die Religionen sowieso. Die Physik geht davon aus, dass nichts verloren gehen kann (Energieerhaltungssatz, Entropiesatz). Wohin sollte es auch verlorengehen? Neben dem Sein bleibt nichts mehr übrig, das Nichts ist ein selbstwidersprüchlicher Begriff, da es dieses nicht geben kann, ohne dass es dann doch nicht nichts wäre . Aber das, was nach dem Tode nicht verloren geht, ist wohl nicht eine individuelle Seele, sondern allgemein „Geist“, wie er ja nach der Quantenphysik der Materie ( als Potentialität, Information) zugrundeliegt Letztlich ist alles Einheit. Auch die Quantenphysik und die Chaosforschung zeigen, dass alles mit allem zusammenhängt

Die Biophilosophie geht davon aus, dass Leben eine emergente Eigenschaft der Materie, Geist eine emergente Eigenschaft komplexer Gehirne ist. Geist ist demnach keine immaterielle Entität, sondern ein Sammelbegriff für alle individuellen geistigen Aktivitäten des Gehirns.



Funktioniert unser Gehirn wie ein Roboter und bilden wir uns das Bewusstsein nur ein ?.

Ich habe das Argument des "chinesischen Zimmers " dafür, dass Bewusstsein kein Algorithmus ist, gebracht .
Außerdem habe ich insbesondere Gödels Unvollständigkeitssätze dafür angeführt, dass eine Erforschung des Bewusstseins logisch unmöglich ist.

Dass das Bewusstsein lediglich ein Konstrukt des Bewusstseins ist ("Einbildung"),ist trivial und hat damit nichts zu tun.Kein Wissenschaftler bestreitet heute mehr,dass unser Denken genauso wie unser Fühlen, Werten und unsere sinnliche Wahrnehmungen lediglich Konstrukte des Gehirns sind. Die Frage aber ist doch,was diese Qualia,wie man sie bezeichnet, und damit auch das Bewusstsein als der Inbegriff aller Qualia und Überbegriff, oder gar das Bewusstsein des Menschen von seinem Bewusstsein (Selbsttranszendenz) darstellt.

Das Bewusstsein erschöpft sich jedenfalls nicht in der Funktion des Gehirns. Die diesbezüglichen wissenschaftlichen Erkenntnisse erheben auch überhaupt nicht den Anspruch, das Bewusstsein erklären zu können.

Wenn namhafte Wissenschaftler wie zum Beispiel Tipler davon ausgehen, dass der Mensch einschließlich seines Bewusstseins im weiteren Verlauf der Evolution durch Computer ersetzt würde, die sogar ins Geschehen des Universums eingreifen könnten, so liegt das an einem Verständnis von Bewusstsein, das dessen Selbstbezüglichkeit außer acht lässt.Das ist ungefähr genauso naiv wie die Fragen, „warum“ etwas ist und nicht nicht ist, oder, was „außerhalb“ von Raum und Zeit ist, oder,ob Gott „existiert“.


Schauen wir uns doch einmal die Hirnfunktionen näher an!

Was ist denn so eine Funktion als solche für eine Erscheinug? Eine komplexe dynamische Struktur sagen die Wissenschaftler. Hat also jedenfalls nichts mit Materie zu tun. Was sind denn die Eigenschaften der Materie (wie zum Beispiel die Gravitation durch Krümmung der Raumzeit) und die Beziehungen zwischen ihren Erscheinungen ( wie zum Beispiel Ursache und Wirkung )? Jedenfalls nichts Materielles.Was ist Energie? Nach der Relativitätstheorie in Materie unwandelbar und umgekehrt. Also wieder keine Materie.Energie lässt sich „ablesen“ am Rollen der angestoßenen Kugel. Aber das Rollen ist nicht die Energie! Was ist in der Biologie die Entelechie, zum Beispiel die Morphogenese? Wieder Fehlanzeige für Materialisten.Und wie will man materiell erklären, dass jedes Ganze etwas Anderes als die bloße Summe seiner Teile ist?.
Schließlich:
Vor mir steht ein Computer. Materie.
Was aber ist sein Sein? Sicherlich keine Funktion der Materie!
„Nicht wie die Welt ist, ist das Mystische, sondern dass sie ist.“ (Wittgenstein)


Zurück zu den Hirnfunktionen. Die Experimente haben lediglich gezeigt, dass diese Funktionen Ursache und Wirkung von Bewusstseinsvorstellungen sind. Nehmen wir einmal die Experimente von Newberg. Reizt man bestimmte Hirnregionen elektromagnetisch und löst dadurch entsprechende Hirnströme aus, kann man dadurch sogar persönlichkeitsfremde Bewusstseinsvorstellungen hervorrufen wie Marienerscheinungen bei Atheisten.Die Hirnfunktion also als Ursache für Bewusstseinserscheinungen, für eine Aktivierung des Bewusstseins.Umgekehrt kann man Hirnfunktionen natürlich nicht nur durch unmittelbare Beeinflussung, sondern – der natürliche Normalfall - auch durch externe Reizungen hervorrufen.Hält man besagtem Atheisten ein Marienbildnis vor, wird eine Aktivierung von Hirnbereichen hervorgerufen, die für Missmut, Ärger usw. zuständig sind.

Und die „Moral von der Geschichte“? Hirnfunktionen sind lediglich der physikalische Ausdruck von etwas Nicht – Raum – Zeitlichem,etwas Nicht - Physikalischem!Und das nennt man halt herkömmlicherweise Bewusstsein.Man kann ein Gehirn theoretisch atomgenau nachbauen.Es wird trotzdem nicht funktionieren! Das Ganze ist nicht lediglich die Summe seiner Teile.Und beim Bewusstsein kommt noch hinzu, dass es kein Ganzes ist, weil es nicht in Raum und Zeit einzuordnen ist und daher auch nicht abgrenzbar ist .

Zum wissenschaftlich überholten Dualismus von Geist (Bewusstsein usw.) und Materie ( Descartes) hat sich Demokrit wie folgt trefflich geäußert:

""Der Verstand sagt: "Scheinbar ist Farbe, scheinbar Süßigkeit, scheinbar Bitterkeit, in Wirklichkeit nur Atome und Leeres"; worauf die Sinne entgegen:"Du armer Verstand, von uns nimmst du deine Beweisstücke und willst uns damit besiegen? Dein Sieg ist dein Fall. ""


Die Tätigkeit des Bewusstseins ist in neurobiologen und neurochemischen Vorgängen des Gehirns ablesbar. Die Frage, ob diese Vorgänge Ursache oder Folge eines Bewusstseins sind, das Bewusstsein also lediglich eine dynamisch-komplexe Struktur der Materie darstellt oder außerhalb der Materie auf diese einwirkt, ist wohl eine Scheinfrage. Denn die Hirnaktivitäten sind atomarer Natur. Es gilt also der Quanteneffekt, das heißt hier, dass insbesondere folgende Unterscheidungen aufgehoben sind:
1) zwischen Ursache und Wirkung; (>Akausalitätsprinzip)
2) zwischen Materie und „Geist“ ( = Feld, Potentialität, Information); (>Nichtlokalitätsprinzip)
3) zwischen Subjekt (Person) und Objekt (Körper). (> Prinzip der Beobachtungsabhängigkeit)



Die Strukturdeterminiertheit unseres Bewusstseins,um diesen Terminus der Kognitionswissenschaften zu übernehmen, ist eigentlich augenfällig.Schon das kleine Kind fragt, was "blau" ist und stellt "Warum"-Fragen ad infinitum.
Und man kann jeden dadurch überzeugen, dass man ihn nicht nur auffordert, Sinneseindrücke und Gefühle zu beschreiben, sondern auch Vorstellungen wie Sein, Wahrheit, Ganzes, Gegensatz, Raum, Zeit, oder gar Wertungen wie Gut, Schön, Gerecht, Richtig usw. Es ist unmöglich.Unser Bewusstsein ist trotz der Bewusstheit seiner selbst selbstbezüglich.

Dagegen scheint das Problem der Bewußtseinsgrenzen zu sprechen.
Wie ist es möglich, dass unser Verstand und unsere Fantasie die „wahrgenommene“ Realität nicht nur abbilden und entsprechend unseren Erfahrungen hypothetisch konstruieren, sondern auch deren Grenzen überschreiten können, nicht nur hinsichtlich der real vorgegebenen Möglichkeiten (zum Beispiel bei der Vorstellung von Wunderkräften und bei Fantasiegebilden), sondern auch bis zur Unvorstellbarkeit (zum Beispiel beim Erdenken mathematische Größen wie „Unendlich“ und bei der Entdeckung der quantenphysikalischen Paradoxien)?

Handelt es sich bei diesen Grenzüberschreitungen lediglich um Interpolationen und Kontrapunktionen aus der Beziehungsweise zur Erlebniswelt, also immer noch um strukturdeterminierte Bewusstseinsinhalte oder um die Sprengung der eigenen Bewusstseinskraft, die Überwindung des Gödel´schen Grundsatzes, dass kein System über sich selbst hinausweisen kann?

Letzterenfalls wäre unser Bewusstsein wohl nicht allein individuell, also körperabhängig, sondern nähme über die Kommunikation der einzelnen Bewusstseinsträger mit ihren gleichartig konstruierten Bewusstheiten untereinander hinaus teil an einem überindividuellen Bewusstsein, so, wie etwa jeder Blutkreislauf nicht nur die Aktivitäten und damit auch die Interaktionen seiner Träger ermöglicht, sondern eben auch unter anderem über die Sauerstoffzufuhr am körperunabhängigen Medium Luft partizipiert.

Aber spätestens hier endet die Wissenschaftlichkeit der Diskussion!

amc
28.09.11, 20:09
Krottenfalsch schreibt man übrigens mit g.

Hey Nick,
das ist Schmarrn. Krottenfalsch schreibt man natürlich mit K, Grottenfalsch schreibt man mit G ;)


Freundlichst,
AMC

richy
28.09.11, 21:32
Hi Knut
Ein sehr interessanter Beitrag. Wobei ich die Diskussione des Existenzbegriffes bei unserer letzten Diskussion am aufschlussreichsten fand. Die Aufloesung der Russelschen Antinomie mittels einer nichttrivialen, erweiterten Mengenlehre.
http://de.wikipedia.org/wiki/Russellsche_Antinomie
Bei einer detallierten Betrachtung des Goedelschen Unvollstaendigkeitssatzes haben wir dann abgebrochen.
Was wir mir dazu noch in Erinnerung verblieb ist die Ueberlegung, wie sich der Unvollstaendigkeitssatz letztendlich physikalisch auswirken koennte. Es gibt Aussagen, die sind prinzipiell nicht Entscheidbar. Die rueckgekoppelte Aussage spielt bem Unvollstaendigkeitssatz schon eine Rolle, aber eine solche waere simulierbar. Verkoepert somit die mathematische Unlosbarkeit von physiklaischen Aufgabenstellungen schon den Satz oder beinhaltet er mehr ? Dass physikalische Phaenomen existieren koennen, die sich nicht aus grundlegenden Axiomen oder Hypothesen herleiten lassen. *Erinner Ich meine das war der Grund warum ich den G.US mal noch genauer unter die Lupe nehmen wollte.
Die vollstaendige Beschreibung aus dem eigenen System heraus. Ja, die ist im Grunde nicht moeglich. Wobei man natuerlich einen Zwillingsbruder beschreiben kann. Inwiefern hier eine moderne Mengenlehre Abhilfe schafft bin ich mir auch nicht ganz im klaren. Schade dass ich den Link zu dem Physiker nicht mehr finde, der hier sehr weit gekommen ist.

Eine Frage an dich : Wird in der Philosophie nur die Kopenhagener Deutung in Betracht gezogen oder auch die realistischen Varianten ? Und was ich nicht so ganz verstehe ist z.B. die Reaktion im AC Forum. Die Interpretation der QM wurde ja bewusst von den Physikern an die Philosophie und Kirche uebertragen. Wenn man diese nun ignoriert und als nicht diskussionswuerdig, geradezu esoterisch betrachtet, dann enstraeche dies der Ignoranz der Physik der Quantenmechanik. Wird dieser Prozess seitens der Philosophen verfolgt und bewertet ?
Viele Gruesse
richy

PS:
Ich meine eine Einteilung deiner vorgestellten Paradoxa in echte Paradoxa und scheinbare Paradoxa waere hilfreich.

@SCR
Bei AC bin ich auch Dauer gesperrt. Da kann ich nicht antworten.

fossilium
28.09.11, 22:49
Hallo Knut,

Materie sei ein in sich widerspruchsvolles Gebilde, ein Zwischengebilde zwischen Sein und Nichtsein, eine Hinneigung, Entelechie (ἐντελέχεια).
Eine verblüffende Parallele der Formulierungen zu den Quantenphysikern, wie ich sie in diesem Thread zitiert habe.

Vorsicht, Vorsicht. Da ist mir zuviel Spekulation. Das ist zu schwammig. Alles was von physikalischer Seite ausgesagt wird, ist entweder sehr präzise, oder wenn unsicher, dann mit präzisen Begründungen der Unsicherheit. Materie ist kein widerspruchsvolles Gebilde, wenn nicht vorher definiert wird, was unter Materie verstanden wird. Sie ist auch kein Zwischengebilde zwischen Sein und Nichtsein - was soll den das sein, wenn es Sein und Nichtsein schon unverstandene Begriffe sind. Und eine Hinneigung (Potentialität) ist Materie, sofern in materiellen Sinn verstanden, nur bei sehr hohen Energien (Potentialitäten), wobei man auch hier wieder mit den zugehörigen Definitionen weit ausholen muss.

Alles was die Griechen gesagt haben, lässt sich auf Heutiges hinbiegen, das war ja auch die totale Höchstkultur (> > Hochkultur).

Mich würde aber auch interessieren, wie Du die Fragen von Richy beantworten würdest, nämlich die zum Verhältnis von Mathematik und Physik. Hier ist insbesondere die Frage interessant: wenn es in der Mathematik, die die Naturgesetze formal beschreibt, unentscheidbare Ausagen gibt, ob sich das nicht bei der mathematischen Beschreibungen der Naturgesetze auswirken muss. Aber wie ?Ist zum Beispiel die Unmöglichkeit einer logische Beschreibung nicht-kausaler Ereignisfolgen auf den Gödelschen Satz rückführbar ? Oder allgemein gefragt: Wie siehst Du die Auswirkungen des Gödelschen Satzes in seinen Folgen für die formale mathematische Beschreibung der Physik ?

Oder kann das vielleicht nur ein Mathematiker beantworten ?
Grüsse
Fassilium

amc
29.09.11, 00:17
In der Quantenphysik dagegen spielt das Bewusstsein beim Paradox des Kollapses der Wellenfunktion von Elementarteilchen ( sog. Superposition) durch die Beobachtung (sogenanntes Messproblem; makrokosmisch parabolisiert durch „Schrödingers Katze“) doch eine zentrale Rolle!

Hi Knut,
wenn du damit das Bewusstsein eines lebenden Organismus meinst, dann ist das jedenfalls nicht die übliche Ansicht. Das Bewusstsein eines lebenden Organismus spielt üblicherweise in modernen Interpretationen der Quantenphysik keine zentrale Rolle. Definitiv nicht. Das muss man hier mal deutlich sagen. Mit sowas muss man vorsichtig sein. Jeder ist natürlich so frei die Dinge für sich selbst zu bewerten.


Freundlichst,
AMC

richy
29.09.11, 01:19
Ablauf der Ereignisse :
Wir stellten experimentell physikalische Phaenomene fest (Nichtlokalitaet), die in unserem Anschauungsraum nicht rein physikalische gedeutet werden konnten.
Die Interpretation ueberliessen wir vornehmlich den Philosophen (Auch Zeilinger ist philosophisch gebildet)
Als realistische, physikalische Ansaetze fuer die Deutung erarbeitet wurden, erschien uns diese physikalische Deutung, ein solch zwingendes physikalisches Verhalten der Quantennatur zu skurill. So verblieb die Aufgabenstellung ueber 80 Jahe bei den Philosophen.
Im Grunde wuerde heute kein Physiker einen Philosophen befragen, aber so ganz koennen wir unsere alte Fragestellungen nun ja auch nicht ignorieren.
Also lassen wir einen Philosophen sprechen, zu welchen Erkentnissen bezueglich unserer abgegebenen Aufgabenstellung sie gekommen sind.
Und natuerlich wird die nichtphysikalische Loesung genauso nichtphyskalisch verbleiben wie wir sie vor 80 Jahren abgegeben haben. Denn Philosophen sind Geisteswissenschaftler.
Auch manche Physiker vertre(a)ten natuerlich die nichtrealistische Deutung :

Werner Heisenberg (1901-1976 ; Quantenphysiker-“Unschärferelation“-; Nobelpreisträger):

„...dass nicht einmal die Eigenschaft des “ Seins“, wenn man hier überhaupt von Eigenschaft reden will, dem Elementarteilchen ohne Einschränkung zukommt. Es ist eine Möglichkeit oder eine Tendenz zum Sein“ (Physik und Philosophie).

Hans-Peter Dürr (Quantenphysiker,*1929):
“Es gibt … gar nichts Seiendes, nichts, was existiert.“


Anton Zeilinger ( Quantenphysiker, * 1945 ) :
„Die Natur selbst ist immer nur unsere geistige Konstruktion.“



Tja wie meinen diese Physiker das denn nur ? Das kann doch nicht sein dass diese angesehenen Physiker dies so meinen wie sie es schreiben. Das waere doch fast Esoterik. Der kleine Mann kommt zur Erkenntnis : Also muessen sie es ganz anders meinen.
Man koennte jedoch auch ganz anders fragen : Was koennen diese Physiker unter Beibehaltung einer nichtrealistischen Deutung denn auf keinen Fall aeussern/meinen ? (Das ueberlasse ich mal als Denksportaufgabe)

Aber bleiben wir bei den Philosophen. Da ist es naemlich weitaus einfacher.
@AMC
Du hast gerade demonstriert wie man sich der Nichtlokalitaet locker entledigt.
Man argumentiert einfach : Na das sind die Ansichten eines Philosophen und ignoriert, dass dies die Ansichten der Schulphysik sind. Und Schwupp hat sich das Problem scheinbar in Luft aufgeloest.
Nochmal die einfache Ablaufkette :

Unloesbares Problem -> Abgabe an Philosophen -> Etwas warten und umruehren -> Ignorieren des Problems da es nun ein Philosoph aeussert -> Problem geloest.

Naja so gehts ja nicht, also wird sich das Problem mit der Dekohaerenz wohl in Luft aufgeloest haben. Hier gebe ich zu bedenken :

Wenn du einen Loewenkaefig mit darin enthaltenem Loewen oeffnest. Ich kann die garantieren. Der Loewe wird durch diese Maßnahme, dem oeffnen der Tuere, sich nicht in Luft aufloesen. :-)

Viele Gruesse

richy
29.09.11, 01:48
Noch ein taktischer Hinweis.
Der Philosoph oder die Kirche wird wahrscheinlich nie einen Realismus ernshaft in Betracht ziehen, denn dann waeren beide wenisgtens diese Aufgabenstellung los. Die Kirche zudem die Feinabstimmung der Naturkonstanten.
=> ?

amc
29.09.11, 01:58
Tja wie meinen die das denn nur ? Das kann doch nicht sein dass diese angesehenen Physiker dies so meinen wie sie es schreiben.

Hi Richy,
erstmal so viel:

hier zitiert (http://www.psychophysik.com/html/re025-zeilinger-anton.html)
info3:
Sie erwähnten vorhin die Notwendigkeit, beim Sprechen von Quanten immer den gesamten Apparat zu berücksichtigen. Gehört der Beobachter für Sie auch dazu?
Zeilinger:
In einem gewissen Sinne schon. Der Beobachter wählt den experimentellen Aufbau aus und definiert ihn. Ich würde allerdings nicht so weit gehen, zu sagen, dass das Bewusstsein des Beobachters einen unmittelbaren Einfluss auf das Geschehen hat. Manche Kollegen gehen soweit. Für mich persönlich gibt es dafür keinen Grund.

Mit aus dem Kontext gerissenen Zitaten muss man also immer vorsichtig sein, sie können falsch verstanden werden. Habe mir deinen Text noch nicht ganz durchgelesen. Werde ich bestimmt noch drauf eingehen. Gut's Nächtle ...


Freundlichst,
AMC

richy
29.09.11, 02:19
Ich würde allerdings nicht so weit gehen, zu sagen, dass das Bewusstsein des Beobachters einen unmittelbaren Einfluss auf das Geschehen hat.Manche Kollegen gehen soweit. Für mich persönlich gibt es dafür keinen Grund.
Wer der Beobachter ist und wessen Bewusstsein damit gemeint ist, haengt davon ab, an welcher Stelle man den Heisenbergschnitt ansetzt. Und dieses Bewusstsein ist immer unwaegbar. Und fuer Zeilinger ist dies die Information in der Natur. Das Universum beobachtet sich selbst. Der Unterschied zum Realismus ist aber, dass dies dann keine physikalische realistische Beobachtung = Messung sein darf.
Realismus / Dekohaerenz : Das Universum misst sich selbst
Kopenhagen / Dekohaerenz : Das Universum beobachtet sich selbst (Wheeler)

Aber wie schon besprochen: Jede Informtion benoetigt einen physikalischen Traeger. Zeilinger vertritt dennoch die Kopenhagener Interpretation (sagt er). (Er betont ausdruecklich dass er keinen Positivismus vertritt (KD)). Was passiert denn, wenn der Heisenbergschnitt entfaellt ? Wenn die Schranke aufgeht ? Wenn die VWI argumentieren wuerde, Naja diese Viele Welten existieren physikalisch nach der Dekohaerenz ueberhaupt nicht mehr. Wuerde sich dann daran jemand stoeren ? Und geht Zeilinger davon aus, dass die Information in der physikalischen Welt verschwindet ? Die KI hat dazu aber noch ein ganz anderes Problem. Existiert kein Wellenkollaps, kein Heisenbergschnitt , wie vollzieht sich dann diese Umwandlung oder Manifestation ins Physikalische ?

Klar. Zeilinger ist sehr vorsichtig und er hat allen Grund dazu. Aber er legt wenigstens ein Teil der Karten offen auf den Tisch . Bei solch einer Gratwanderung voellig angebracht und ihm geht es vor allem um die Moeglichkeit seine Versuche durchzufuehren.

Die beste Methode ist tatsaechlich sich immer zu ueberlegen was die KI nicht annehmen darf um in einen Realimus zu verfallen. Und dass stets die Nichtlokalitaet erklaert werden muss.Dekohaerenz alleine erklaert keine Nichtlokalitaet. Mein Physiklehrer wusste das uebrigends schon vor 30 Jahren. Denn als ich ihm mal vorschlug, dass es doch eine einfache Erklaerung waere, dass die Natur sich selber misst, war ich ganz erstaunt, dass er diese tolle Idee gar nicht gut fand.

Aus dem Kontext gerissen .... Zweilingers Zitat sehe ich in deiner Fassung gar nicht. Was Zeilinger natuerlich immer aeussern kann, ist, dass in seiner Vorstellung (entgegen Kollegen) kein menschlicher Beobachter notwendig ist. Es darf aber auch fuer ihne kein physikalischer Beobachter sein, denn der waere dann hochdimensional. Rein geistige Beobachter sind in der Natuer aber sehr selten :-) .... Einen solchen muesste er sich konsequenterweise ausdenken. Das wird er aber nicht tun, sondern einen Kompromiss suchen.
Und meiner Meinung nach wird es noch sehr lange dauern bis man die Vorgaenge zwischen Information und Informationstraeger erklaeren kann. Vielleicht gelingt dies sogar ueberhaupt nicht. Darauf weist indirekt auch Penrose hin.

Ich wollte aber die Meinungen von Knut nicht unterbrechen ...
Das Bewusstsein eines lebenden Organismus spielt üblicherweise in modernen Interpretationen der Quantenphysik keine zentrale Rolle. Definitiv nicht. Das hier kann man halt nicht einfach so stehen lasen. Was ist modern ? Und bei Zeilinger ist das definitiv das "Bewusstsein" besser Information im Universum. Blos sagt er das nicht so deutlich wie zum Beispiel in diesem Bild :
http://www.thur.de/philo/project/qt3.gif
(Wheeler)

Wheeler führte mit Bryce DeWitt die Wheeler-Dewitt-Gleichung als eine Wellenfunktion des gesamten Universums ein und stand dem Realismus nahe.
Angelich vetrat er aber auch eine informtionstheoretische Deutung. Fuer mich waere das kein Widerspruch.

Viele Gruesse

Nick Rymer
29.09.11, 09:48
Welch reichhaltige Beiträge, Knut Hacker!
Die behavioristische, materialistische wissenschaftliche Auffassung, wonach Bewusstsein lediglich eine emergente Eigenschaft der Komplexität ist, stützt sich gerne u.a. auf das Gleichnis vom „chinesischen Zimmer“: In einem abgeschlossenen, uneinsehbaren Zimmer befindet sich ein Mensch, der nur Deutsch versteht.und Regale, in denen, auf Papierbögen geschrieben, alle chinesischen Sätze wohl sortiert archiviert sind.Ein Katalog enthält alle deutschen Sätze und verweist auf die entsprechenden chinesischen. Das Zimmer ist mit der Außenwelt durch einen Schlitz verbunden, durch den Papierbögen ein- und ausgeschoben werden können.Den Außenstehenden wird folgende Information erteilt: Im Zimmer befinde sich ein chinesischer Dolmetscher und sonst überhaupt nichts.Ein Außenstehender will einen Satz übersetzen lassen. Er schreibt ihn auf einen Papierbogen und reicht ihn in das Zimmer.Der angebliche Dolmetscher sucht anhand seines Kataloges den entsprechenden chinesischen Satz und reicht ihn hinaus.Der Außenstehende wird dem angeblichen Dolmetscher Kenntnisse des Chinesischen zuschreiben, obwohl sich die Beherrschung einer Sprache nicht in der Kenntnis von Wörtern und Grammatik erschöpft, sondern Bewusstseinstrukturen wie Abstraktion, Kombination, Intuition u.s.w.voraussetzt.
An dieser Stelle möchte ich auf meinen Glauben an die Experimentalphysik stützen, was ich mithin als die einzigen ernstzunehmenden Fakten akzeptiere. Jede Theorie muss sich auf diese Fakten stützen. Die darauf dann folgende Diskussion aber ist m.E. sehr gut mit deinem Gedankenexperiment beschrieben.
Das Bewusstsein eines Menschen, einer Gruppe ist sehr häufig von Macht bestimmt. Menschen suchen Sicherheit und sie stellen sich hinter denjenigen, der diese am ehesten zu versprechen scheint. Ihr „Bewusstsein“ kopieren sie aus dem Bewusstsein dessen, der ihnen Sicherheit anbietet. Das ist auch im 3. Jahrtausend immer noch der Mächtige. Menschen scheinen instinktiv zu wissen, dass es das absolute Bewusstsein nicht gibt (da hast du mich überzeugt). Somit kann das praktisch absoluteste Bewusstsein, nur der Mächtige halten.
Macht selbst allerdings funktioniert jedoch nicht nur mit fachlicher Kompetenz. Das Kriterium Macht als Indikator höchsten Bewusstseins muss fehlgehen, denn wir alle wissen, dass die Wege zur Macht weiß Gott vielschichtig sind. So ist es vor diesem Bewusstseinskonstrukt der Massen durchaus möglich, dass sich das Bewusstsein weiter und weiter von der Wahrheit entfernt und nicht, wie du es eingangs vermutetest, auf das Bewusstsein das Überbewusstsein folgt usw.
Die Konstruktion des Menschen ermöglicht ebenso gut die Massenpsychose.
Mir erscheint da zum Beispiel die Auffassung von Dietfurth überzeugender, wonach zwar das Leben durch die Evolution erklärt werden könne, aber dem Bewusstsein alle Merkmale einer solchen Entwicklung fehlten, weshalb es als Erscheinung außerhalb von Raum und Zeit zu begreifen sei.
Alein der Umstand, dass es Entwicklungsfähigkeit im Bewusstsein gibt, deutet m.E. darauf hin, dass es irgendwo da draußen das absolute Bewusstsein gibt, von dem wir uns ja schließlich zumindest angezogen fühlen – wenn auch mit kleinen Psychosen, doch selbst diese verschärfen ja das Bewusstsein gegenüber sich selbst. Der Mensch kann sein Denken und Handeln schließlich reflektieren, und wer weiß, womöglich liegt selbst in der Massenpsychose (zB III. Reich) noch ein Körnchen negativer oder sogar positiver Wahrheit.
„Woher weißt du, dass das, was du haben möchtest, ein Apfel ist?“ Bertrand Russel
Ja, mir wurde dann auch klar, dass sich unser Bewusstsein immer nur auf die Erfahrungen stützen kann, die wir schon gemacht haben. Wie gesagt, es erscheint mir wie die DGL A. Einsteins zur Schwerkraft: Die Dinge existieren und entwickeln sich aus sich selbst heraus, da sie a) womöglich miteinander verwandt sind und b) es jedoch grundlegende Gesetze gibt, vor denen der Umstand „Entwicklung“ immanent ist.
"Der Verstand sagt: "Scheinbar ist Farbe, scheinbar Süßigkeit, scheinbar Bitterkeit, in Wirklichkeit nur Atome und Leeres"; worauf die Sinne entgegen:"Du armer Verstand, von uns nimmst du deine Beweisstücke und willst uns damit besiegen? Dein Sieg ist dein Fall. "" Demokrit
a) denke ich, uns Menschen würde genügen zu erklären, was wir sinnlich erfahren und b) ist unser Wirken weniger ein Kampf, in dem es um Sieg oder Niederlage geht. Wir sind nur neugierig, was vielleicht nerven kann.

Mit dem Durcharbeiten der ersten zwei deiner gestrigen Beiträge komme ich zu dem Schluss, das Erkenntnis am Bilde der Physik durchaus massebehaftet ist.
So wie unsere Erfahrungen doch allesamt auf größtenteils sinnlicher Erfahrung beruhen - und damit auf Gefühlen - und andererseits diese uns wieder und wieder von der Richtung abweichen lassen, so gibt es zudem auch noch das träge Beharrungsvermögen unseres Bewusstseins. Die Trägheit der Masse unseres Erkenntnisvermögens ist daher vermutlich das, was wir unser Leben nennen, die Gefühle. Die schwere Masse ist auf der anderen Seite hingegen unser Wunsch, doch endlich, endlich am Ziel angelangt zu sein – wieder ein Gefühl.

Damit kann ich mich nicht wehren, aus diesen irdischen Einsichten heraus dir recht zu geben: Eine Metaebene gibt es für uns nicht. Es gibt für sie nur ein Indiz, nämlich den Sog in uns hin zum absoluten Bewusstsein, die Neugier. Gäbe es diesen Sog nicht, gäbe es keine Erkenntnis, kein Bewusstsein und wer weiß, vielleicht nicht einmal Gefühle oder gar Leben. Das Karussell hätte nie zu Drehen angefangen.

Fortsetzung folgt!

amc
29.09.11, 10:28
Das hier kann man halt nicht einfach so stehen lasen. Was ist modern ? Und bei Zeilinger ist das definitiv das "Bewusstsein" besser Information im Universum.

Hi Richy,
darum hab ich in meiner Antwort auf Knuts Beitrag gefragt, ob er damit das Bewusstsein eines lebenden Organismus meint, also das individuelle Bewusstsein, welches wir uns alle üblicherweise zusprechen. Ich glaube nämlich so hat er es gemeint, zumindest habe ich es nicht so verstanden, dass er an jener Stelle ein übergeordnetes Verständnis von Bewusstsein meinte.

Und mit modern wollte ich andeuten, dass man es möglicherweise früher, in den Anfangszeiten der QM, etwas anders gesehen hat.

Aus dem Kontext gerissen .... Zweilingers Zitat sehe ich in deiner Fassung gar nicht.

Nein, so war es nicht gemeint. Ich weiß nicht woher Knuts Zeilinger Zitat stammt. Die Zitate wirkten in dem Zusammmenhang, als würden Zeilinger, Dürr und Heisenberg dem bewussten Beobachter eine entscheidene Rolle zusprechen. Ich weiß nicht ob dies von Knut beabsichtigt war.

Ihre Aussagen, so wie ich es verstehe, beziehen sich jedoch darauf, dass eben das Messergebnis(Eigenschaft des Objektes) als solches nicht absolut und stetig existent vorliegt, und man es nur abzulesen braucht, sondern es es wird erst durch den Messvorgang geschaffen. Diesen Umsatnd drücken die drei aus. Dabei geht es aber nicht um die Rolle eines bewussten Beobachters.

Weil die angeführten Zitate missverstanden werden können habe ich dann auch ein Zeilinger Zitat angeführt, aus dem hervorgeht, dass er dem individuellen Bewusstsein des Beobchters keine entscheidene Rolle zuspricht.

Du hast gerade demonstriert wie man sich der Nichtlokalitaet locker entledigt.

Warum? Ist mir unverständlich. Kannst du das nochmal lockerleicht präzise und knapp nahebringen? Wenn man annimmt, dass es schlicht Wechselwirkungsprozesse sind, die Realität schaffen, dann ändert das doch nichts an der Nichtlokalität?

Ich sehe es so: Verschränkung, Nichtlokalität etc. lässt sich noch nicht erklären, weil wir es noch nicht vertehen. Wenn man es nicht versteht ist eine Nichterklärung möglicherweise vorerst die beste Erlärung. Spekulieren und annehmenen kann man ja quasi unendlich Vieles. Ich bin aber fest überzeugt, diese Phänomene werden sich irgendwann, wenn wir ein besseres Verständnis von Raum, Zeit und Materie gewonnen haben im Grunde von selbst erklären. Wir müssen einfach abwarten und vorerst akzeptieren, dass wir noch keine Lösung gefunden haben.


Freundlichst,
AMC

Nick Rymer
29.09.11, 10:42
Fortsetzung:

Massebehafteter Charakter der Erkenntnis bringt es mit sich, die Neugier ins Auge zu fassen. Ist nicht auch Gravitation letztlich eine Art Gier? Massen sind begierig, andere Massen zu erreichen, ja, zu sich zu ziehen. Wenn die Neugier uns Richtung absolutes Bewusstsein zieht, muss da nicht irgendwo draußen oder systemimmanent etwas Gierendes wohnen, eine Masse, ein Gravitationspotential? Das absolute Bewusstsein, dessen Trachten es ist, sich mit kleineren Erkenntnissen zu vereinen, indem sie diese zu sich zieht und teilhaben lässt an ihrem Reichtum?
So oder so ähnlich könnte es doch sein?

Doch zu deinem letzten, gestrigen Beitrag.
Aber das, was nach dem Tode nicht verloren geht, ist wohl nicht eine individuelle Seele, sondern allgemein „Geist“, wie er ja nach der Quantenphysik der Materie ( als Potentialität, Information) zugrundeliegt Letztlich ist alles Einheit.
Mal so am Rande: Nimmst du auch deswegen hier teil?
Geist ist demnach keine immaterielle Entität, sondern ein Sammelbegriff für alle individuellen geistigen Aktivitäten des Gehirns.
Geist, in meinem Sinne, kann durchaus auch abstrakten Dingen innewohnen – zB Geld. Denn Geist ist eine gestaltende, Kraft. Nicht nur im Hirn. Was wir in unserem Bewusstsein reflektieren, ist der Geist, der den Dingen innewohnt – zB den Quanten. Der Geist tut sich kund im Verhalten eines Systems – zB der Mensch, aber der Mensch ist auch nur ein System von vielen. So gesehen wohnt durchaus auch einem Bienenstock Geist inne. Wäre dem nicht so, würde er uns gar nicht interessieren. Geist wohnt auch der Masse inne, wie ich oben beschrieb. Nichts unserer bekannten Welt ist unter dieser Sicht ohne Geist. Deswegen ist die Welt ja so interessant: Geist oder/und Bewusstsein ist massebehaftet/ gravitativ und weckt die Neugier, den Wunsch, alles zu vereinen, zumindest im Geist – das absolute Bewusstsein.

Es ist doch vermessen, Dinge wie Geist und Bewusstsein immer nur auf sich selbst zu beziehen. Egomanie. Wir sind Produkte der Natur, und alles, dessen wir uns rühmen, ist bereits irgendwo in der Schöpfung wiederzufinden. Vor uns entwickelte Naturprodukte besitzen zwar nur Teile dessen, was wir in uns tragen, vielleicht sogar versteckt, aber diese Teile sind vor uns entwickelt oder aber zumindest bereits in der Entwicklung befindlich. Somit ist es ratsam für den Menschen, aus seinem Kämmerlein herauszutreten und sich umzuschauen, wo denn diese oder eine andere seiner Eigenschaften bereits in viel ursprünglicherer Form vorliegen und sich damit leichter einer Beurteilung durch unser fortentwickeltes Bewusstsein kundtun. Denn eine Beurteilung unserer Eigenschaften aus unserem System heraus ist, wie du vehement vertratst, aussichtslos. Bewusstsein liegt m.E. in seinen Ansätzen schon viel früher in der Schöpfung vor und bietet sich damit der Erkenntnis durch den Menschen an.
Die Experimente haben lediglich gezeigt, dass diese Funktionen Ursache und Wirkung von Bewusstseinsvorstellungen sind.
Scheinbar scheint niemand das Leben ins Kalkül mit ein zu beziehen. Mir scheint sogar, bis auf die Erforschung biochemischer Prozesse, ist hierzu niemandem etwas bekannt. Es kommt mir vor wie das kosmolog. Standardmodell. Man rechnet im statischen Begrenzungsraum. Das Leben jedoch Bewegung bedeutet, fällt niemandem ein. Die Triebfeder der Bewegung ist vor dem Horizont unseres Bewusstseins nicht ersichtlich, dabei gilt es doch nur zu akzeptieren, dass uns , allem das Leben geschenkt wird, da sich das Gesamtsystem auf die Existenz eines Außerhalb stützt, welches uns in Bewegung versetzt, und um mir jetzt nicht EMIs Rüffel einzuhandeln, nenne ich dieses einmal Gott, auch wenn ich ihm einen anderen Namen geben könnte.
Das Ganze ist nicht lediglich die Summe seiner Teile.
Füge den Teilen das Außerhalb hinzu, und du erkennst, warum das Ganze mehr ist als die Summe seiner Teile.
Letzterenfalls wäre unser Bewusstsein wohl nicht allein individuell, also körperabhängig, sondern nähme über die Kommunikation der einzelnen Bewusstseinsträger mit ihren gleichartig konstruierten Bewusstheiten untereinander hinaus teil an einem überindividuellen Bewusstsein
Eine schöne Idee. Dann wäre die Menschheit Gott im Geiste. Ändert aber nichts an den Ursachen zB der Neugier. Gott wäre mit Sicherheit ein anderes Wort für das absolute Bewusstsein – und so stellt sich mir die Frage, weshalb, so lange es Menschen gibt, ihnen ein solcher im Bewusstsein klebt.

Freundlichst
Nick

Nick Rymer
29.09.11, 10:45
Hi amc,
das ist Schmarrn. Krottenfalsch schreibt man natürlich mit K, Grottenfalsch schreibt man mit G ;)
Von dem her......so gesehen.......
Es is ja wies is:rolleyes:

Gruß, Nick

Nick Rymer
29.09.11, 11:08
Hi richy,

Die vollstaendige Beschreibung aus dem eigenen System heraus. Ja, die ist im Grunde nicht moeglich. Wobei man natuerlich einen Zwillingsbruder beschreiben kann.
Ich habe da nur eine Ahnung, was du mit einem Zwillingsbruder meinst.
Könntest du mich informieren?

Gruß, Nick

Nick Rymer
29.09.11, 11:20
info3:
Sie erwähnten vorhin die Notwendigkeit, beim Sprechen von Quanten immer den gesamten Apparat zu berücksichtigen. Gehört der Beobachter für Sie auch dazu?
Zeilinger:
In einem gewissen Sinne schon. Der Beobachter wählt den experimentellen Aufbau aus und definiert ihn. Ich würde allerdings nicht so weit gehen, zu sagen, dass das Bewusstsein des Beobachters einen unmittelbaren Einfluss auf das Geschehen hat. Manche Kollegen gehen soweit. Für mich persönlich gibt es dafür keinen Grund.
Also, wenn ich das mal ganz irdisch beurteilen darf: Der Beobachter baut den Versuch nach seinem Bewusstsein auf. Die Antworten, die er bekommt, beziehen sich demgemäß auf sein Bewusstsein - und nicht etwa auf ein absolutes Bewusstsein! ;)

Aber auch die Versuchsergebnisse des Versuchsaufbaus aufgrund des aktuellen Bewusstseinsstandes werden Informationen des absoluten Bewusstseins beinhalten.
Oder etwa nicht?

Gruß, Nick

P.S.: Wir wollen nicht den Faden verlieren. Guckst du auch diesen Beitrag (http://www.quanten.de/forum/showthread.php5?t=2033/#7).

richy
29.09.11, 16:27
Hi
wenn es in der Mathematik, die die Naturgesetze formal beschreibt, unentscheidbare Ausagen gibt, ob sich das nicht bei der mathematischen Beschreibungen der Naturgesetze auswirken muss. Aber wie ?Ist zum Beispiel die Unmöglichkeit einer logische Beschreibung nicht-kausaler Ereignisfolgen auf den Gödelschen Satz rückführbar ?
Das halte ich fuer eine sehr interessante Aufgabenstellung. An den GUS wird man z.b. im Buch Goedel Escher Bach ueber 800 Seiten anfangs systematisch herangefuehrt. Aber nicht alle Beispiel gehen dann wirklich in den GUS ein. Goedels Beweis des US ist sogar recht kurz.
Oder kann das vielleicht nur ein Mathematiker beantworten ?
Irgendwo hab ich einen Link zu Goedels Original Paper hier angegeben.

Das Bewusstsein eines Menschen, einer Gruppe ist sehr häufig von Macht bestimmt. Menschen suchen Sicherheit und sie stellen sich hinter denjenigen, der diese am ehesten zu versprechen scheint. Ihr „Bewusstsein“ kopieren sie aus dem Bewusstsein dessen, der ihnen Sicherheit anbietet.So ist es. Dass dies auch in der Physik der Fall ist, ist fuer mich uebrigends eine der groessten Enttaeuschungen. Das Ganze ist evolutionaer bedingt. Der Mensch war noch nie ein Einzelgaenger und kann nur in einem sozialen Netzwerk ueberleben. Genauso wie eine Ameise. Letztere gehoeren zu den erfolgreichsten Lebewesen ueberhaupt. Die Methode muss somit nicht negativ sein, wenn sie richtig angewendet wird.
Von einem absoluten Bewusstsein hab ich dagegen kaum eine Vorstellung.
Ist damit die Summe aller nichtwaegbaren Objekte gemeint ?
Naeher kenne ich etwas vom Familienbewusstsein, etwas vom "Nationalbewusstsein" (das kann auch Europa sein), dem Weltbewusstsein, etwas vom Fussballclubbewusstsein, dem Bewusstein der Espressofeteschisten :-) ...
Davon haben wir aber im Gegensatz zu unserem eigenen Bewusstsein immer nur eine vage Vorstellung. So wie die Ameise immer ganz zielstrebig weiss was sie zu tun hat. Aber von diesem ganze Ameisenstaat, da hat sie keinen blassen Schimmer.

Die Trägheit der Masse unseres Erkenntnisvermögens ist daher vermutlich das, was wir unser Leben nennen, die Gefühle. Ich bin durchaus ein Freund von Analogien, aber in dr Art kann man Bewusstsein nicht ploetzlich als waegbar betrachten. Es besteht auch kein Grund dazu.

darum hab ich in meiner Antwort auf Knuts Beitrag gefragt, ob er damit das Bewusstsein eines lebenden Organismus meint, also das individuelle Bewusstsein, welches wir uns alle üblicherweise zusprechen.
Das haengt bei Kopenhagen wie bereits erwaehnt davon ab wo man den Heisenbergschnitt ansetzt. Es gibt hierzu alle moeglichen Varianten und undurchsichtige Argumentationsketten, die im Grunde unnoetig sind, wenn man sich stets die eigentliche Aufgabenstellung vor Augen fuehrt. Und das ist die Nichtlokalitaet. Wenn ich die Nichtlokalitaet mit abstraktem, unwaegbarem erklaere, dann habe ich nunmal den Geist in die Welt gesetzt. Und den krieg ich durch drumherumreden nicht mehr los. Wenn der Bebachter der KI eine Rolle spielt, dann ist es die unwaegbare Komponente desselben. Und wenn das ein Mensch ist, ist es zwangslaufig das Bewusstsein. Anders lassen sich solche Meldungen auch gar nicht erklaeren :

http://www.spiegel.de/wissenschaft/weltall/0,1518,518990,00.html
Das führt zu dem radikalen Schluss, dass wir durch unsere Beobachtungen tatsächlich das All schneller sterben lassen würden!
.....
"Sehr gewagte Interpretation"
Das ist keine gewagte Interprettion sondern im Rahmen der KD eine sehr Konsequente.
Max Tegmark vom Massachusetts Institute of Technology verwies gegenüber dem "New Scientist" darauf, dass der Quanten-Zeno-Effekt nicht zwangsläufig Menschen erfordere. "Galaxien haben die dunkle Energie schon 'gesehen', bevor es uns gab", sagte er. "Wenn wir Menschen Licht ferner Galaxien beobachten, dann verändert das nichts außer unserem eigenen Wissen."
Genau, so einfach ist das. Denkt man. Dieser Max Tegmark hat das vernuenftig erkannt. Denkt man. Blos wer ist Max Tegmark ?
Wiki :

Tegmark has also formulated the "Ultimate ensemble theory of everything", whose only postulate is that "all structures that exist mathematically exist also physically".
Existiert PSI in der KI physikalisch ? Tegmark ist natuerlich Realist ....
und daher geht seine Aussage in Ordnung. Der darf so argumentieren.
Ein Vertreter der Schulphysik nicht ! Das wird im Spiegel aber nicht differenziert und so ist man in der allgemeinen Verwirrung wieder ein Stueck weiter gekommen :-)

He developed the quantum suicide thought experiment from earlier proposals by Hans Moravec and Bruno Marchal, and has come up with a mathematical argument for the multiverse.
.....

He has also been a strong critic of those who would infer a theory of consciousness from quantum effects, such as Roger Penrose and Stuart Hameroff.
http://de.wikipedia.org/wiki/Roger_Penrose
Aufgepasst. Penrose ist Realist aehnlich wie Wheeler. Und genauso wie diesr nimmt er ohne eine zwingende Notwendigkeit ein Bewusstsein an. (Bei der KI ist die Annahme zwingend) Vielleicht stoert dies Tegmark um so mehr.

Gruesse

Nick Rymer
29.09.11, 17:12
@richy,
unbeholfen wie ich bin wage ich ein weiteres Mal, und diesmal zur Erklärung der Nichtlokalität, meine These vom Außerhalb ins Spiel zu bringen:
Mancher spricht von wirklich räumlichen zusätzlichen Dimensionen, mancher rollt sie auf. Beidem gemeinsam ist zwingend, dass jedem Objekt, hier vorzugsweise ein Quant, eine Verbindung zum Zentrum des Begrenzungsraumes zur Verfügung steht. Zumindest in Gestalt einer Distanz.
Wie ich hier (http://www.quanten.de/forum/showthread.php5?t=2033/#7) schrieb, möge es einmal sein, Quanten wohnt permanent nicht nur ein Zustand inne, sondern alle Zustände sind zu jeder Zeit nebeneinander präsent. Das Problem der Dominanz eines bestimmten Zustandes sei vorerst dahingestellt. Daraus stellt sich jedoch die Frage, wo mögen sich diese parallelen Zustände aufhalten?
Kreativ, wie ich bin, schlage ich vor, diese Zustände befinden sich irgendwo auf der Linie zwischen dem Jetzt und seiner Verbindung zum Zentrum des Begrenzungsraumes - in der zusätzlichen Dimension also. Dann ist klar, welchen Weg die Information im Rahmen der Nichtlokalität nimmt. Sie wählt den kürzesten Weg über die, jetzt aufgerollte, vierte räumliche Dimension hin zum Zentrum und zurück zum Wirkungsempfänger. Das Zentrum hat somit die Aufgabe eines Satelliten.

Viele Druiden, Schamanen, Esoteriker und Heipraktiker klatschen mir Beifall, verifizieren sich doch so all ihre Vorstellungen. Doch beileibe, ich denke, alles hat seine Grenzen. Dennoch ist zB die Verbindung zwischen Mutter und Kind über Grenzen hinweg stabil. Die brauchen nicht mal Telefon.

Gruß, Nick

P.S.: Mit der von dir als Wägbarkeit von Erkenntnis übersetzten Trägheit des Bewusstseins meinte ich im Grunde nur den Charakter ihrer Ausbreitung. Es erfordert analog zu Energie schon sehr großen Einsatz, das Bewusstsein einer großen Gruppe mit der Erkenntnis eines Einzelnen zu infiltrieren. Das dauert. So gesehen hat Bewusstsein Masse.

P.S.II: Vielleicht gibt das Quant auch nur einen Impuls weiter in den mit Quantenzuständen übervollen Hohlraum im Begrenzungsraum. Die Fortpflanzug des Impulses zum Zentrum und zurück ginge dann ähnlich vor sich wie das Fließen von Strom - blitzschnell.

Knut Hacker
29.09.11, 17:31
Hallo Knut Hacker,
leider hast du mir wenig Definierendes über den Geist genannt.

Hallo Nick,

nachdem du mich durch diese Bemerkung zu den vorstehenden ausführlichen Darlegungen veranlasst hast, möchte ich diese wie folgt zusammenfassen:

Der Begriff des Geistes ist ebenso wie sein Gegenstück, der Begriff der Materie, und zum Beispiel auch die Begriffe des Seins, Bewusstseins, Nichts, der Wahrheit, des Sinnes, des (hinreichenden) Grundes, der Willensfreiheit usw., ein philosophischer Leerbegriff.
Versucht man, solche Begriffe mit einem Bedeutungsinhalt zu füllen, gerät man zwangsläufig in Paradoxien.Es handelt sich um verbale Hybridisierungen von Trivialvorstellungen, um semantische Monster, um animistische Beschwörungsformeln, Versatzstücke aus der Sprache der Schamanen. Solche Letztbegriffe verschwimmen in ihrer Abstraktheit zur irrationalen Glaubensfetischen.

Es ist in der Menschheit noch keinem Denker gelungen,abstrakte Begriffe wie "Geist" zu definieren. Und es wird auch niemandem gelingen. Denn Definition bedeutet die Beschreibung mit abstrakten Begriffen. Wo jedoch ein Begriff berets der höchsten Abstraktionsebene angehört, wie der des Geistes, verbleiben keine noch abstrakteren Begriffe, mit denen er definiert werden könnte. Das ist im übrigen ohnehin das Problem der Philosophie. Sie diskutiert und beschreibt abstrakte Vorstellungen mit abstrakten Begriffen und stellt daher praktisch eine bloße Tautologie dar

Nick Rymer
29.09.11, 17:49
Versucht man, solche Begriffe mit einem Bedeutungsinhalt zu füllen, gerät man zwangsläufig in Paradoxien.Es handelt sich um verbale Hybridisierungen von Trivialvorstellungen, um semantische Monster, um animistische Beschwörungsformeln, Versatzstücke aus der Sprache der Schamanen. Solche Letztbegriffe verschwimmen in ihrer Abstraktheit zur irrationalen Glaubensfetischen.
Es mag ja sein, dass Derartiges nicht mehr zur Physik gehört, und vielleicht sogar dem sprachlichen Ausdruck gar nicht mehr zugänglich ist.

Auch danke ich dir für die abschließenden Worte.

Doch möchte ich noch einen Begriff einbringen, der sich wirklich jeder rational beschreibaren Theorie dazu entzieht: Die Intuition.
Die Intuition ist durchaus in der Lage die semantischen Monster zu erfassen. Zwar ohne Artkulation aber dennoch ist sie allerorten präsent. Du und ich, wir alle, sind ständig in Geist und Bewusstsein mit Intuition erfüllt, ihrer mächtig und, wer weiß, mancher ihr ausgeliefert. Sie ist der Motor der Dynamik von Gruppen genau so wie sie dem Erfinder seine Ideen spendet. So allgegenwärtig sie ist, sie ist ein Geheimnis, wie es auch die Quantenwelt ist.
Verwandtschaft?

Gruß, Nick

richy
29.09.11, 18:38
Hi Nick
Mancher spricht von wirklich räumlichen zusätzlichen Dimensionen, mancher rollt sie auf.
Manche ? Alle Stringtheorien gehen davon aus. Und diese raeumlichen Dimensionen muessen kompakifiziert, z.B. aufgerollt sein.
Der ganze Hype um die Mini Black Holes beim LHC basierte auf der Grundlage zusaetlicher raeumlicher Dimensionen. Der Sicherheitsbericht rechnet alle moeglichen Varianten dazu durch. Das nimmt man (zumindestens die Mathematiker die davon profitieren) recht ernst und in den Forschungsbereich fliessen Millarden rein. Ganz im Gegensatz zu Ansaetzen mit imaginaeren Dimensionen, die man nicht aufrollen muss und im Grunde sehr einfach verstaendlich sind. Warum dieser Ansatz kaum unterstuetzt wird weiss ich nicht.

Beidem gemeinsam ist zwingend, dass jedem Objekt, hier vorzugsweise ein Quant, eine Verbindung zum Zentrum des Begrenzungsraumes zur Verfügung steht. Zumindest in Gestalt einer Distanz...... Also da darfst du mich nicht fragen. Der Kabeljau ist so ein komplexer Fisch, an den traue ich mich nicht ran. Ich hab ehrlich gesagt keinerlei Vorstellung wie kompaktifizierte Dimensionen zu einer Globalitaet fuehren koennten. Bei ausgebreiteten zeitlichen Dimensionen ist das recht einfach. Nehmen wir an unsere Realitaet ist durch x5=0 (i*m) ausgezeichnet und du koenntest diesen Wert veraendern, dann hast du fuer x5=0.1 (i*m) eine andere Welt.
Beispiel :
Triffst du in x5=0 die Entscheidung bei rot uber die Ampel zu fahren, dann existiert nun eine Welt x5=0 bei der du bei rot ueber die Ampel gefahren bist.
Die Welt in der du angehalten hast liegt zum Beispiel bei x5=0.1, gehoert also nicht mehr zu unserer Realitaet. Dort kriegst du keine Post von der Stadt. Das ist wie bei der Zeit, in der immer ein Gegenwartspunkt t=0 existiert. Allerdings ist dieser verschmiert und so ist wohl unsere Moeglichkeitsrealitaet nicht ganz scharf.
Der ganze Zirkus loest sich somit in der Wahl einer oder mehrerer Zahlen auf.
Bei Everett ist das konkret im Detail natuerlich etwas komplizierter, da dieser keine einzelne ausgebreitete Diension annimmt, sondern einen Konfigurationsraum. Eine Interpretation soll im Gegensatz zu einer Theorie moeglichst keine Erweiterungen enthalten. Daher nimmt Everett keine einzelne zusaetzliche Dimension an, sondern fuer jeden Zustand eines Mehrteilchensystems eine eigene Dimension. Die minimalste Version. Damit bleibt die SGL so wie sie ist. Bei einem einzelnen Teilchen sogar 3+1 dimensional. Man muss auch keinen Wellenkollaps einfuehren oder Umformungen wie bei der BM. Man koennte alle moeglichen Zustaende zum Verstaendnis aber auf eine ausgebreitete einzige zeitartige Koordinate abbilden. Dann wird das noch anschaulicher.
Viele Druiden, Schamanen, Esoteriker und Heipraktiker klatschen mir Beifall, verifizieren sich doch so all ihre Vorstellungen.
Nee, die klatschen bei realistischen Modellen keinesfalls Beifall. Denn diese machen sie zunaechst mal ueberfluessig. Es haengt vom Wohlwollen des Physikers dann ab, ob er wie Wheeler oder Penrose hier Eingestaendnisse zulaesst. Das ist dann sicherlich motiviert durch das Erleben der eigenen Persoenlichkeit oder Aspekte der Nichtbeschreibbarkeit die Knut anhand einiger Beispiele beschrieben hat. Ein Realismus benoetigt weder Esoteriker noch Geister noch Druiden. Realistische Deutungen wie VWI oder BM verwenden ausschlieslich reine Physik. Daher auch der Name : Realismus. Philosophen oder die Kirche kann man zusaetzlich in Betracht ziehen. Muss man aber nicht.
Gruesse

Knut Hacker
29.09.11, 18:38
Du und ich, wir alle, sind ständig in Geist und Bewusstsein mit Intuition erfüllt, ihrer mächtig und, wer weiß, mancher ihr ausgeliefert. Sie ist der Motor der Dynamik von Gruppen genau so wie sie dem Erfinder seine Ideen spendet. So allgegenwärtig sie ist, sie ist ein Geheimnis, wie es auch die Quantenwelt ist.


Hallo Nick,

auch Albert EinsteinHatte diese zentrale Bedeutung der Intuition hervorgehoben.In Bezug auf die Wissenschaften schrieb er:
" Sämtliche großen Wissenschaftstaten (liegen) in der intuitiven Erkenntnis...."

(Alexander Moszkowski, Einstein-Einblicke in seine Gedankenwelt )

Intuition heißt zwar wörtlich "Eingebung". Aber das Problem der Selbstbezüglichkeit des Bewusstseins ist dadurch nicht gelöst. Denn "eingegeben" kann etwas von außerhalb des Bewusstseins sein, aber auch vom Bewusstsein selbst. Und diese Unterscheidung trifft ja nur unser Bewusstsein, so dass eine Entscheidung zwischen den beiden Möglichkeiten ebenfalls nur im Bewusstsein getroffen werden kann.

Knut Hacker
29.09.11, 19:27
Das ist wie bei der Zeit, in der immer ein Gegenwartspunkt t=0 existiert. Allerdings ist dieser verschmiert und so ist wohl unsere Moeglichkeitsrealitaet nicht ganz scharf.
Das ist bei jedem Kontinuum der Fall, insbesondere auch beim Raum (wie ja schon die Teilungsparadoxien des Altertums und der Neuzeit - Thompsonsches Lampen- Paradoxon - zeigen). Mathematisch behilft man sich im Quantenbereich durch die Planck-Zeit von 10 hoch -43 s und die Planck-Länge von 10 hoch -35 m . Zumindest die Zeit lässt sich jedoch völlig eliminieren, wie die Wheeler-De-Witt Gleichung zeigt.
Die ART krankt,weil sie sich selbst die Raumzeit als dynamisches Kontinuum schafft, an der Inkonsistenz unendlich hoher Materiekonzentrationen als Anfangsbedingung des Urknalls oder im Zentrum schwarzer Löcher. Könnte nicht die Quantentheorie im Quantenbereich dieser Singularitäten zur Hilfe kommen, indem sie diskrete statt kontinuierliche Abstände erlauben würde? Sie legt als Inertialsystem die klassische Newtonsche absolute Zeit und den absoluten Raum zugrunde, weil die relativistischen Effekte der Beschleunigung und der Gravitation vernachlässigbar sind.Aber beruht nicht gerade auch darauf das Problem der Nichtlokalität?

amc
29.09.11, 21:25
... wenn man sich stets die eigentliche Aufgabenstellung vor Augen fuehrt. Und das ist die Nichtlokalitaet. Wenn ich die Nichtlokalitaet mit abstraktem, unwaegbarem erklaere, dann habe ich nunmal den Geist in die Welt gesetzt. Und den krieg ich durch drumherumreden nicht mehr los.

Hallo Richy,

verstehe ich deine Aussagen in etwa richtig, wenn ich annehme, dass sie in diese Richtung zielen: Um bestimmte quantenmechanische Sachverhalte mathematisch korrekt beschreiben zu können, kann man grundsätzlich zwei Lösungsansätze wählen - zum einen, man nimmt an, dass reale physikalische Vorgänge beschrieben werden, oder man wählt abstrakte Größen. Weil man diese abstrakten Größen zur korrekten mathematischen Beschreibung benötigt, kann man dann hinterher aber nicht einfach so tun als hätte dies gar keine Bedeutung?

Verstehe ich ungefähr worum es (dir) bei der Problematik geht?


Freundlichst,
AMC

richy
29.09.11, 21:29
Hi Knut

Dieses "Verschiert sein" unseres Gegenwarts/Realitaetspunktes ist zunaechstmal eine physiologische Angelegenheit. Uebrigends eine durchaus interessante. Denn man kann darueber verschiedene Realitaetsformen rein numerisch festlegen. Je nachdem welche Vergangenheitswerte man hinzunimmt. Und interessanterweise verwendet das Gehirn fuer das Gedaechtnis je nach Laenge des Zeitintervalls verschiedene Methoden. Die Menscheit selbst dann selbst fuer laengere Zeitraeume wiederm andere Speichermethoden wie Papier und fuer sehr lange Zeitintervalle Stein. (Der Koelner Dom wird den Eiffelturm und jede CD,DVD locker ueberdauern)

Jedes enfache physikalische dynamische System hat natuerlich ebenfalls ein Gedaechtnis. Ganz ohne Rucksack. Der Istzustand ergibt sich aus einer zeitlichen Integration. Und diese erstreckt sich ueber die vergangenen Werte. Man spricht tatsaechlich im Ingeniuersbereich von Systemen mit Gedaechtnis.
Von daher meine ich, dass diese physiologische Unschaerfe, besser Verschmiertheit einfach auf dem Prinzip dynamischer Systeme beruht. Ob diese quantisiert sind oder nicht halte ich fuer weniger erheblich. Dann waere das Integral enfach eine Summe.

Die ART krankt,weil sie sich selbst die Raumzeit als dynamisches Kontinuum schafft, an der Inkonsistenz unendlich hoher Materiekonzentrationen als Anfangsbedingung des Urknalls oder im Zentrum schwarzer Löcher.
Ja, das ergibt gewisse Grenzen. Das ist jedoch keine typisch "Krankheit" der ART, sondern es wird in der Physik nun mal die Infinitesimalrechnung verwendet. Und diese benoetigt ein Kontinuum.
Und angesichts der Fortschritte die die Physik plus Infinitesimalrechnung uns gebracht hat, sollte man hier besser nicht den Begriff "krankt" verwenden.
Weiterhin:
Mit der Digitaltechnik, digitalen Nachrichtenuebertragung hat die mathematische Behandlung diskreter Systeme schon lange den selben Stellenwert erreicht. Und letztendlich werden komplexe Aufgaben der ART am Rechner simuliert. Dann sind die Gleichungen automatisch diskretisiert.

Die Chaostheorie zeigt jedoch, dass eine diskretisierte DGL im nichtlinearen Fall ein voellig anders Verhalten aufweist als die Ursprungsgleichung. Und daher mueste man die ART selbst unter diskretisierten physikalischen Aspekten betrachten. Auch das kannst du haben. Das Modell von B.Heim ist eine vollstaendig diskretisierte Form der ART. Mit all den Eigentuemlichkeiten und formalen Maengeln die bekannt sein duerften. Mich hat dieses Modell uebrigends anfangs nur wegen den diskreten Rechenverfahren interessiert. (Metronenrechnug=nichtlineare finite Volumen) Aber darueber findet sich bei Heim leider nichts konkretes.

Da dir die Chaostheorie ein Begriff ist. Ich habe in der Zwischenzeit eine erstaunliche Entdeckung gemacht. Loesungen des quadratischen Prototypens der nichtlinearen Differenzengleichungen (logistische Abbildung) hat bereits im 19. Jahrhundert ein Mathematikprofessor aus Karsruhe hergeleitet. Mittels dem nach ihm benannten Schreoder Funktionaloperator.
http://www.quanten.de/forum/showthread.php5?t=1924
Um nichtlineare diskrete Systeme analytisch betrachten zu koennen benoetigt man solche Loesungsverfahren. Und davon gibt es so gut wie nichts. Und wenn es etwas gibt, dann duempeln diese wie man sieht, ausgenommen fuer einige Spezalisten, im Dunkeln herum. Irgendein Institut fuer Fischerei in Miami kennt z.B. Schroeders Loesungsverfahren. Immerhin :-)

Die kontinuierlichen Solitonengleichungen der QM sind gluecklicherweise loesbar. Mich wuerde momentan interessiere wie es hier bei den diskreten Versionen aussehen wuerde. Das sind aber partielle Gleichungen 2 ter Ordnung. Alleine der Prototyp wuerde mich dennoch interessieren.

Aber beruht nicht gerade auch darauf das Problem der Nichtlokalität? So ein grosses Problem ist die Nichtlokalitaet ja gar nicht. Blos mag niemand so recht die realistischen Loesungen.
Von Penrose gibt es uebrigends ein Zitat dass dich interessieren koennte. Und zur "Existenz der Existenz" und "Nichtexistenz der Nichtexistenz" wollte ich noch etwas schreiben.
Gruesse

richy
29.09.11, 23:20
Hi AMC
Um bestimmte quantenmechanische Sachverhalte mathematisch korrekt beschreiben zu können, kann man grundsätzlich zwei Lösungsansätze wählen -

Hmmm, das faengt nicht so gut an. Ueber die mathematische Beschreibung gibt es ja keinerlei Zweifel. Das ist die SGL die oben steht. Aber ok weiter. :-)

zum einen, man nimmt an, dass reale physikalische Vorgänge beschrieben werden ...
Ja genau. Das ist der normale Fall in der Physik. Und wie erwaehnt, kann man nochmals zwischen physikalischen Eigenschaften und einem physikalischen Objekt, Teilchen unterscheiden. Wobei das nicht der entscheidende Punkt ist.
Dennoch gehe ich mal diesen Aspekt etwas genauer durch :

"Der Klatschmohn ist rot" Dieses "rot" ist letzendlich kein "Ding" sondern eine Eigenschaft der Molekuele des Klatschmohns. Eine Geometrie. Ok ich kann sagen "Die Wellenlaenge des reflektierten Lichts liegt im roten Bereich" Das ist aber lediglich eine Konsequenz der Geometrie der Molekuele in diesem Klatschmohn. Ok diese Geometrie selbst wiegt nichts, ist abstrakt (mal ohne Feldbetrachtung) aber da sind physikalische Teichen, die diese Geometrie erst ermoeglichen. Und daher ist dieses "rot" stets mit etwas physikalischem verbunden. Betrachten wir jetzt die Einzelteile der Molekuele selbst. Elemente, Atome. Welche Farbe haben diese ? Undefined. Zugegeben, je weiter wir uns in die Mikrowelt begeben, umso mehr verlieren Eigenschaften die wir kennen ihren Sinn. Wir koennen aber die Masse des Atoms bestimmen, die Anzahl seiner Protonen,Neutronen, die Elementarladung eines Elektrons. Wir naehern uns elementaren immer schwerer Verstaendlichen Groessen, aber an einem Prinzip aenderte sich bisher nichts. Wenn wir etwas physikalisches beschreiben, dann muss es etwas geben, dass beschrieben wird. Etwas physikalisches.
Jetzt gehen wir das Spiel mal weiter durch, ohne uns um irgendeinen Wellenkollaps zu kuemmern. Und vielleicht liest auch Knut hier mit. Es ist keine exotische Vorstellung anzunehmen, dass Materie nicht in den Raum geschuettet ist, sondern eine (dynamische ) Geometrie desselben darstellt. Somit waere tatsaechlich fast alles Eigenschaft, naemlich Geometrie, bis auf die Raumzeit selbst. Das fuehrt auf die Frage, ob die Raumzeit selbst als physikalische Entitaet betrachtet werden kann. Und ich meine hier gibt uns die ART ohne die bisherigen Vorbetrachtungen, auf voellig anderem Weg eine Antwort. Ja, die Raumzeit ist eine physikalische Entitaet.

Gehen wir den bisherigen Weg (Makro nach Mikro) aber noch einen kleinen Schritt weiter und folgen Knuts Gedankengaengen. Dabei stossen wir ebenfalls wie bei der ART auf die Raumzeit hoechstpersoenlich. Tja was ist das nun ? Ein Feinstoff ? Im Grunde ist das voellig egal, da wir dazu soundso keine Aussage mehr treffen koennen ausser derjenigen ob diese Raumzeit denn nun als physikalisch oder abstrakt einzustufen ist. Wobei die ART uns hier einen Anhaltspunt gegeben hat. Endstation ?
Nein. Wir koennen noch einen Schritt weiter gehen. Und dieser ist am besten verstaendlich wenn man zunaechst annimmt, das die Raumzeit physikalisch quantisiert waere. Wie laesst sich die Raumzeit quantisieren ? Aus welchem Stoff sind diese Quantsierungslinien, Raender ? Das ist scheinbar eine so kniffelige Frage wie das Wesen der Raumzeit selbst. Die Loesung ist aber verblueffed einfach. Die Quantisierung ergibt sich dadurch, dass die Raumzeit die Eigenschaft tragen kann zu existieren und nicht zu existieren. (Und wenn man genau nachdenkt, dann kann es nur umgekehrt sein wie man es sich eigentlich vorstellt. Da schwimmen nicht Existenzinseln im NICHTS, sondern es existiert ein zusammenhaengender Rand) Ganz einfach wie bei diesem Pilz hier.
http://www.proplanta.de/Agrar-Nachrichten/Umwelt/image/1284829311_1024.jpg
Dieser Gitteling stellt die Raumquanten der Loop Quantengravitation dar. Wuerde der Pilz auf die Idee kommen auch mal die duale Struktur auszuprobieren. (Rot=Pilz und da ist Nichts vertauschen) dann wuerde der arme Gitterling wohl auseinanderfallen. In der LQG sind uebrigends oft beide Strukturen dagestellt, was nicht unbedingt zum besseren Verstaendnis beitraegt.
Wie dem auch sei. PHYSIKALISCHE Existenz und PHYSIKALISCHE Nichtexistenz koennten die fundamentalen Elemente der Physik sein. Wer eine kontinuierliche Raumzeitstruktur bevorzugt, der bildet einfach limit (Diskretisierung=0)
Puh am Ziel. Nee noch nicht ganz. Denn Knut hat schon vor einem halben Jahr ein logisches Argument angefuehrt, dass unsere bisherige (auch aus anderen Gruenden noch nicht vollstaendige) Betrachtungsweise zerdeppern koennte.

Welche der folgenden Aussagen ist eine Antinomie im Sinne Russels ?
a) die Existenz einer Existenz
b) die Nichtexistenz einer Existenz
c) die Existenz einer Nichtexistenz
d) die Nichtexistenz einer Nichtexistenz

Wenn man diese ganzen Faelle durchspielt ergeben sich natuerlich Wort und Logikkonstrukte, die man z.B im AC Forum besser nicht verwenden sollte (Nur ein Tipp and dich Knut :-)
Aber tatsaechlich eine Antinomie, eine Art Wiederspruch. Demnach waere die ART , die QM, die komplette physikalische Welt auf einem Widerspruch "besser einer unendlichen Klassifizierung" aufgebaut. Das waere nicht befriedigend.
Und tatsaechlich laesst sich dieser Widerspruch durch eine nichttriviale Mengenalgebra aufloesen und darauf prima eine rein realistische Physikalische Welt errichten.
@Knut
Mich wundert, dass du dies anscheinend nicht zur Kenntnis genommen hast.

Und mit dieser Mengenalgebra endet dieser Ausflug an die mikroskopische Grenze unseres Universums.

Sodele, tauchen wir also wieder auf.... Plopp

richy
30.09.11, 00:38
Das war eine moegliche, sehr vereinfachte, rein physikalische, realistische Betrachtungsweise. Wie inzwischen allgemein bekannt sein sollte fuehrt diese Betrachtungsweise nach dem heutigen Stand der Dinge zu einer Interpretation der Nichtlokalitaet der Quantenmechanik die wie folgt lautet :
Das Universum ist nichtlokal und die Raumzeit muss daher um mindestens eine Dimension erweitert werden.
Wieviele Paralleluniversen passen eigentlich auf eine einzige zusaetzliche Dimension ? So viele Universen wie Zahen auf einer Zahlengerade oder auf einem Intervall derselben Platz haben.
In das Intervall o bis 1 passen zum Beispiel unendlich viele Zahlen und damit auch unendlich viele Universen. Das ist sehr viel :-)
Bei Everett existieren 10^100 ( das waere eine ungeheuer laecherlich kleine Zahl ) oder vielleicht auch 10^100^100 Universen. ( Das ist etwas mehr aber aus mathematischr Sichtweise gegenueber limit(1/x, x->0) immer noch eine recht "kleine" Zahl)
Annahme : Die Zeit sei nicht quantisiert. Wir betrachten den Zeitverlauf als eine Folge von Serielluniversen. Wieviele Serielluniversen koennte man im Zeitintervall einer Sekunde erzeugen ?
Betrachten wir die Plankzeit als Quantisierungsgroesse. Wieviele Serielluniversen werden im Verlaufe eines Jahres erzeugt ?
Ok, wenn man will kann man dies dennoch als ein Problem betrachten.

oder man wählt abstrakte Größen. Weil man diese abstrakten Größen zur korrekten mathematischen Beschreibung benötigt,
Das verstehe ich jetzt nicht wie du das meinst. Klar eine Beschreibung verwendet immer abstrakte Groessen.
kann man dann hinterher aber nicht einfach so tun als hätte dies gar keine Bedeutung?
Da kommen wir der Sache vielleicht ein Stueck naher was die SGL gemaess Kopenhagen beschreibt. Ganz grob und dennoch recht zutreffend ausgedrueckt : Sie beschreibt NICHTS !
Sie ist selbst. Dem wuerde Prof Weizsaecker erwidern :

„Die Kopenhagener Deutung wird oft, sowohl von einigen ihrer Anhänger wie von einigen ihrer Gegner, dahingehend missdeutet, als behaupte sie, was nicht beobachtet werden kann, das existiere nicht. Diese Darstellung ist logisch ungenau. Die Kopenhagener Auffassung verwendet nur die schwächere Aussage: "Was beobachtet worden ist, existiert gewiss; bezüglich dessen, was nicht beobachtet worden ist, haben wir jedoch die Freiheit, Annahmen über dessen Existenz oder Nichtexistenz einzuführen." Von dieser Freiheit macht sie dann denjenigen Gebrauch, der nötig ist, um Paradoxien zu vermeiden.
– Carl Friedrich von Weizsäcker: Die Einheit der Natur, Hanser 1971, S. 226“
Meine Sperre war durch eine genauere Analyse dieser (recht alten) Aussagen begruendet. Du musst das also selbst analysieren.

Abgesehen von diesem Zitat halte ich Weizsaecker fuer einen hervorragenden Philosophen :
Ohne Wertung. Nur mal um zu zeigen wie die Dinge eigentlich liegen / lagen :
Im Jahr 1969 reiste Weizsäcker durch Indien und hatte im Ashram von Sri Ramana Maharshi in Tiruvannamalai ein spirituelles Erlebnis, in dem „alle Fragen beantwortet waren“ und dessen Substanz nach eigenen Worten immer bei ihm war oder
http://www.zit.at/personen/weizsaecker.html

Die Reise die uns im letzen Beitrag bis an die Fundamente einer physikalischen Welt gebracht haben endet bei Kopenhagen sehr viel frueher. Naemlich beim Wellenkollaps oder moderner dem Zustand vor der Dekohaerenz. Dieses Weltbild kann dir aber Knut sicherlich besser vermitteln.
Und wenn wir nun beachten, dass bereits C60, C70 Fullerene interferieren, dann rueckt die Vorstellung einer rein abstrakt aufgebauten Welt in eine fuer mich bereits bedrueckende Naehe, die ich nicht mit meinem Weltbild vereinbaren kann. Ich kann die C70 Versuche nicht rein informatorisch wie Zeilinger interpretieren Ja, auch informatorisch, aber nicht rein informatorisch. Meine Vorstellung entspricht im Grunde der von Wheeler, Heim oder Penrose. Das muss aber jeder fuer sich selbst entscheiden.

Gruesse

Nick Rymer
30.09.11, 07:41
„Die Kopenhagener Deutung wird oft, sowohl von einigen ihrer Anhänger wie von einigen ihrer Gegner, dahingehend missdeutet, als behaupte sie, was nicht beobachtet werden kann, das existiere nicht. Diese Darstellung ist logisch ungenau. Die Kopenhagener Auffassung verwendet nur die schwächere Aussage: "Was beobachtet worden ist, existiert gewiss; bezüglich dessen, was nicht beobachtet worden ist, haben wir jedoch die Freiheit, Annahmen über dessen Existenz oder Nichtexistenz einzuführen." Von dieser Freiheit macht sie dann denjenigen Gebrauch, der nötig ist, um Paradoxien zu vermeiden.
– Carl Friedrich von Weizsäcker: Die Einheit der Natur, Hanser 1971, S. 226“
Nur zur Erhärtung: Einst erschuf sich das Weltall, der Kosmos, mit seinem Geist, den Naturgesetzen. Wie konnte es diese gegeben haben, wo wir sie doch noch gar nicht kannten, ja, sie heute sogar nur ansatzweise kennen?

Gruß, Nick

Marco Polo
30.09.11, 08:18
Guten Morgen richy,

Meine Sperre war durch eine genauere Analyse dieser (recht alten) Aussagen begruendet.

vielleicht nicht ganz, richy. :rolleyes:

Dennoch kann ich deine vergangene Sperre bis heute nicht so recht nachvollziehen.

Um so mehr freue ich mich, dass du hier wieder tatkräftig mitmischst. :):):)

Aber lass uns die möglicherweise zweifelhaften Gründe hier bitte nicht thematisieren. Hauptsache du bist wieder da. :)

Das ist auch der eigentliche Grund für meinen Beitrag: Lass die Sache bitte ruhen und reite nicht darauf herum.

Viele Grüsse, Marco Polo

amc
30.09.11, 13:03
Hi Richy,

oder man wählt abstrakte Größen. Weil man diese abstrakten Größen zur korrekten mathematischen Beschreibung benötigt,Das verstehe ich jetzt nicht wie du das meinst. Klar eine Beschreibung verwendet immer abstrakte Groessen.

Ich bezog mich damit auf diese Aussage:

Wenn ich die Nichtlokalitaet mit abstraktem, unwaegbarem erklaere, dann habe ich nunmal den Geist in die Welt gesetzt. Und den krieg ich durch drumherumreden nicht mehr los.

Mir geht es im wesentlichen darum zu verstehen, wie du zu dieser Aussage kommst:

Nicht vergessen : Jede Erklaerung der QM muss eine Erklaerung der Nichtlokalitaet enthalten.

Vielleicht dann nochmal ein leicht abgewandelter Versuch:

Weil man die SGL benötigt, um bestimmte Vorhersagen über quantenmechanische Systeme zu treffen, muss man sich auch mit der Frage auseinandersetzen, was beschreibt die SGL eigentlich? Hier kann man dann eben einen realistischen Ansatz wählen, oder man sagt, es handelt sich nur um einen mathematischen Formalismus, der keinen realen Vorgang beschreibt. Was aber dann zwangsläufig bedeutet, die Notwendigkeit der SGL mit etwas abstraktem erklären zu müssen ...

Also meine Frage ist: Warum kann man nicht einfach sagen, wir können Nichtlokalität noch nicht erklären und gut ist? Woraus resultiert die Notwendigkeit zur Erklärung der Nichtlokalität? Bin ich der Antwort auf diese Frage mit oben stehenden Erklärungsversuch ein wenig näher gekommen?

Dann hoffe ich mal, dass du verstehst was ich will, bzw. dass es dir nicht zu müßig wird zu versuchen zu verstehen was ich will. Vielleicht hast du es mir ja auch schon längst erklärt und ich checks noch nicht ...


Freundlichst,
AMC

Harti
30.09.11, 13:32
Hallo Richy,

Du schreibst, jede Erklärung der QM muss eine Erklärung der Nichtlokalität enthalten.

Als Nichtphilosoph und Nichtphysiker, der aufgrund von begrifflichen Unklarheiten nur wenig versteht, versuche ich es trotzdem. Dazu wähle ich als Beispiel den Zahlenstrahl.

Dieser ist zunächst ein mathematisches Gedankenkonstrukt. Er hat einen Mittelpunkt, die Null, und zwei Richtungen, nach rechts plus und nach links minus. Er wird in beiden Richtungen als unendlich vorgestellt. Diese Unendlichkeitsvorstellung belegt, dass es sich nur um ein Gedankenkonstrukt handelt; denn in der realen Welt können wir zwar Unbegreztheit aber nicht Unendlichkeit feststellen.
Wenn ich diesen Zahlenstrahl auf die Wirklichkeit übertrage, brauche ich ein real existierendes Objekt; dazu nehme ich als Beispiel einen Holzstab. Um mich auf diesem Holzstab zu orientieren, lege ich einen Punkt als Nullpunkt fest und bezeichne den Bereich rechts vom Nullpunkt als plus uns links vom Nullpunkt als minus. (Es handelt sich bei dieser theoretischen Bildung eines Gegensatzpaares, plus- minus, folglich um eine Maßnahme, die der Orientierung (Verständigung) auf dem System Holzstab dient.) Wenn ich diesen Holzstab aus einer um 180° versetzen Position (Gegenposition) betrachte, kehren sich die Gegensätze um. Was vorher rechts und plus war, wird links und minus und umgekehrt. Dies belegt, dass zwischen plus und minus in der Realität eigentlich kein Gegensatz besteht. Es ist lediglich eine Frage der Perspektive, ob ich etwas als plus und minus oder umgekehrt bezeichne.
Nun betrachte ich zwei konkrete Punkte auf dem Holzstab, z.B. +5 und -5. Sie sind räumlich getrennt (nichtlokal i.S. der QM). Wenn ich mein Orientierungssystem, die Unterscheidung zwischen plus und minus, was ja nur ein Denkmodell war, aufgebe, habe ich nur noch zwei räumlich getrennte Punkte in einem einheitlichen System "Holzstab".

Ergebnis: Man könnte die Nichtlokalität erklären, wenn man annimmt, dass die beiden betrachteten Objekte zu einem System gehören und den vorgestellten Gegensatz zwischen den Objekten als Gedankenkonstrukt, das lediglich der Orientierung dient, aufgibt.

MfG
Harti

Knut Hacker
30.09.11, 17:24
Hallo ihr!

Ich bedanke mich für die Fülle interessanter Gedanken und Denkanstöße. Leider ist es mir zeitlich nicht möglich, auf alle Gesichtspunkte einzugehen. Daher nach und nach eine sehr subjektive Auswahl:



Eine Frage an dich : Wird in der Philosophie nur die Kopenhagener Deutung in Betracht gezogen oder auch die realistischen Varianten ? Und was ich nicht so ganz verstehe ist z.B. die Reaktion im AC Forum.

Die Kopenhagener Deutung ist ja immer noch herrschend. Alles andere erscheint mir als der naive Versuch, zum überholten begrifflichen Denken der Newtonschen Physik zurückzukehren. Warum lassen diese Physiker die Relativitätstheorien unangetastet, obwohl es sich dabei zwar um klassische Physik handelt, aber doch auch unsere traditionellen Vorstellungen versagen.

Die Philosophie nimmt erstaunlich wenig Kenntnis von den modernen Naturwissenschaften, obwohl Heisenberg diese als die Philosophie des Jahrhunderts bezeichnet hatte. Es gibt Ausnahmen wie Karl Popper usw.

Ich habe nicht mitbekommen, was im AC passiert ist. Das Forum nennt sich doch kritisch. Dazu gehört aber auch die Hinterfragung tradierter Vorstellungen, die kein Geringerer als Albert Einstein als Vorurteile bezeichnet hat.



Ich meine eine Einteilung deiner vorgestellten Paradoxa in echte Paradoxa und scheinbare Paradoxa waere hilfreich.



Es gibt solche Einteilungen, ich habe darauf schon in meinem früheren Thread über Paradoxien Literaturhinweise gegeben. Mathematische Lösungen sind allerdings keine Auflösungen, sondern entsprechend der Natur der Mathematik lediglich Beschreibungen.
„ Erklärung“ heißt ja Erfassung mit unseren begrifflichen Vorstellungen.Die Infinitesimalrechnung beispielsweise leistet dies aber gerade nicht bei den Teilungsparadoxien.Sie beschreibt nur das Dilemma der unendlichen Annäherung.

„Parádoxon“ heißt ja „neben den Ansichten“. Man sollte alles darunter verstehen, was mit dem so genannten gesunden Menschenverstand , den nicht nur Einstein hart gegeißelt hat, nicht erfassbar ist.

Knut Hacker
30.09.11, 18:08
Hallo fossilium!
Sie ist auch kein Zwischengebilde zwischen Sein und Nichtsein - was soll den das sein, wenn es Sein und Nichtsein schon unverstandene Begriffe sind. Und eine Hinneigung (Potentialität) ist Materie, sofern in materiellen Sinn verstanden, nur bei sehr hohen Energien (Potentialitäten), wobei man auch hier wieder mit den zugehörigen Definitionen weit ausholen muss.

Auch ich bin vorsichtig bei der Übertragung physikalischer Ergebnisse auf philosophische Fragen .
Die Formulierungen „Potentialität“, „ Information“und „Materie besteht nicht aus Materie“ usw. stammen von Quantenphysikern wie Heisenberg, von Weizsäcker, Zeilinger und Dürr auf der Basis der Kopenhagener Deutung.Das Dilemma ist doch, dass sich die Ergebnisse der modernen Naturwissenschaften nicht begrifflich ausdrücken lassen, weil dies zu Paradoxien führt. Deshalb bedient man sich ja der Sprache der Mathematik. Positivistische Naturwissenschaftler setzen dies allerdings mit Erklärung gleich. Obwohl es lediglich eine Beschreibung ist.


Hier ist insbesondere die Frage interessant: wenn es in der Mathematik, die die Naturgesetze formal beschreibt, unentscheidbare Ausagen gibt, ob sich das nicht bei der mathematischen Beschreibungen der Naturgesetze auswirken muss. Aber wie ?Ist zum Beispiel die Unmöglichkeit einer logische Beschreibung nicht-kausaler Ereignisfolgen auf den Gödelschen Satz rückführbar ? Oder allgemein gefragt: Wie siehst Du die Auswirkungen des Gödelschen Satzes in seinen Folgen für die formale mathematische Beschreibung der Physik ?


Soweit habe ich das Forum noch nicht nachgelesen, was sich auch heute wahrscheinlich nicht mehr schaffe.

Was heißt logische Beschreibung? Es gibt ja verschiedene Logiken, insbesondere die mehrwetige Quantenlogik, die Komplementär ist, also letztlich im Sinne der zweiwertigen aristotelischen Logik „unentscheidbare Aussagen“ in dem Sinn trifft, dass es sich bei der Unentscheidbarkeit um ein Wesensmerkmal der Natur handelt, das sich im Bereich der klassischen Physik jedoch deshalb nicht auswirkt, weil dort die Unbestimmtheitseffekte vernachlässigbar sind, es wird ja nur eine begrenzte Genauigkeit verlangt.
Die Gödelschen Unvollständigkeitssätze berühren die Physik nach ihrem Selbstverständnis nicht.Die Physik ist ja Erfahrungswissenschaft, das heißt ihr geht es nur um das Erfahrbare, nicht um eine Hinterfragung desselben.Wenn wir beispielsweise im Doppelspaltsexperiment den Dualismus Welle-Korpuskel „erfahren“, dann versuchen wir ihn nicht von einer Metaebene aus zu verifizieren oder falsifizieren, sondern beschreiben die Erfahrung vorbehaltlich späterer weiterer Erkenntnisse mit der lediglich probabilistischen Schrödinger´schon Wellengleichung. Begriffe wie Superposition oder Nichtlokalität dienen bestenfalls dem Vergleich mit der klassischen Physik und verleiten schlechtestenfalles zu philosophischen Aussagen, die das Selbstverständnis der Physik missachten. Die klassischen Quantenphysiker waren jedoch alle philosophisch vorgebildet (Besuch des humanistischen Gymnasiums), und haben daher philosophische Konsequenzen aus ihren Erkenntnissen gezogen.Sie haben aber immer strikt zwischen solchen und den experimentellen Befunden unterschieden.
Die Gödelschen Gesetze besagen ja nur, dass sich kein System aus sich selber heraus begründen kann, also die Mathematik nicht mithilfe der Mathematik und die Physik nicht mithilfe der Physik, auch die Philosophie nicht mithilfe der Philosophie und die Logik nicht mithilfe der Logik.So enanntes Münchhausensyndrom: Niemand kann sich am eigenen Schopf aus dem Sumpf ziehen.

Knut Hacker
30.09.11, 18:19
Das Bewusstsein eines lebenden Organismus spielt üblicherweise in modernen Interpretationen der Quantenphysik keine zentrale Rolle.

Es spielt natürlich überhaupt keine Rolle. Ich bitte um Entschuldigung für die populärwissenschaftliche Ausdrucksweise. Ich meinte Bewusstwerdung als Zielergebnis der Messung. Ein physikalischer Vorgang ist nur dann und insoweit von Bedeutung für die Physik als Erfahrungswissenschaft,als er dem Bewusstsein durch Messung zugänglich gemacht wird. Niels Bohr:" Es ist falsch anzunehmen, die Aufgabe der Physik bestehe darin, herauszufinden, wie die Natur beschaffen ist. Die Physik beschäftigt sich vielmehr damit, was wir über die Natur aussagen können."

Knut Hacker
30.09.11, 18:34
.
Der Philosoph oder die Kirche wird wahrscheinlich nie einen Realismus ernshaft in Betracht ziehen,
Was ist denn Realität? Doch dasjenige, was uns unser Bewusstsein sagt. Wir können schließlich nicht aus diesem heraustreten.Also befinden wir uns in einem System, so dass die Gödelschen Gesetze gelten, dass sich kein System aus sich heraus beweisen kann. Dafür fehlt die Metaebene. Das beginnt doch schon mit der Kindesfrage, was "rot" ist. Dieser Sinneseindruck hat mit dem Wellenspektrum des Lichtes nicht das Geringste zu tun.Das gilt für alle sogenannten Qualia. Die Kognitionswissenschaften sagen uns, dass unser sinnliches Wahrnehmen ebenso wie unser Denken, Fühlen und Werten strukturdeterminiert ist, das heißt durch die Beschaffenheit unseres Gehirns entsteht. Auch der Begriff der Realität ist daher ein bloßes geistiges Konstrukt.Und auch diese Aussage.Die sogenannten Realisten verabsolutieren den Verstand (philosophisch zu unterscheiden von der Vernunft), obwohl alles Denken letztlich selbstbezüglich ist, also zum unstrittigen Münchhausen Trilemma führt: entweder zu Denkzirkeln oder zu unendlichen Regressen oder zu willkürlichen Abbrüchen.

Knut Hacker
30.09.11, 18:52
Wenn man diese ganzen Faelle durchspielt ergeben sich natuerlich Wort und Logikkonstrukte, die man z.B im AC Forum besser nicht verwenden sollte (Nur ein Tipp and dich Knut :-)
Aber tatsaechlich eine Antinomie, eine Art Wiederspruch. Demnach waere die ART , die QM, die komplette physikalische Welt auf einem Widerspruch "besser einer unendlichen Klassifizierung" aufgebaut. Das waere nicht befriedigend.
Und tatsaechlich laesst sich dieser Widerspruch durch eine nichttriviale Mengenalgebra aufloesen und darauf prima eine rein realistische Physikalische Welt errichten.
@Knut
Mich wundert, dass du dies anscheinend nicht zur Kenntnis genommen hast.



Ojeojeojemmine, ich komme wirklich mit dem Lesen nicht mehr nach und erst recht nicht mit dem Antworten, zumal ich auch morgen nicht dazukomme.

Wort-und Logikkonstrukte kann man doch nicht einfach abtun, sondern sie sind Wesenszüge unseres Denkens.Die gesamte Philosophie und die Kognitionswissenschaften befassen sich ausschließlich mit Paradoxien.
Es kommt auch nicht darauf an, ob Antinomien - wie sie zum Beispiel Kant ausführlich behandelt hat - befriedigend sind.Das ist doch ein Einwand a la Christian Morgenstern:"... dass nicht sein kann, was nicht sein darf!"

Mathematisch lässt sich überhaupt kein Dilemma auflösen. Denn die Mathematik ist doch rein deskriptiv.
Der wohl größte Mathematiker der Neuzeit, Bertrand Russell, schreibt in seiner „Philosophie des Abendlandes“ im 31. Kapitel:
„Dass 2 + 2 = 4 ist, glauben wir nicht, weil wir so oft die Beobachtung gemacht haben, dass ein Paar und noch ein Paar zusammen ein Quartett ergeben. In diesem Sinne ist die mathematische Erkenntnis nicht empirisch. Sie ist aber auch keine apriorische Erkenntnis der Welt. In Wirklichkeit handelt es sich bei ihr nur um eine verbale Erkenntnis. „3“ bedeutet „2 + 1“, und 4 bedeutet „3 +1“. Es ergibt sich also (wenn der Beweis auch lang ist), dass „4“ das gleiche bedeutet wie „2 + 2“.

richy
30.09.11, 19:02
Hi Nick
Nur zur Erhärtung :....
Erhaertung fuer was ? Dass abstrakte Elemente wie Zahlen und mathematische Axiome auch Psi im physikalischen Universum eine Existenz zugesprochen werden sollte ? (Man kann diese Annahme nur intuitiv sich erklaeren) Goedel vertrat diese Meinng. Aber er konnte es nie beweisen. Also sollte man dies als zunaechst unentscheidbar betrachten. Jedoch faellt es uns relativ leicht, dass ein bewusstes Lebewesen wie der Mensch von solch abstrakten Groessen gebrauch macht. Wie beurteilst du das Weizsaecker Zitat ? Trifft er Aussagen bezueglich "nichts" und "nichts physkalisches"
Ist die von ihm geaeusserte Freiheit in Annahmen eventuell eingeschraenkt ?

richy
30.09.11, 20:27
Rehi Marco
Ich wollte nur Begruenden, warum ich dieses Zitat von Weizsaecker nicht nochmal in den Einzelheiten durchgehe. Das Zitat war angeblich mit ein Grund. Ansonsten weiss ich auch nicht.
Ich schrieb damals

Mit Paulis Objektivitaetsbegriff, den du recht deutlich dargestellt hast, haette dieser damals schon das Dekohaerenzprogramm unter den Aspekten der KI vorweggenommen. Daher kann ich dir zustimmen :
Zitat von Gandalf
halte ich "Pauli für den größeren Quantenpyhsiker"
Keine Ahnung was daran negativ sein sollte. Dass Pauli metaphysisch interessiert war, sollte kein Geheimnis mehr sein. Da kann ich ebensowenig dafuer wie fuer Weizsaeckers Erfahrungen in Indien. Und ich werte dies auch nicht persoenlich, sondern nur um mir ein genaueres Bild von der Persoenlichkeit zu erhalten. Bei Pauli ist das uebrigends schwieriger als bei anderen Physikern.
Der Ausdruck naiver Realismus, den ich von Zeilinger uebernommen hatte, hat Zeilinger selbst wohl etwas unvorsichtig verwendet. Am besten schlaegt man bei jedem Wort bei Wiki nach :-) Vielleicht waren es die Heisenbergschnitte, die ich als nicht mehr zeitgemaess betrachte.

Hi Amc
Weil man die SGL benötigt, um bestimmte Vorhersagen über quantenmechanische Systeme zu treffen, muss man sich auch mit der Frage auseinandersetzen, was beschreibt die SGL eigentlich? Hier kann man dann eben einen realistischen Ansatz wählen, oder man sagt, es handelt sich nur um einen mathematischen Formalismus, der keinen realen Vorgang beschreibt. Was aber dann zwangsläufig bedeutet, die Notwendigkeit der SGL mit etwas abstraktem erklären zu müssen ...
"Kein realer Vorgang" bedeutet vor allem "keinen physikalischen Vorgang". Aber das geht aus deine Satz hervor :
... die Notwendigkeit der SGL mit etwas abstraktem erklären zu müssen ...
Daher vollste Zustimmung fuer die komplette Aussage von dir.

Aber gerade deshalb verstand ich deine Aussage nicht so recht :
Das Bewusstsein eines lebenden Organismus spielt üblicherweise in modernen Interpretationen der Quantenphysik keine zentrale Rolle. Definitiv nicht.
Wenn du mit modern die VWI oder BM meinst, ist die Aussage richtig, aber ansonsten nun mal nicht. Ob ein lebender Organismus eine Rolle spielt, haengt davon ab welche Form der Kopenhagener Deutung verwendet wird. An welcher Stelle man den Heisenbergschnitt ansetzt. (Voraussetzung: Nur lebende Systeme kennen Bewusstein)

http://home.arcor.de/richardon/2011/Hschnitt.gif

Das Bewusstsein oder Geist oder Information als Ausdruck einer abstrakten Komponente spielt bei Kopenhagen immer eine Rolle. Zwingend, weil erst damit die Nichtlokalitaet "erklaert" werden koennte. Das EPR Experiment, ist der zentrale Punkt der zwingend zum einem Quantengeist oder alternativ zu vielen Welten fuehrt. Nun wird Prof Lesch in seinen Sendungen aber niemals einen Quantengeist als Erklaerung verwenden sondern dies und das erklaeren aber niemals eine Erklaerung fuer die Nichtlokalitaet sebst. Er wird diese hochstens beschreiben. Hierzu gibt es ein interessantes Video in dem er bei einem Vortrag dazu aufgefortert wird die Nichtlokalitaet zu erklaeren.
Er erklaert dann die Vorstellungen vom Urknall und der Heisenbergschen Unschaerferelation und wie ein Kuehlschrank funktioniert haette auch gepasst. Wenn Prof Lesch sich weigert die Erklaerung genau darzustellen, also gar keine Erklaerung liefert, wie soll dann das breite Publikum sich das EPR Experiment, die Verschraenkung erklaeren koennen ? Und differenzieren koennen wann denn philosopisch vertretbare Aussagen vorliegen oder Quantenesoterik ? Selbst im Forum hier sind die wenigsten dazu in der Lage.

Nehmen wir den Heisenbergschnitt von Wigner. Wuerde man unter dieser Denkweise die Nichtlokalitaet erklaeren, dann wuerde man sich ziemlich laecherlich machen. Dieser Arktikel ueber das schneller alternde Univesum zeigt dies recht beispielhaft. Man ist fast geneigt die Meinung der beiden Wissenschaftlern als Esoterik zu betrachten, dabei argumentieren sie voellig konsistent zu Wiegners Variante der Kopenhagener Interpretation. Dieser Spiegel Bericht ist ein Desinforationsbericht, denn auf diesen Sachverhalt wird nicht hingewiesen. Schlimmer noch. Die beiden werden praktisch der Laecherlichkeit preisgegeben, weil die realistische Variante von Tegmark doch scheinbar so einfach klingt. Ohne zu bemerken, welche Folgen diese realistische Erklaerung mit sich bringt (Viele Welten)
Jeder Leser, der die Zusammenhaenge nicht kennt wird meinen, dass diese beiden Vertreter aus Kopenhagen etwas durchgeknallt sind und dieser Tegmark sicherlich ein vernuenftiger Vertreter des Mainstreams.

Und darum reite ich auch so sehr auf diesem Thema herum. Weil dies hier ein Forum der Quantenmechanik ist. Und wenigstens hier sollte man sich ein Bild ueber die eigentlichen Zusammenhaenge schaffen koennen.

Also meine Frage ist: Warum kann man nicht einfach sagen, wir können Nichtlokalität noch nicht erklären und gut ist?
Weil damit der zentrale Punkt der QM unerklaert bliebe. Das EPR Experiment hat die langjaehrige Diskussion zwischen Bohr und Einstein gegen Einstein entschieden. Die Dinge sind in der QM noch "skuriler" als z.B. Einstein angenommen hatte. Der Zufall, Unbestimmtheit mag eine Rolle spelen, aber der zentrale Punkt, das zunaechst voellig Unverstaendliche ist die Nichtlokalitaet.

shut up and calculate. Die Erklaerung ist, dass wir nichts erklaeren wie in der Ensembleinterpretation. Minimalinterpretation, Positivismus. Uebergabe des Problems an die Philosophen.

Das kann man tun und so wurde/wird auch vorgegangen. Aber die VWI und die BM koennen die Nichtlokalitaet erklaeren. Dann muesste man Kopenhagen verwerfen. Schon eilt aber der edle Ritter in der Not :-) Anton Zeilinger. Wenn man oben die Heisenbergschnitte betrachtet, dann wuerde ich diesen unter Everett schreiben. Wie erklaert der Quantengeist die Nichtlokalitaet ? In der Welt der Geister ist alles zulaessig, also auch solche Phaenomene.
Na so gehts ja nicht. Die Mathematik gehoert auch zu der Welt der Geister (ist abstakt). Dennoch muessen auch hier logische Regeln und Prinzipien eingehalten werden. Ich kann nicht sagen der Satz des Pythagoras ist ein Mathematikmysterium das zu akzeptieren ist. Natuerlich laesst er sich auf Axiome zurueckfuehren.
Und Zeilinger waere eine Erklaerung in diese Sinne nun moeglich. Er muesste lediglich den Formalismus von Everett oder Bohm uebernehmen und die darin verwendeten Groessen als abstrakt deklarieren. Also abstrakt viele Welten oder eine abstrakte Bohmsche Mechanik. Naja irgendwie waere das ja etwas blamabel oder ? Vielleicht findet er aber noch eine andere Moeglichkeit.

BTW:

Ich würde allerdings nicht so weit gehen, zu sagen, dass das Bewusstsein des Beobachters einen unmittelbaren Einfluss auf das Geschehen hat.Manche Kollegen gehen soweit. Für mich persönlich gibt es dafür keinen Grund.
Es sollte jetzt klar sein, warum manche Kollegen so weit gehen und er es sich leisten kann sich davon etwas zu distanzieren. Weil er keinen Heisenbergschnitt mehr verwendet sondern das VWI Dekohaerenzprogramm.
Nur dies gibt ihm einen gewissen Spielraum gegenueber seinen Kollegen. Aber eine voellig freie persoenliche Entschedung wird dies mit der KI nie sein.
Ebensowenig wie es in der VWI keine persoenliche freie Entscheidung ist zusaetliche Dimensionen anzunehmen.

Gruesse

richy
30.09.11, 22:30
Hi Harti
Du betrachtest diesen Holzstab als physikalischen Gegenstand und demonstrierst damit eine abstrakte Beschreibung, z.B. die einer Zahlengerade in deinem Kopf.
Das ist die uebliche Vorgehensweise. Da ist etwas Physikalisches (der Holzstab) und wir beschreiben diesen ( In Form von Koordinaten )
Und du verstehst unter Nichtlokalitaet, dass die -5 und 5 raeumlich getrennt sind, aber dennoch zusammengehoeren. Denn wenn ich die 5 betrachte liegt auf der anderen Seite die -5. Und betrachte ich die -5 dann liegt dort die 5. Das ist sehr aehnlich wie Bells Erklaerung des Einsteinschen Realismus ueber Bertelmanns Socken.
Der trug als zerstreuter Professor immer verschiedene Sockenfarben. Und sah man nun dass er links eine rote Socke trug, so wusste man, dass die rechte Socke nicht rot sein wird, sondern blau. Eine ganz einfache realistische Loesung.
Dieses EPR Experiment und die Bellsch Ungleichung zeigt nun aber, dass diese Loesung mit allergroesster Wahrscheinlichkeit auszuschliessen ist. Im Grunde voellig auszuschliessen.
Wenn man bei deinem Holzstab bleibt und zeichnen bei 5 eine rote Socke und bei -5 eine blaue Socke, dann ist es eher so, dass vor der Dekohaerenz diese Sockenfarbe noch gar nicht existiert. Nehmen wir einen weiteren Holzstab, der fuer unsere raeumliche 3 D Realitaet stehen soll. Dann koennte man mit deinem Modell vereinfacht die Eigenschaft des Realisierens in der Form demonstrieren, dass dein Sockenholzstab zunaechst senkrecht zu dem Holzstab steht auf dem irgendwelche 1D Bewohner hausen. Jetzt dreht jemand den Stab und schwupp erscheinen fuer die 1D Bewohner die entsprechend korrelierten Sockenfarben.
Das waere eine realistische Variante. (Natuerlich dreht sich dabei real nichts, sondern der Sockenstab ist ein hoeherdimensionales Obkjekt)

Es gibt viele Varianten der Kopenhagener Interpretation. Aber allen gemeinsam waere in deinem Beispiel, dass der Sockenholzstab physikalisch ueberhaupt nicht existiert. Er existiert also zum Beispiel nur im Kopf, einem Bewusstsein, oder das Physikalische an ihm geht bei naeherer Betrachtung voellig verloren oder die interessierenden Groessen liegen in irgendeiner Weise in der Natur als Information vor.

Das ist im Grunde so unverstaendlich wie Goedels Annahme der realen Existenz von Zahlen. Man wird die Zahlen versuchen immer an etwas Physikalischem festzumachen. 3 Kuehe 12 Autos ... So meinte dies Goedel natuerlich nicht. Auch nicht, dass Zahlen durch die Landschaft wandern wie in machen Supermarktwerbespots. Obwohl dies dem Sachverhalt vielleicht sogar am naechsten kommt. Man kann nur intuitiv erahnen was Goedel damit meinte. Es ist im Grunde ein voelliger Wahnsinn, voellig Unverstaendlich. Die voellig unabhaengige objektive Existenz abstrakter Dinge ohne jedlichen physikalischen Traeger. (Auch ohne ein bewusstes Gehirn). Dennoch kann man dies nicht ausschliessen, blos weil es unmoeglich ist zu begreifen.
Vieles deutet sogar darauf hin, dass wir abstrakte Zusamenhaenge nicht erschaffen, sondern lediglich entdecken.

Goedels Leben endete jedenfalls tragisch.

Gruesse

Marco Polo
30.09.11, 23:04
Rehi Marco

Rehi Marco? Hmm...nehmen wir mal an, du wolltest so was ähnliches wie Hallo Marco schreiben.

Selbst wenn man mit den fettest denkbaren Wurstfingern ausgestattet derbst auf der Tastatur danebengreift, kommt dabei nie und nimmer Rehi heraus.

Ich gebs auf. :D

richy
30.09.11, 23:29
Ach so...
Rehi ist chat slang So wie rotfl oder 4u,2u, NAK, morf. Re=wieder.
Rehi=Hallo ich bin wieder da. oder hallo da bist du ja wieder. Frueher wurde das verwendet wenn ein User aus dem IRC durch einen Systemabsturz kurzzeitig abwesend war und dann meist etwas zerknirscht sich wieder einloggte.

Zum Gebrauch von @ im Forum.
Man wird die Semantikumwandlung wohl nicht mehr voellig korrigieren koennen, aber @ ist kein Zeichen um Namen zu kennzeichnen sondern das heisst ganz einfach "at". IMHO

%-) t2ul8r ACK ?
EOT

Harti
01.10.11, 08:50
Hallo Richy,

vielen Dank erst mal, dass Du auf meine Überlegungen eingehst und sie einordnest. Es erleichtert mich, dass mich jemand versteht und meine Überlegungen nicht vollständig abwegig sind.


Vieles deutet sogar darauf hin, dass wir abstrakte Zusamenhaenge nicht erschaffen, sondern lediglich entdecken.

Gerade mein Beispiel (Holzstock mit gedachter Nullstelle und Zahlenmarkierungen) zeigt doch aber, dass wir Gegensätze (z.B. plus-minus) nur gedanklich konstruieren, um uns in der Wirklichkeit orientieren und mit einander kommunizieren zu können. Wenn wir dies nicht machen würden, hätten wir keine Möglichkeit uns darüber zu verständigen, wo wir uns auf dem Holzstock befinden.
Dies gilt im Prinzip auch für das Sockenbeispiel. Der Unterschied (Gegensatz) von rot und blau, erklärt sich als verschiedene Stellen im einheitlichen elektromagnetischen Wellenspektrum. Daran ändert auch nichts die Tatsache, dass wir den Unterschied unmittelbar sinnlich wahrnehmen können.
Ich bin der Ansicht, dass es die Grundlage unseres Denkens und der Erfassung der Umwelt ist, Dinge zueinander in Beziehung zu setzen, sie zu vergleichen. Dieses Vorgehen kann, grob gesprochen, drei Ergebnisse haben.
Wir bewerten Dinge als weitgehend identisch (gleich), als teilweise unterschiedlich (quantitativ verschieden) und als vollkommen verschieden (qualitativ verschieden, gegensätzlich).
Gegensatzpaare können rein gedankliche Modelle sein, wie es z.B. Absolutheitsvorstellungen über die Wirklichkeit sind; Beispiele: Gott existiert-Gott existiert nicht; Sein-Nichtsein; absoluter Zufall (ein Geschehen hat keinerlei Ursache)-absolute Kausalität (jedes Geschehen hat immer eine Ursache).
Ich bin deshalb eher der Meinung, dass wir Zusammenhänge mit unserem Denken erschaffen, weil wir uns nicht anders in der Welt orientieren und verständigen können.
Um nicht noch weiter vom Thema abzukommen mach ich hier mal Schluss.

MfG
Harti

richy
01.10.11, 15:34
Hi Harti

Gerade mein Beispiel (Holzstock mit gedachter Nullstelle und Zahlenmarkierungen) zeigt doch aber, dass wir Gegensätze (z.B. plus-minus) nur gedanklich konstruieren, um uns in der Wirklichkeit orientieren und mit einander kommunizieren zu können.
Ja klar, das ist die voellig einsichtige und unkomplizierte Denkweise. Da ist ein Holzstab und ich kann diesen abstrahiert erfassen, vermessen und dies ueber ein abstraktes Modell ausdruecken. Eine Koordinate. Und das Prinzip von Koordinaten stellt mir mein Gehirn zur Verfuegung.
Oder umgekehrt.
Ich moechte das Prinzip einer Koordinate veranschaulichen und verwende dazu ein physikalisches Objekt, zum Beispiel einen Holzstab oder eine Zeichnung davon.

Das physiklische Objekt und dessen Beschreibung sind hier so eng miteinander verknuepft, dass man sich schon ein wenig anstrengen muss beides zu trennen. Solche Vorgaenge ueber etwas aus uebergeordneter Position nachzudenken wird uebrigends mit der Vorsilbe "Meta" bezeichnet.

Dieses Prinzip der Abstraktion kennen wir unmittelbar nur von uns selbst. Aber es laesst sich ueberall in der belebten Natur beobachten. Wenn z.B. das Prinzip der Kommunikation verwendet wird, dann muessen Vorgaenge abstrahiert werden um sie zu codieren und weiterzugeben. Die von mir so gerne zitierten Ameisen sind dazu in der Lage und ohne Zweifel koennen selbst diese abstrahieren. Denn sie kommunizieren und wenden somit dieses Prinzip an. Deren funktional ausgekluegelten Bauwerke waeren ansonsten gar nicht moeglich.

Irgendwann hat der Mensch erkannt, dass dieses Prinzip so nuetzlich und elementar ist dass er die Wissenschaften selbst danach eingeteilt hat. Die moderne Wissenschaft basiert auf einem Zusammenspiel zwischen Geistes (Mathematik) und Naturwissenschaften. (Zusammenspiel in beiden Richtungen). Tolle Leistung :-) Eine Ameise wuerde deshalb vielleicht nur mal kurz mit dem Fuehler wackeln :-)

Wenn man sich einen groben Ueberblick darueber verschaffen will, wie die Dinge im Universum denn funktioneren koennten, ist es oft hilfreich sich selbst und die Natur zu betrachten, nicht nur einen Labortisch. Insbesonders auch Kollegen wie Ameisen zu betrachten. Die belebte Natur, die von der Physik meist ausgeklammert wird. Und dann sehen wir, dass viele Prinzipien, scheinbar menschliche besondere Leistungen im Grunde alte Huete sind. Wir kopieren nur die Natur oder den Menschen selbst und oft bemerken wir dies nichteinmal.

Gruesse

richy
01.10.11, 16:13
Teil 2
Eine Modellbildung ist wie gezeigt eine voellig natuerliche Vorgehensweise.
Jetzt hatte ich Goedels Existenz von Zahlen in Zusammenhang mit der Kopenhagener Interpretation gebracht.

Goedel ging davon aus, dass Zahlen an fuer sich existieren. Wie kann man sich das vorstellen ? Im Grunde fast gar nicht. Man entfernt alles Physikalische, also das komplette Universum, die Raumzeit und dann meinte Goedel bleiben dennoch die Zahlen existent. Anders ausgedrueckt : Zahlen existierten bereits vor dem Urknall. Verrueckt aber intuitiv durchaus noch vorstellbar. Bohr war ein Zeitgenosse von Goedel und vielleicht von diesem inspiriert, wobei ich diese Motivation als Supergau der Physik betrachten wuerde.

Kopenhagen nimmt nun an, dass die Wellenfunktion tatsaechlich ein Eigenleben fuehrt, wie es Goedel fuer die Zahlen allerdings nur vermutete.
Die Wellenfunktion eine Beschreibung ohne physikalisches beschriebenes Objekt darstellt. Die SGL etwas beschreibt ohne dass es etwas gaebe, dass beschrieben wird. Dabei ist die Interpretationsform von Wigner, die inzwischen als laecherlich abgetan wird, noch eine der Vernuenftigsten. Denn immerhin steht das Bewusstsein eines Organismus wie dem Menschen noch in einem Zusammenhang zu etwas Physikalischem. Dem Gehirn. Wie dieses Gehirn aber konkret auf einen Versuchsaufbau einwirken soll bleibt natuerlich ein kleineres problematisches Detail :-)

"Aber mit der Dekohaerenz wird alles gut". Ueberhaupt nicht. Um es mal ganz einfach auszudruecken. Dies fuehrt zu dem Modell, dass das komplette Universum mit einem abstrakten Anteil unmittelbar verknuepft ist. Noch expliziter. Das Universum wird nun selbst zu einem lebendigen Organismus der ueber ein Bewusstsein verfuegt. Und dieses Bewusstsein nennt Zeilinger Information.
Naja, das klingt zwar abgefahren, aber wenn man dies lediglch bildlich betrachtet, so entspricht dies doch vielen Beobachtungen und unseren eigenen Erfahrungen wird man denken.

Es kommt aber noch schlimmer. Denn letztendlich darf Kopenhagen diese Informationsbausteine, Bits keinem physikalischen Traeger zuordnen. Bei einem Heisenbergschnitt war dies noch unproblematisch, denn die physikalische Welt war durch diesen von der abstrakten Quantenwelt getrennt. Mit der Dekohaerenz geht diese Trennung aber verloren und die Konsequenz ist, dass das Universum selbst ein rein geistiges Konstrukt ist. Seine Bausteine sind rein geistig abstrakt und daran aendert sich auch nichts wenn man diese mit einem eingaengigen Namen wie "Information" versieht.

So eine Weltvorstellung waere nahe dem Wahnsinn und so ist es sicherlich auch Zeilinger klar, dass er hier Eingestaendnisse machen muss. Dass er der Information einen physikalischen Informationstaeger zuordnen muss.
Auf der anderen Seite, dem Realismus, sind solche Eingestaendnisse weniger zwingend, aber dennoch werden sie vollzogen. Siehe Penrose, Wheeler.
Diese Eingestaendnisse seitens der Realisten sind jedoch weitaus einfacher, denn das Hinzufuegen abstrakter, geistiger Elemente entspricht unseren Erfahrungen.

Zeilinger muesste dagegen etwas hinzufuegen, dass nunmal nicht unseren Erfahrungen entspricht. Sobald er neben der Dekohaerenz weitere Zugestaendnis an den Realismus macht bekommt er das Viele Welten Komplettprogramm ab. Ein bischen Physkalitaet gibt es nunmal nicht.
Zeilinger wird es gelassen sehen, denn ihn interessiert es sicherlich nur, dass er seine Experimente durchfuehren kann. Das ist auch gut so. Die Experimente sind wichtig und wer dies durchfuehrt ist im Grunde genommen egal.

Bauhof
02.10.11, 11:24
Weil man diese abstrakten Größen zur korrekten mathematischen Beschreibung benötigt, kann man dann hinterher aber nicht einfach so tun als hätte dies gar keine Bedeutung?

Hallo amc,

von Richy wurdest du aufgefordert, ein Zitat von Weizsäcker zu analysieren. Hier ist das nochmals das Zitat, allerdings etwas erweitert. Weizsäcker schreibt auf Seite 226 seines Buiches [1] folgendes:

Die Kopenhagener Auffassung wird oft, sowohl von einigen ihrer Anhänger wie von einigen ihrer Gegner, dahin missdeutet, als behaupte sie, was nicht beobachtet werden kann, das existiere nicht. Diese Darstellung ist logisch ungenau. Die Kopenhagener Auffassung verwendet nur die schwächere Aussage: "Was beobachtet worden ist, existiert gewiss; bezüglich dessen, was nicht beobachtet worden ist, haben wir noch die Freiheit, Annahmen über seine Existenz oder Nichtexistenz einzuführen."

Von dieser Freiheit macht sie dann denjenigen Gebrauch, der nötig ist, um Paradoxien zu vermeiden. So bedeuten Heisenbergs Gedankenexperimente über das Unbestimmtheitsprinzip nur die Widerlegung eines Vorwurfs der Inkonsistenz. Akzeptieren wir den Formalismus der Quantentheorie in seiner üblichen Deutung, so müssen wir zugeben, dass er keine Zustände umfasst, in denen ein Teilchen zugleich einen bestimmten Ort und einen bestimmten Impuls hat; es erscheint notwendig, diese Zustände auszuschließen, um Widersprüche gegen die Wahrscheinlichkeitsvorhersagen der Theorie zu vermeiden.

Nun wird der Einwand erhoben, dass sowohl Ort wie Impuls messbar sind und somit existieren. Diesem Einwand begegnet die Feststellung, dass, falls die Quantentheorie richtig ist, Ort und Impuls nicht zugleich gemessen werden können, und dass darum der Verteidiger der Quantentheorie nicht gezwungen werden kann, zuzugeben, dass sie zugleich, d.h. im selben Zustand des Teilchens, existieren.

Weizsäcker zu verstehen, ist leider nicht einfach. Kannst du vielleicht trotzdem eine Quintessenz mit einfacheren Worten aus diesem Zitat ziehen?

Mit freundlichen Grüßen
Eugen Bauhof

[1] Weizsäcker, Carl Friedrich von
Die Einheit der Natur. Studien. (http://www.amazon.de/Einheit-Natur-Carl-Friedrich-Weizs%C3%A4cker/dp/B002DU7JJ0/ref=sr_1_2?s=books&ie=UTF8&qid=1317550692&sr=1-2)
München 1984. dtv. Vierte Auflage.
ISBN=3-423-10012-5

amc
02.10.11, 14:33
Weizsäcker zu verstehen, ist leider nicht einfach. Kannst du vielleicht trotzdem eine Quintessenz mit einfacheren Worten aus diesem Zitat ziehen?

Hallo Eugen,

insgesamt verstehe ich von diesen Dingen ja nicht allzu viel - ich versuche es aber gerne:

Die bisher unwiderlegte Erkenntnis, dass der Ort und der Impuls eines Teilchens zur selben Zeit nicht exakt bestimmbar ist, lässt die Annahme zwingend erscheinen, diese Eigenschaften niemals als dem Teilchen an sich innewohnend, von dem eigentlichen Ereignis (Messung, Wechselwirkung) losgelöst definierte Zustände, begreifen zu dürfen.

Würden alle Zustände zu jeder Zeit definiert sein, so würde der Begriff der Wahrscheinlichkeit in der Quantenphysik keine Rolle mehr spielen können - alles wäre determiniert. Wenn nun also die Zustände vor dem eigentlichen Ereignis als nicht definiert anzusehen sind, dann kann man sich auch die Frage stellen, wie sinnvoll es überhaupt noch ist hier von Zuständen (vor der Messung) zu sprechen ...


Freundlichst,
AMC

richy
02.10.11, 18:36
Hi AMC

Super zusammengefasst ! Vor allem kurz und praegnant.
Vielleicht kannst du in der Form noch ein wenig weiter vermittelnd wirken, indem du deine Einschatzungen dazu schreibst. Mir stand nur der erste Teil des Zitates zur Verfuegung, ohne das Beispiel der Heisenbergschen Unbestimmtheitsrelation. Die einleitenden Worte beziehen sich jedoch auf die ganze Kopenhagener Interpretation nicht nur dieses Beispiel. Weizsaecker spricht hier konkret die Begriffe "Existenz" und "Nichtexistenz" an. Darum hatte ich dieses Zitat auch nochmals erwaehnt.
Da kommen wir der Sache vielleicht ein Stueck naeher was die SGL gemaess Kopenhagen beschreibt. Ganz grob und dennoch recht zutreffend ausgedrueckt : Sie beschreibt NICHTS !
(Anmerkung: Nichts physikalisches)
Sie ist selbst. Dem wuerde Prof Weizsaecker erwidern ...

Was beobachtet worden ist, existiert gewiss; bezüglich dessen, was nicht beobachtet worden ist, haben wir noch die Freiheit, Annahmen über seine Existenz oder Nichtexistenz einzuführen."
Von dieser Freiheit macht sie dann denjenigen Gebrauch, der nötig ist, um Paradoxien zu vermeiden.

- Wie verstehst du hier die Begriffe Existenz und Nichtexistenz ?
- Ist speziell eine physikalische Existenz gemeint ?
- Wie koennte man eine physikalische Nichtexistenz deuten ?
- Waere der Waegbarkeitsbegriff (incl. Eigenschaften bezogen auf Waegbares und Unwaegbares) fuer eine Differenzierung geeignet ?
- Welche Masse hat ein C70 Molekuel im ueberlagerten Zustand ?
- Ist Masse eine abstrakte Eigenschaft ?

Mein Kritikpunkt hier war uebrigends lediglich, dass mit der Kopenhagener Deutung keine voellige Freiheit bezueglich des physikalischen Existenzbegriffes besteht.

So bedeuten Heisenbergs Gedankenexperimente über das Unbestimmtheitsprinzip nur die Widerlegung eines Vorwurfs der Inkonsistenz.Meine Vorstellung zur Unscharferelation besser Foriertransformation hatte ich hier versucht darzustellen :
http://www.quanten.de/forum/showpost.php5?p=63267&postcount=36
Soll man den Orgelbauern eine Inkonsitenz bei Residualpfeifen vorwerfen ?
Was ich an Weizsaeckers Satz ueberhaupt nicht verstehe ist das Woertchen "nur". Kannst du mir erklaeren was "Heisenbergs Gedankenexperimente über das Unbestimmtheitsprinzip" denn nun nicht zeigen sollen/koennen ? Das wird durch den Zusatz "nur" impliziert.

in denen ein Teilchen zugleich einen bestimmten Ort und einen bestimmten Impuls hat; es erscheint notwendig, diese Zustände auszuschließen, um Widersprüche gegen die Wahrscheinlichkeitsvorhersagen der Theorie zu vermeiden.
In deiner Zusammenfassung kam sogar fast noch besser um Ausdruck, dass es hier um drei (vier) Groessen geht. Der Ort, der Impuls, das "Teilchen" vor und nach der Messung, (der Zufall).
Weizsaecker hatte ein Beispiel fuer die Existenz und Nichtexistenz gewisser Groessen vor der Dekohaerenz (er nennt das noch Messung) angekuendigt. Orten wird man wohl eine Existenz zubilligen und ein Impuls ist eine Eigenschaft eines in der Regel physikalischen Objektes. Dazu verknuepft mit dessen Dynamik und damit auch dessen Ort. Somit geht es doch um den Existenzbegriff dieses Objektes, das er Teilchen nennt. Warum ueberhaupt Teilchen ? Dass ein Teilchen nach der Messung existiert war wenigstens vor dem Dekohaerenzprogramm keine strittige Frage. Von Interesse ist der Zustand vor der Dekohaerenz. Dazu wollte Prof Weizsaecker ein Beispiel geben und spricht nun von einem Teilchen. Seine Argumentation wuerde daher sehr gut fuer die Bohmsche Mechanik zutreffen.
Ich kann nicht erkennen inwiefern er fuer dieses "Teilchen" den Existenzbegriff vor der Messung im Rahmen der KI erklaert hat.
Was hat Herr Weizsaecker denn nun erklaert / interpretiert ?

"Morgends wird es hell und abends dunkel."
Das ist eine Beobachtung. Eine Erklaerung, Interpretation ist etwas anderes. In einer solchen wuerde in dem Beispiel die Sonne eine Rolle spielen.

Ich zweifle uebrigends nicht daran, dass die Kopenhagener Interpretation die Nichtlokalitaet interpretieren kann. (Den Zufall zu interpretieren ist unmoeglich und die Heisenbergsche Unbestimmheit eine Folge der Fouriertransformation)
Vielleicht kann Eugen in "Die Einheit der Natur. Studien." eine entsprechende Stelle zitieren.

Gruesse

richy
02.10.11, 19:05
zu
Super zusammengefasst ! Vor allem kurz und praegnant.
Das bedeutet natuerlich nicht, dass ich die Meinung dieser Zusammenfassung in allen Punkten teile. Es ist klar, dass konjungierte Groessen wie Zeit und Frequenz nicht zu einem gemeinsamen Beschreibungssystem gehoeren. sin(2*Pi*f*t) Das ist ein Spezialfall und wenn ich hier lediglich f angebe, lasse ich alles andere weg.
Ich meine nicht, dass man ueber y=sin(2*Pi*f*t) und den Zufall die Nichtlokalitaet der QM erklaeren kann und damit auch nicht ueber die Unbestimmtheitsrelation. Und man kann auch nicht sagen Zeit und Frequenz sind nicht gemeinsam definiert.
Anscheinend gibt es zur Interpretation der Unbestimmtheitsrelation konkretere Kritikpunkte seitens der realisitischen Varianten. Rein interessehalber suche ich die mal in Zehs Paper und stelle sie hier in den Thread.

Knut Hacker
02.10.11, 19:12
Das Problem ist wohl der Widerstreit zweier physikalischer Grundprinzipien, die insbesondere in der Chaosforschung eine zentrale Rolle übernommen haben

1) Das eine besagt, dass das Ganze nicht lediglich die Summe seiner Teile ist, sondern qualitativ - das heißt wegen seiner dynamischen Strukturen - etwas anderes. Wir sind Teil der Natur, daher passt die Natur nicht in unsere Köpfe. Demnach sind uns unüberwindliche Erkenntnisgrenzen gesetzt. Das ergibt sich, wie schon diskutiert, auch aus den Gödelschen Sätzen, wonach kein System sich aus sich selbst heraus verstehen kann, also auch nicht unser Erkenntnissystem.

2) Das andere Grundprinzip ist das der Selbstähnlichkeit.In jedem Teil bilden sich die dynamische Strukturen des Ganzen ähnlich ab. Demnach könnten wir als Teil der Natur diese durchaus analog erfassen.
Hier taucht jedoch das Problem auf, dass diese Strukturen allesamt zirkulär oder infinit (nichtlinear, fraktal) sind. Siehe das Mandelbrod - Äpfelchen! Angenommen wir fänden eine Weltformel, dann könnte sich diese nicht selbst erklären, sondern es blieben zwei Fragen: warum gibt es überhaupt eine solche und warum gerade sie und keine andere.

Zu Friedrich von Weizsäcker, der hier dankenswerterweise viel Anklang gefunden hat, noch folgende Zitate, wobei ich das letzte besonders hervorheben möchte:

1) aus „Der Mensch in seiner Geschichte“:

„Die Quantentheorie ist eine statistische, eine Wahrscheinlichkeitstheorie. Ihr Kern ist eine nicht klassische Wahrscheinlichkeitsrechnung, charakterisiert durch das sogenannte Superpositionsprinzip. Die Wahrscheinlichkeitsrechnung setzt Logik voraus. Die nichtklassische Wahrscheinlichkeitsrechnung der Quantentheorie selbst nach J. v. Neumann eine nichtklassische Logik voraus, die er Quanten-Logik nannte.“

„Das tiefe, noch nie genug verstandene philosophische Problem der Quantentheorie liegt aber darin, dass wegen des Superpositionsprinzips jede klassische, faktische Beschreibung des Geschehens nur eine Näherung ist. Wenn die Quantentheorie richtig ist, so ist die Wirklichkeit in Strenge niemals faktisch.“

2) aus „Ein Blick auf Platon“:

„Die logischen Paradoxien verwenden stets die Negation beziehungsweise die Falschheit, so der Kreter, der versichert, das, was er soeben sage, sei falsch, oder die Menge aller der Mengen, die sich selbst nicht als Element enthalten. Das von Brouer kritisierte Tertium non datur versichert, dass ein Satz und seine Negation die Menge der Möglichkeiten ausschöpfen. J. v. Neumanns Quantenlogik behauptet, dass eine Eigenschaft einem physikalischen Objekt nicht mit Notwendigkeit entweder zukommt oder nicht zukommt.“

3) aus „Große Physiker“:

„... es kann nicht eindeutig unter allen Umständen behauptet werden, dass ein gegebenes Teilchen sich in einer bestimmten Position befindet oder nicht.... es gibt nicht eine unzweideutige Beschreibung der Welt.“

„... das nach der Quantentheorie Ganze nicht aktuell, sondern nur potentiell aus Teilen „bestehen“. Ein Ganzes kann durch Zerstören seiner Struktur in Teile zerlegt werden, aber es kann nicht korrekt als aus den Teilen bestehend beschrieben werden,...“ (zum EPR- Paradoxon)

„Stellen wir uns nun ein Universum vor, welches alle Dinge und alle Beobachter enthalten würde. Würde es sich in einem wohldefinierten Quantenzustand befinden, dann würde keines der Dinge und keiner der Beobachter wirklich existieren. Es könnte dann nicht sinnvoll gesagt werden, dass irgendeines seiner Objekte irgendeinem seiner Beobachter bekannt wäre; in metaphorischer Sprache könnte man nur sagen, dass alle Objekte und alle Subjekte in dem einen Geist verschwunden sind.“

Knut Hacker
02.10.11, 20:02
Ich will zum Ausgangsproblem der Quantenlogik zurückkehren, Diese mehrwertige, komplementäre Logik war bereits vor der zweiwertigen, ein Drittes ausschließenden aristotelischen Logik bekannt, nicht nur in Indien und natürlich in China im Buddhismus/Taoismus, sondern auch im Judentum. Der Talmud ist ein einziges Lehrbuch der Quanten – Logik! Es wird immer gefragt, warum das jüdische Volk so unverhältnismäßig viele intelligente Köpfe hervorgebracht hat. Ein Grund dafür könnte die bereits frühkindliche Schulung am Talmud sein.

Einstein war zwar nicht streng jüdisch - religiös – er bezeichnete sich als Spinozist - , aber er schätzte das talmudische Querdenken.So prangerte er mit seinem Ausspruch: “Der gesunde Menschenverstand ist eigentlich nur eine Anhäufung von Vorurteilen, die man bis zum 18. Lebensjahr erworben hat“, an, dass die Leute darüber klagen, sie „verstünden“ die moderne Physik (Relativitätstheorie, Quantenphysik) nicht. Die Unverstehbarkeit sei aber doch gerade das Revolutionäre an den Erkenntnissen der modernen Naturwissenschaft.

Zur Auflockerung des Threads möchte ich das Thema Quantenlogik zunächst mit drei jüdischen Witzen illustrieren:

Ein Ratsuchender kommt zum Rabbi und fragt diesen, ob es Gott gibt. Der Rabbi antwortet: "Gott ist so über allem erhaben, dass er es nicht nötig hat, zu existieren."

Zwei streitende Brüder kommen zum Rabbi. Der eine trägt vor: „ Meine Frau ist krank. Ich bitte Gott, dass er sie heilt. Tue ich recht“, Der Rabbi antwortet: „Ja, mein Sohn, du tust recht.“ Darauf trägt der andere vor: „Gott tut immer das Beste und lässt sich in seiner Allmächtigkeit nicht umstimmen. Es ist also frevelhaft, ihn um etwas zu bitten.Habe ich recht?“ „ Ja, du hast recht!“ antwortet der Rabbi. Verstört ziehen die Streithälse von dannen. Der Schüler des Rabbi, der diesem zugehört hatte, hält seinem Lehrmeister vor: „ Die beiden hatten gegensätzliche Standpunkte. Sie können doch nicht beide recht haben!“ Der Rabbi antwortete: „Du hast recht!“

Zwei Juden besuchen ein Lokal und bestellen sich jeder einen Fisch. Zwei Fische werden auf einem Tablett serviert, ein großer und ein kleiner. Der eine Jude zum anderen: „Nu, so greif doch zu!“ Der andre: „So nimm doch du zuerst!“ Der erstere:“Aber bitte, ich lasse dir den Vortritt.“ Der andere: "Vielen Dank, aber ich überlasse dir die erste Wahl.“ So geht das eine Weile hin und her, bis sich schließlich der erstere den größeren Fisch nimmt. Nach kurzem Schweigen bemerkt der andere: „ Hätte ich zuerst zugegriffen, hätte ich den kleineren genommen!“ Darauf der erste: “Nu, hast du nicht den kleineren?“

Und nun eine heitere Erläuterung des Talmuds und damit der Quanten – Logik:

Ein Talmudgelehrter wurde einmal gefragt, worin eigentlich die Weisheit des Talmud bestehe.

„Das ist sehr schwierig“, antwortete er. „Ich will es Euch an einem Beispiel erklären und Euch eine talmudische Frage stellen: Zwei Juden wandern über die Dächer einer Stadt. Der eine schaut in einen Schornstein hinein und lehnt sich dabei so weit vor, dass er hineinfällt. Der zweite will ihm helfen, aber er fällt auch hinein. Wie sie nun herauskommen, ist der eine schwarz von Ruß, der andere ist sauber geblieben.Welche wäscht sich?“

„Der Schmutzige natürlich.“

„Falsch. Der Saubere geht sich waschen, denn er sieht, dass der andere schmutzig ist, und glaubt, er selbst sei es auch. Der Schmutzige aber sieht, dass der andere sauber ist, und glaubt, er sei auch sauber. Also wäscht er sich nicht.“

„Aha“, sagte der Frager.

„Das ist aber erst der Anfang“, fuhr der Talmudist fort. „Ich will Euch eine zweite Frage stellen: Die beiden fallen noch einmal in den Schornstein, einer kommt schwarz heraus, der andere sauber. Welcher wäscht sich?“

„Jetzt weiß ich's: der Saubere.“

„Nein, beide. Der Schmutzige sieht, dass er schmutzig ist, also wäscht er sich. Der Saubere aber sieht den Schmutzigen und glaubt, er sei auch schmutzig, also geht er sich waschen. Aber das ist noch nicht alles. Ich will Euch eine dritte Frage stellen: Zwei Juden gehen über die Dächer und fallen in einen Schornstein.Einer kommt schwarz heraus, der andere sauber. Welcher wäscht sich?“

„Beide.“

„Wieder falsch. Keiner von beiden geht sich waschen. Der Sauberes sieht, dass er sauber ist, also braucht er sich nicht zu waschen. Der Schmutzige aber sieht, dass der andere sauber ist, also glaubt er, dass er auch sauber sei, und geht sich nicht waschen. Es geht aber noch weiter. Die beiden fallen wieder in den Schornstein, einer kommt schwarz heraus, der andere sauber. Welcher wäscht sich?“

„Ich weiß ich nicht.“

„Nun, der Schmutzige natürlich. Weshalb sollte sich denn der Saubere waschen? Der Schmutzige aber sieht, dass er schwarz ist, also geht er sich waschen. Ich sagte ja, dass es sehr schwierig ist.Aber auch damit sind wir noch nicht am Ende. Erstens: Habt Ihr schon einmal zwei Juden über die Dächer spazieren sehen? Und gesetzt den Fall, sie taten es: Selbst wenn einer in den Schornstein gefallen wäre, wozu sollte der andere ihm unbedingt folgen? Also nehmen wir einmal an, es war so. Ist es jemals vorgekommen, dass von zwei Männern, die durch den gleichen Schornstein gefallen sind, der eine schwarz herauskommt und der andere sauber? Seht Ihr, so ist es mit dem Talmud.“ (Nach einer alten Überlieferung wiedergegeben von Filipp Goldscheider)

richy
02.10.11, 20:15
Hi Knut
Ich will auf jeden Fall auf die von dir angesprochenen Problematiken im Zusammenhang mit Goedel und der nichtlinearen Systemdynamik (Chaostheorie) eingehen. Zuvor moechte ich aber genauer verstehen inwiefern die Unbestimmtheitsrelation insbesonders in der Kopenhagener Interpretation als Interpretationshilfe, Argument ueberhaupt verwendet wird. Fuer mich ist das zunaechst reine Mathematik mit einem interessanten Aspekt, Analogon zu unseren Sinnenwahrnehmungen.
Ich wuerde dazu hier gerne noch naeher ergruenden, wann denn der Begriff Unschaerferelation und wann der Begriff Unbestimmtheitsrelation angebrachter ist. Das sind zwei paar Stiefel, wobei die Unschaerferelation wohl die allgemeinere Form ist um den Zusammenhang zwischen Ur und Zeitbereich einer F-Transformierten darzustellen und die Unbestimmtheitsrelation den Spezialfall darstellt, falls der Prozess stochastischer Natur ist. Wobei mir hier als Ing im Grunde sofort zusaetzlich einfaellt, dass man die F Transformation in diesem Fall keinesfalls verwenden sollte, sondern moderne Spektral Schaetzungsmethoden. Die F-Transformation wird dabei wohl jedoch die physikalische Vorgabe der Natur sein, also sachgemaess. Wenn ich nun |PSI|^2 als Spektrum betrachte duerfte dies in Details im Grunde gar nicht richtig zusammenpasen. *an Kopp kratz :)

Ich denke mal dass du einen guten und vor allem freien Ueberblick bezueglich der Kopenhagener Interpretation hast. Kannst du kurz skizzieren welche Rolle hier die Unschaerferelation im Rahmen der Interpretation einnimmt ? (Ohne jedliche Aspekte einer Selbstbezueglichkeit)
Gerne rein persoenlch fuer deine Sichtweise. Wie du das Ganze verstehst, also Aspekte der KI fuer dein persoenliches Weltbild verarbeitest.
Zwischenzeitlich sichte ich mal Zehs Paper weiter nach einer Antwort.
Gruesse

richy
02.10.11, 21:52
Zeh zu Kompelmentaerem :

Diese pragmatische Haltung beruft sich überwiegend auf die Kopenhagener Deutung von Niels Bohr, doch meistens ohne deren volle Konsequenzen zu erfassen. Bohr hatte auf Grund seiner langen und tiefen Erfahrung schon vor 1925 erkannt, daß es „keine einfache Lösung“ der Probleme der Quantenphysik geben könne. Damit erteilte er allen „naiven“ (traditionellen) Versuchen einer Interpretation eine auch nachträglich berechtigte Absage. Er ging aber weiter, indem er auf einem rein operationalistischen Umgang mit der Theorie bestand, der ausdrücklich begriffliche Inkonsistenzen („komplementäre“ klassische Beschreibungsweisen) zuläßt. Daß dies die Aufgabe eines Realitätsbegriffes in der Mikrophysik verlangt, kam Bohr sogar entgegen, da er philosophisch nachweisbar vom Irrationalismus beeinflußt
war, der damals auch für einige führende Naturwissenschaftler im Zeitgeist lag. Tatsächlich ergab sich diese explizite Absage an eine „reale Quantenwelt“ aber als ein Ausweichen auf ganz konkreten Konsistenzfragen, etwa ob das Elektron nun „wirklich“ ein Teilchen oder eine Welle sei.
http://www.rzuser.uni-heidelberg.de/~as3/KarlsruheText.pdf

Was ist Operationalismus ?
********************
http://de.wikipedia.org/wiki/Operationalismus

Nach Bridgmans Auffassung ist wissenschaftliche Objektivität gegründet auf die Verwendung von operationalen Definitionen. Damit ein wissenschaftlicher Begriff empirisch gültig sei, müsse er in Form von spezifischen Messverfahren definiert werden; so wie etwa „Länge“ dadurch definiert werde, dass man ein Metermaß an das jeweilige zu messende Objekt anlege. Die Bedeutung eines Begriffes erschöpfe sich also in einer Reihe angebbarer Messoperationen; der Begriff sei bedeutungsgleich mit den betreffenden Operationen. Damit hat Bridgman nicht nur die Kriterien der Gültigkeit (Validität) eines Begriffs außerhalb des menschlichen Bewusstseins angesiedelt, sondern den Begriff selber mit einer Menge bewusstseinsunabhängiger Operationen gleichgesetzt.

Das klngt vernuenftig. Nur was ist Messen ? Das Interferenzmuster kann ich vermessen. Aus operationalistischer Sicht ist es existent. Klar es ergibt sich erst aus vielen Ereignissen. Aber jedes dieser Einzelereignisse ist z.B. als schwarzer Punkt existent, da vermessbar. Und damit gibt es keine Komponente die nicht vermessbar waere und damit muss dem Wellencharakter auch aus operationeller Sicht eine Existenz zugesprochen werden. Und da jeder Punkt des Bildes zur Existenz des Interferenzbildes beitraegt ist in jedem Punkt die Existenz einer Welle manifestiert. Sogar in unserer Relitaet.

Objektiver Zufall
************
Nehmen wir mal an es gaebe weder einen objektiven noch determinierten Zufall=Chaos.
Wie wuerde dann der Spaltversuch ablaufen ?
Das ist mir eben gerade so eingefallen. Ich kann das auch nicht spontan beantworten. Wuerde man dann bei dem Versuch lediglich einen scharfen Punkt beobachten ? Das waere das logisch sinnvollste.
Und wir haetten keinerlei Probleme und wuerden daraus schliessen :
So ein Elektron ist ganz klar ein Teilchen.

Nehmen wir mal an es gibt irgendeinen Grund, dass es fuer ein vernunftbegabtes Wesen von einer gewissen Bedeutung sein koennte, dass
ein Elektron kein Teilchen ist, sondern eine (hochdimensionale) Welle. Wie koennte man dies diesem vernunftbegabten Wesen dies vermitteln ?
Ohne Zufall wuerde dann beim Spaltversuch das Interferenzmuster aehnlich wie bei einem TV Geraet erzeugt werden. Naja. Das waere eine recht triviale Methode. An diese Stelle moechte ich nochmals, insbesonders Knut, auf meine Phas-o-mat Bilder hinweisen.

Unbestimmtheitsrelation
******************
Wenn Ort und Impuls eines Teilchens/einer Welle ueber eine FT miteinander verbunden sind und dies dazu fuehrt, dass nur das eine oder andere scharf, mit Bestimmtheit gemessen werden kann, dann sehe ich hierin keine Eigentuemlichkeit ausser jener, warum beide Messgroessen in dieser Form miteinander verknuepft sind.
Der Thread ist schon wieder viel zu lang und aussschweifend :(
Ich schicke ihn also mal in der Form zunaechst ab.

richy
02.10.11, 22:02
Hi Eugen
Ich moechte dir eine einfache Frage stellen :
Laesst sich mit der Heisenbergschen Unbestimmtheitsrelation die Nichtlokalitaet der Quantenmachanik erklaeren ?

Vielen Dank fuer eine Antwort

richy
02.10.11, 23:04
Hi Knut

Ich gehe deine Beitraege erstmal von vorne nach hinten durch.
Einstein war sicherlich juedischer Herkunft. Wobei man bedenken sollte, dass sich jedliches Erbmaterial so schnell vermischt, so schnell kannst du gar nicht hinschaeuen. Aber gewisse Besonderheiten von Kulturen werden schon mal von Generation zu Generation weitergegeben. Diese abstrakten Elemente einer Kultur sind weitaus resistenter als irgendwelche physikalsischen DNA Faktoren.
Dein erster Schwank gefaellt mir besonders gut :) Aber es ist ja hier ein Physikforum. Der zweite Schwank :

Zwei Juden besuchen ein Lokal und bestellen sich jeder einen Fisch. Zwei Fische werden auf einem Tablett serviert, ein großer und ein kleiner. Der eine Jude zum anderen: „Nu, so greif doch zu!“ Der andre: „So nimm doch du zuerst!“ Der erstere:“Aber bitte, ich lasse dir den Vortritt.“ Der andere: "Vielen Dank, aber ich überlasse dir die erste Wahl.“ So geht das eine Weile hin und her, bis sich schließlich der erstere den größeren Fisch nimmt. Nach kurzem Schweigen bemerkt der andere: „ Hätte ich zuerst zugegriffen, hätte ich den kleineren genommen!“ Darauf der erste: “Nu, hast du nicht den kleineren?“

He he. Eine geniale Situationskomik die zum Denken anregt. :) Der juedische Humor ist vortrefflich :)

Kennst du die Geschichte der Waldenser ?
Deine Vorfahren waren wohl Juden. Meine Vorfahren waren Waldenser.

Viele Gruesse

Marco Polo
03.10.11, 02:43
Meine Vorfahren waren Waldenser.

Ach daher auch der Name Richardon. Die stammten dann wohl aus Savoyen?

Deine Vorfahren waren Waldenser und du bist Badenser (Gelbfüssler). :D

Grüzi, MP

Bauhof
03.10.11, 10:34
Hi Eugen
Ich moechte dir eine einfache Frage stellen :
Laesst sich mit der Heisenbergschen Unbestimmtheitsrelation die Nichtlokalitaet der Quantenmachanik erklaeren? Vielen Dank fuer eine Antwort

Hallo Richy,

die Nichtlokalität der Quantenmechanik lässt sich meines Wissens nicht mit der Unbestimmtheitsrelation erklären. Ich habe jedenfalls darüber in der Literatur nichts gefunden, wo solches behauptet würde. Die Nichtlokalität ist eine intrinsische Naturtatsache, die sich im Quantenprozess zeigt. Die Quantenmechanik ist nicht inhärent nichtlokal. Aber sie zeigt nichtlokales Verhalten, wenn sie aus klassischer Sicht untersucht wird.

Falls ich Claus Kiefer richtig verstanden habe, zeigt sich die Nichtlokalität der Quantentheorie durch die Verschränkung mit der Umgebung. Er schreibt dazu in seinen Büchern [1] und [2] folgendes:

Der Prozess der Dekohärenz erklärt, warum makroskopische Objekte lokalen Beobachtern klassisch erscheinen - diese klassischen Eigenschaften wohnen dem Objekt nicht inne, sie werden erst durch die Wechselwirkung mit der Umgebung definiert. Lokale klassische Eigenschaften werden also durch die Nichtlokalität der Quantentheorie (Verschränkung mit der Umgebung) bewirkt.

[...]

Nur für den Spezialfall, dass der Gesamtzustand (die Gesamtwellenfunktion) gleich dem Produkt der Einzelzustände ist, sind die Teilsysteme voneinander unabhängig. Im allgemeinen Fall sind sie verschränkt - es gibt dann nur einen Zustand für das Gesamtsystem. Da sich dieser eine Zustand nicht als Produkt von Zuständen der Teilsysteme schreiben lässt, sich also nicht weiter aufspalten lässt, bezeichnet man ihn - und damit das Gesamtsystem selbst - als nichtlokal oder nichtseparabel (nichttrennbar).

Es ist dann einfach sinnlos, von dem Zustand eines Teilsystems zu sprechen. Es ist dieser Aspekt der Nichtlokalität, der vielen Wissenschaftlern Unbehagen bereitet, zeigt er doch, dass das Ganze mehr ist als die Summe seiner Teile.

Wesentlich erscheint mir auch, was Zeilinger in seinem Buch [3] auf Seite 84 zum 'Wellenpaket' schreibt:

Hier sind wir bei einem sehr wichtigen Punkt angelangt, der immer wieder betont werden muss: Wir müssen jedem einzelnen Elektron ein Wellenpaket zuordnen. Das Elektron trägt daher nicht einen bestimmten Impuls, also eine wohl definierte Geschwindigkeit, und es ist nicht an einem wohldefinierten Ort lokalisiert.

Das Wellenpaket drückt sowohl die Unschärfe der Geschwindigkeit als auch die Unschärfe des Ortes aus. Wenn wir uns entscheiden, den Ort zu messen, werden wir das Elektron irgendwo innerhalb des Wellenpakets finden. Die Unschärfe des Ortes des Elektrons ist plötzlich sehr viel kleiner geworden.

Das Elektron, sagen die Physiker, ist durch die Messung lokalisiert worden. Das Experiment hat also nicht einen Ort ergeben, an dem sich das Elektron vor der Messung befand. Durch diese Messung hat das Elektron aber auch eine viel größere Impulsunschärfe erhalten, eine viel größere Unscharfe in seiner Geschwindigkeit.

Alles, was wir vorher hatten, war das Wellenpaket, das lediglich vorschreibt, wo das Elektron mit welcher Wahrscheinlichkeit gefunden werden könnte, mehr nicht.

Mit freundlichen Grüßen
Eugen Bauhof

[1] Kiefer, Claus
Quantentheorie. (http://www.science-shop.de/blatt/d_sci_sh_produkt&_knv_dok_nr=021150013)
Frankfurt am Main 2002.
ISBN=3-596-15356-5

[2] Kiefer, Claus
Der Quantenkosmos. (http://www.science-shop.de/artikel/973199)
Frankfurt am Main 2008.
ISBN=978-3-10-0395061

[3] Zeilinger, Anton
Einsteins Spuk. (http://www.science-shop.de/blatt/d_sci_sh_produkt&_knv_dok_nr=051370051)
Teleportation und anderer Mysterien der Quantenphysik.
München 2005
ISBN=978-3-570-00691-7
(In diesem Buch kann geblättert werden)

Knut Hacker
03.10.11, 19:41
Hallo richy,

zunächst allgemein:
Deine Warnungen vor dem AC waren berechtigt. Ich habe mich dort verabschiedet. Die dortigen Administratoren halten tatsächlich die Quantenphysik für Esoterik, obwohl sie ohne sie nicht vor dem Computer sitzen könnten.
Meine Vorfahren sind keine Juden, sondern, zurückverfolgbar bis auf einen gewissen Aquila, der Luther bei der Bibelübersetzung half, durch die Bank evangelische Pfarrer. Du wirst nun sarkastisch denken: Das erklärt einiges! Ich selbst bin jedoch areligiös und antiklerikal.

Doch nun zu deinen interessanten Fragen, wobei ich mir „Schmalspur – Quanten Physiker“ (siehe mein Profil) nur eine beschränkte Kompetenz beimesse ( Mich faszinieren an der Quantenphysik ebenso wie an den beiden Relativitätstheorien und der Chaosforschung vor allem die erkenntnistheoretischen Konsequenzen):



Ich wuerde dazu hier gerne noch naeher ergruenden, wann denn der Begriff Unschaerferelation und wann der Begriff Unbestimmtheitsrelation angebrachter ist.

Der Urheber Heisenberg scheint beide Begriffe synonym zu gebrauchen.

Er schreibt dazu ( „Wandlungen in den Grundlagen der Naturwissenschaft“ ):
„Wir können die Naturvorgänge im atomaren Bereich nicht mehr in der gleichen Weise ergreifen wie die Vorgänge im Großen. Wenn wir die gewohnten Begriffe verwenden, so wird ihre Anwendbarkeit durch die sogenannten „Unbestimmtheitsrelationen“ eingeschränkt. Für den weiteren Verlauf des atomaren Vorganges können wir in der Regel nur die Wahrscheinlichkeit voraussagen. Nicht mehr die objektiven Ereignisse,, sondern die Wahrscheinlichkeiten für das Eintreten gewisser Ereignisse können in mathematischen Formeln festgelegt werden. Nicht mehr das faktische Geschehen selbst, sondern die Möglichkeit zum Geschehen – die `Potentia´ , wenn wir diesen Begriff der Philosophie des Aristoteles verwenden wollen – ist strengen Naturgesetzen unterworfen.“
Das scheint dem Wortlaut nach eher die Schrödingerschen Wellengleichungen, also die Superposition,das Prinzip der Nichtlokalität zu charakterisieren, doch schreibt Heisenberg weiter:
„Aus den Unbestimmtheitsrelationen wissen wir, dass eine Ortsbestimmung einen umso schärferen Eingriffe erfordert, je genauer sie vorgenommen werden soll. Eine unendlich scharfe Ortsbestimmung würde sogar einen unendlich großen Eingriff voraussetzen und kann daher gar nicht realisiert werden.“
Damit ist eindeutig die Formel Δx (Unbestimmtheit des Ortes)Δp(Unbestimmtheit des Impulses)>h( Plancksche Konstante) gemeint.



Ich denke mal dass du einen guten und vor allem freien Ueberblick bezueglich der Kopenhagener Interpretation hast. Kannst du kurz skizzieren welche Rolle hier die Unschaerferelation im Rahmen der Interpretation einnimmt ?

Aus ihr wird in der Kopenhagener Deutung das von Bohr erdachte Komplementaritätsprinzip hergeleitet.

Heisenberg schreibt am angegebenen Ort: „... diese Physik uns gezwungen hat, in verschiedenen Zusammenhangsbereichen zu denken, die zueinander in jenem Verhältnis stehen, das Bohr mit dem Begriff „Komplementarität“ ausgedrückt hat. Die Zusammenhangsbereiche können sich einerseits ausschließen, andererseits aber doch auch ergänzen, so dass erst durch das Spiel zwischen den verschiedenen Bereichen die volle Einheit sichtbar wird. Wie das ohne die geringste Unklarheit möglich ist, zeigt die quantentheoretische Mathematik. Im Vergleich zur klassischen Physik rückt die Quantentheorie daher deutlich ab von jener etwas zu schroffen Zweiteilung der Welt in der Descartes´schen Philosophie.“( dort die strikte Trennung von Beobachter und Beobachtungsobjekt).

Einstein schließt sein zusammen mit Leopold Infeld herausgegebenen Buch „Die Evolution der Physik“ wie folgt:
„Die Quantenphysik bringt Gesetze, die für Kollektive und nicht mehr für deren Individuen gelten. Nicht Eigenschaften, sondern Wahrscheinlichkeiten werden beschrieben; nicht für die zukünftige Entwicklung von Systemen werden Gesetze aufgestellt, sondern für Veränderungen der Wahrscheinlichkeiten in der Zeit, Gesetze, die für große Ansammlungen von Individuen gelten.“

Das System ist also Beobachtungsobjekt (einschließlich des Beobachters = Messproblem).Das Detail bleibt daher unscharf, nur in Teilaspekten fassbar. Eine unübersehbare Parallele zur Chaosforschung, wo dem Teil keine abgrenzbare Bedeutung für das Ganze zukommt, sondern alles mit allem verbunden ist. So ist beispielsweise die Elektronenhülle eines Atoms stabil, trotz und gerade wegen der im einzelnen akausalen Energiezustandssprünge. Und es lassen sich Ort und Impuls eines Quantenobjekts nicht gleichzeitig mit beliebiger Schärfe bestimmen, doch beide getrennte Messungen komplementär zu einem Beobachtungsobjekt etwa in der Nebelkammer realisieren, ebenso wie der Dualismus Welle- Korpuskel der Erfassung beider Aspekte als nichtlokales System nicht entgegensteht oder die räumliche Trennung verschränkter Elementarteile deren Systemverbundenheit unberührt lässt.

EMI
03.10.11, 21:42
Deine Warnungen vor dem AC waren berechtigt. Ich habe mich dort verabschiedet.
Die dortigen Administratoren halten tatsächlich die Quantenphysik für Esoterik, obwohl sie ohne sie nicht vor dem Computer sitzen könnten.Das geht zu weit Knut,

mag ja sein, dass Du das so einschätzt, hier in diesem Forum zieht NIEMAND über ein Nachbarforum her!

Klär das dort bitte Knut und nicht über uns/mit uns hier!

Gruß EMI

PS: Dieses Nachbarforum ist hochqualifizierrt und hat hervoragende Moderatoren!

fossilium
03.10.11, 23:39
Hi Knut,

kann es sein, dass all die Probleme mit der Quantenmechanik (Unstetigkeiten, Unschärfe, Realitätsfrage, Nichtlokalität, Dualismus usw.) Scheinprobleme sind ?

Was für Probleme präsentierst Du uns denn da eigentlich ? Lass uns mal darüber sprechen:

Wir Physiker beobachten die Natur und erarbeiten ein Modell, mit dem wir Gesetzmäßigkeiten und Zusammenhänge in der Natur aufzeigen und Vorhersagen über Ereignisse machen können. Die Güte des Modells wird an Hand von Beobachtungen immer wieder überprüft und verfeinert. Unser Ziel ist es doch in erster Linie, ein möglichst realistisches M o d e l l der Natur nachzubilden, und nicht, die Natur zu erkennen, wie sie ist. Das ist das Ziel der Metaphysiker, nicht der Physiker.

Natürlich ist die Versuchung gross, von gut funktionierenden Modellen auf das Wesen der Natur zurückzuschliessen. Wenn die Vorhersagen über zukünftige Ereignisse richtig und die Erklärungen über Vergangenes nachvollziehbar sind, dann liegt es ja auf der Hand, dass die Natur sehr ä h n l i c h wie unser Modell strukturiert sein muss. Man gerät aber leicht in die Gefahr, die Grenzen zwischen Natur und Modell zu verwischen, Realität und Modell gar gleichzusetzen.

Nur wer diese Gleichsetzung bedenkenlos vollzieht, kriegt mit den quantenmechanischen Modellen Probleme:

Wenn in unserem Atom m o d e l l gebundene Elektronen nur diskrete Energien haben, dann hat man natürlich ein Problem, wenn man daraus schliesst, die Natur per se würde unerklärliche Sprünge machen. Wenn sich in einem unserer Modelle gemeinsame exakte Messwerte bei bestimmten Grössen ausschliessen, so hat man ein Problem, wenn man daraus schliesst, dass sich hier die Grenzen der menschlichen Erkenntnis wiederspiegeln. Und wenn wir im Modell eine für Messvorhersagen erfolgreiche Gleichung benutzen, von der das Modell offen lässt, was diese Gleichung konkret beschreibt, so hat man natürlich ein Problem, wenn man daraus schliesst, die Natur existiere in geheimnisvollen Zuständen, oder spalte sich in 10 hoch 100 separate Universen auf. Selbst der Zusammenbruch der Realität, Kausalität oder Lokalität im Mikrokosmos kann man so beschwören, auch wenn unser Modell lediglich behauptet, dass hier unter speziellen experimentellen Bedingungen zwei Teilchen ihre Identität verlieren und eine Wirkung wie ein einzelnes ausüben.

Erst wenn wir so tun, als ob das, was in unseren Modellen steht, auch in der Wirklichkeit stattfindet, dann erst fliegen uns all die Probleme um die Ohren, von denen Du sprichst.

Ist es nicht vielmehr anzuraten, bei diesen Extrapolationen vom Modell auf die Wirklichkeit einfach mehr auf dem Teppich zu bleiben ? Alles was in unseren Modellen vorhergesagt ist doch erstmal vorläufig, unsicher und nicht wesensbestimmend für irgendeine Wirklichkeit. Weitreichende Schlüsse über real Existierendes sind nur mit grösster Vorsicht zu ziehen ! Denn unsere Modelle sagen nichts über das Wesen der Dinge aus - sie spiegeln nur einen kleinen Ausschnitt aus der Natur wieder, und sind zudem mit subjektiv und sozial begründeten Vorurteilen verquickt.

Und sind die Probleme mit der Quantenmechnik nicht auch deshalb Scheinprobleme, weil
wir sie verabsolutieren ?

Bezüglich der Unschärfe hatte ich ja schon mal behauptet, dass damit keine neue Weltsicht begründet werden kann, da wir diese aus dem Alltag schon gut kennen: bei verschiedenen Perspektiven auf eine Sache bleiben notwendigerweise immer Aspekte aussen vor, das gehört zum Wesen des Bewusstmachens von Wirklichkeit, und ist eben kein epistemisches, sondern doch ein ontologisches Problem. Und sind uns Dualismen und Komplementarilät nicht so gut bekannt, das wir sie gar nicht mehr wahrnehmen. Was ist so neu an einem fehlenden Determinismus ? Treffen wir nicht ständig Entscheidungen auf Grund von Wahrscheinlichkeitsüberlegungen ? Was wundert uns so an der Nicht-Lokalität (schreckliches Wort) ? Sollten wir nicht einmal überlegen, ob der Begriff der Identität physikalisch zu erweitern ist, statt starr vor Erstaunen über die experimentellen Ergebnisse alle grundlegenden Denkkategorien wie Logik, Kausalität, Realität gleich über den Haufen zu schmeissen ?

Warum also sollten uns die Probleme mit der Quantenmechanik schrecken ?

Die Erfahrung ist doch: man überwindet die Probleme am besten, in dem man sie erstmal am besten ignoriert. Wenn ich daher hier die Probleme der Quantenmechanik als Scheinprobleme deklassiere, so mag das hier ein bisschen wie mit der Brechstange aussehen, aber wie sonst sollen wir die Probleme angehen ? Wir können doch nicht vor lauter Schreck, dass unser Denken notwendigerweise zu Paradoxien führt, dieses einstellen. Das brächte uns in ein noch grösseres Dilemma, da unser Verstand sofort dagegen revoltieren würde. Kein Geringerer als Einstein hat es uns doch vorgemacht: um die spezielle Relativitätstheorie zu erfinden, hat erstmal den Äther zum Scheinproblem erklärt ! Wir müssen einfach sagen: „wo ist das Problem ?“ und weitermachen – ich meine das wäre die richtige Einstellung. Also negieren wir am besten alles was Du uns erzählst !

Das ist natürlich nicht ernst gemeint. Erstens finde ich Deinen Beitrag wunderbar, und zweitens ist genauso zulässig, zu behaupten, unsere Modelle würden trotz allem eine Näherung an die Natur, wie sie ist, darstellen, so als sei die Physik ein Mittel der Wahrheitsfindung an sich.

Aber dann kriegen wir genau die Probleme, die Du hier in Deinem Beitrag aufführst.

Also bleiben wir lieber dabei: die Probleme gibt es nur in unseren Modellen, nicht in der Wirklichkeit.

Grüsse Fosslium

richy
03.10.11, 23:50
Hi Knut
Was ist denn Realität? Doch dasjenige, was uns unser Bewusstsein sagt. Das gehoert dazu. Aber wenn man den Raum erweitert kann man die physikalische Realitaet auch quantitativ erfassen. So wie bei der Zeit. Und dann entsteht auch mehr Raum fuer eine uebergeordnete Beschreibung. So wird auch der Zufall in solchen realistischen Systemen gedeutet.
Wir können schließlich nicht aus diesem heraustreten.Also befinden wir uns in einem System, so dass die Gödelschen Gesetze gelten, dass sich kein System aus sich heraus beweisen kann. Ja, das koennte man schon so sagen, aber in der Physik wuerde ich das nicht als primaeres Problem betrachten. Wenn du hiermit auf die von dir angesprochene Problematik des Existenzbergriffes bei Raum und Zeit anspielen moechtest. So weit kommst du im Rahmen der Kopenhagener Interpretation physikalisch gar nicht. Da ist aktuell schon bei den C70 Fuelleren quasi physikalische "Endstation". Ich wuerde diese Selbstbezueglichkeit vielleicht eher als Rueckkopplung betrachten, dann ist es insbesonders bei nichtlinearen diskreten Systemen tatsaechlich eine reale Problematik, die auch nach Loesungen sucht. Und frueher waren das alles Gebiete der Chaostheorie.
Das beginnt doch schon mit der Kindesfrage, was "rot" ist. Dieser Sinneseindruck hat mit dem Wellenspektrum des Lichtes nicht das Geringste zu tun.Naja wir nehmen hier Spektren wahr und damit genauso wie im Gehoer eine Art Fouriertransformierte. Klar, ganz kann man diese Vorgaenge bis heute nicht erklaeren.
Die sogenannten Realisten verabsolutieren den Verstand (philosophisch zu unterscheiden von der Vernunft), obwohl alles Denken letztlich selbstbezüglich ist, also zum unstrittigen Münchhausen Trilemma führt:Hier gibt es wohl unterschiedliche Stroemungen. Auch ein Realist wird kaum immaterielle Dinge voellig ausklammern. Alleine Eigenschaften und Beschreibungen stellen schon solches dar.Und ein hochdimensionales Uhrwerkuniversum moechte ich mir nicht vorstellen. Diese Textstelle von Penrose zum freien Willen koennte fuer dich interessant sein :
http://www.quanten.de/forum/showpost.php5?p=61395&postcount=78
Das deckt sich recht gut mit deinen Aspekten. Und eine Nichtberechnbarkeit kann man direkt unter dem Stichwort La Placescher Daemeon erhalten :

A) Das Dreikörperproblem (vor 1888)
B) Die Relativitätstheorie (1905)
C) Die Unschärferelation auf Quantenebene (um 1925)
D) Berechnungsgrenzen, Beschreibung seiner selbst (1960er)

Gruesse

EMI
03.10.11, 23:53
...die Probleme gibt es nur in unseren Modellen, nicht in der Wirklichkeit.So sehe ich es auch fossi,

selten so einen tiefgründigen Beitrag gelesen. Danke!

Gruß EMI

richy
04.10.11, 00:36
Teil 2

Wenn man diese ganzen Faelle durchspielt ergeben sich natuerlich Wort und Logikkonstrukte, die man z.B im AC Forum besser nicht verwenden sollte (Nur ein Tipp and dich Knut :-)

Wort-und Logikkonstrukte kann man doch nicht einfach abtun, sondern sie sind Wesenszüge unseres Denkens.Die gesamte Philosophie und die Kognitionswissenschaften befassen sich ausschließlich mit Paradoxien.
Ich meinte nur, dass dies meist recht anstrengend zu lesen und zu durchdenken ist. Mir machen solche Paradoxa auch Spass. Aber nicht immer :-)
Es kommt auch nicht darauf an, ob Antinomien - wie sie zum Beispiel Kant ausführlich behandelt hat - befriedigend sind.Das ist doch ein Einwand a la Christian Morgenstern:"... dass nicht sein kann, was nicht sein darf!"
Man kann das Problem ohne Antimonie loesen. Also warum sollte man dies nicht in Betracht ziehen ? Und man erhalt zusaetzlich die Erkenntnis, dass es dann verschiedene Formen von Existenz geben muss. Und diese Existenzueberlegungen scheinen in der LQG eine Rolle zu spielen. Also durchaus gerechtfertigte Ueberlegungen, wobei meinen "Gitterling" anscheinend niemand hier begeistert :-) Warum die LQG zwei duale Strukturen verwendet (Vornoi-Delauny) waere noch interessant zu erfahren.

Zur Chaosforschung.
Die gibt es leider nicht mehr in dieser Form. Sie hat sich leider in einzelne Teilbereiche aufgeloest die teiweise eine grosse Rolle spielen wie neuronale Netzwerke (Sprach/Schrifterkennung) aber von denen man nicht mehr so viel hoert wie Ende der 80 er. Durch die Aufteilung sind wohl leider die philosopischen Aspekte stark reduziert worden. Man merkt allgemein, dass das Interesse und damit Verstaendnis nichtlinearer Systeme geschwunden ist. Abgesehen von den Spezialisten. Schade, denn die Welt ist nunmal nichtlinear.

Gruesse

richy
04.10.11, 00:44
Die Erfahrung ist doch: man überwindet die Probleme am besten, in dem man sie erstmal am besten ignoriert.
Das hab ich bei Klassenarbeiten in Geschichte auch oft getan und die Lehrein hat dann eine 5 druntergeschrieben.

... und in welchen Wohlgefallen soll sich dann die Verschraenkung aufloesen ?
Rueckkehr des einfachen Realismus ? Der ist vom Tisch.
Eine andere Frage ist natuerlcih ob die Menschheit z.B. einen Quantenrechner benoetigt. Ich meine fast man sollte darauf verzichten.

amc
04.10.11, 00:46
Hi Richy,

vermutlich bin ich nicht in der Lage wirklich sinnvolle Spekulationen zu deinen Fragen anzustellen. Meine Kenntnisse sind zwar nicht die allerschlechtesten, insgesamt aber doch recht dürftig. Ich kann einfach die Gedanken nehmen, die mir zu deinen Fragen mehr oder weniger spontan durch den Kopf gehen, die natürlich auch in völlig falsche Richtungen gehen können. Falls sie ein wenig vermittelnd wirken ist das positiv für mich, zuviel erwarten kann man dabei sicher nicht, weil ich dafür einfach zuwenig weiß, und die Dinge oft nicht richtig verstehe und einschätzen kann.

- Wie verstehst du hier die Begriffe Existenz und Nichtexistenz ?

Ich habe es so verstanden, dass hier in erster Linie konkrete Eigenschaften des Quantenobjektes gemeint sind (Impuls,Ort), welche man nicht zu jederzeit als existent betrachten darf. Es wird im Grunde vermieden über die Existenz oder Nichtexistenz des Teilchens an sich zwischen den Ereignissen konkrete Aussagen zu machen, weil dies auch ohne Messung schlecht möglich ist.

Wikipedia - Kopenhagener Deutung (http://de.wikipedia.org/wiki/Kopenhagener_Deutung)
In welcher Form oder wo ein Teilchen zwischen zwei Messungen existiert, macht die Quantenmechanik nach der Kopenhagener Deutung keine Aussage.

Allerdings:

Die Kopenhagener Deutung in ihrer ursprünglichen Version von Niels Bohr verneint nun die Existenz jeglicher Beziehung zwischen den Objekten des quantentheoretischen Formalismus einerseits und der „realen Welt“ andererseits, die über dessen Fähigkeit zur Voraussage von Wahrscheinlichkeiten von Messergebnissen hinaus geht. Einzig den durch die Theorie vorhergesagten Messwerten, und damit klassischen Begriffen, wird eine unmittelbare Realität zugewiesen. In diesem Sinne ist die Quantenmechanik eine nichtreale Theorie.

Ich denke dies hier:

Die mit dem Begriff „Teilchen“ nach Maßstäben unserer Alltagserfahrung verknüpfte Vorstellung, diese Portion müsse sich in jedem Moment an einem bestimmten Ort befinden und damit permanent als Teilchen Bestandteil der Realität sein, ist hingegen experimentell nicht gedeckt und führt im Gegenteil zu Widersprüchen mit den empirischen Messergebnissen. Diese Vorstellung ist in der Kopenhagener Deutung aufgegeben.

... ist sehr vernünftig und steht kaum zur Diskussion, oder? Die entscheidende Frage ist (IMHO), ob diese, ich nenne es mal Wellenrealität, auch eine tatsächliche, sich allerdings zu der bisher bekannten klassischen raumzeitlichen Realität andersartig darstellende Realität ist? Aber als solche auch als existent zu betrachten ist. Nur wäre sie eben nicht Teil unserer Realität im herkömmlichen Sinne, daher sie besäße keine Zeit und keinen Ort, wäre aber unverszichtbarer Teil unserer Wirklichkeit (hier bewusst Wirklichkeit und nicht Realität). Sie könnte sich so als real existierendes verbindendes Element (zwischen klassischen Ereignissen) in unserer Wirklichkeit darstellen. Ich halte dies für durchaus plausibel.

- Ist speziell eine physikalische Existenz gemeint ?

Wenn mit physikalischer Existenz klassische raumzeitliche Ereignisse gemeint sind, dann sind wohl diese speziell gemeint, über mehr versucht die Kopenhagener Deutung, wie ich sie verstehe, keine Aussage zu machen:

Was beobachtet worden ist, existiert gewiss; bezüglich dessen, was nicht beobachtet worden ist, haben wir jedoch die Freiheit, Annahmen über dessen Existenz oder Nichtexistenz einzuführen.

Es wird bewusst versucht es möglichst offen zu lassen ...

Ich denke vorallem ist aber Existenz an sich gemeint. Warum zwischen Existenzen unterscheiden? Wenn etwas existiert dann existiert es, und wenn nicht dann nicht. Vielleicht sollte man sich anders nähern: Alles ist eine Existenz, und was sich in ihr befindet, sie ausmacht, muss grundsätzlich physikalisch beschreibbar sein, und sich auch bemerbar machen, um als existent gelten zu können.

- Wie koennte man eine physikalische Nichtexistenz deuten ?

Wie schon oben erwähnt, halte ich eine Unterscheidung für nicht sinnvoll (nur gedacht). Allerdings, und das ist IMHO die Frage, gibt es eine vom raumzeitlichen Ereignis losgelöste, oder besser - vorgeschaltete Existenz? Im klassischen Sinn mag diese vielleicht als nicht physikalisch erscheinen, ich denke aber wenn sie als existent zu betrachten ist, dann muss sie auch zu einem gewissen Grad physikalisch beschreibbar sein.

- Waere der Waegbarkeitsbegriff (incl. Eigenschaften bezogen auf Waegbares und Unwaegbares) fuer eine Differenzierung geeignet ?

Wie gesagt, denke ich, dass jede Art von Existenz sich auf irgendeine Weise bemerkbar machen muss. Man müsste also, wenn die Wellenrealität existent ist, diese auf irgendeine Art bemessen können. Mir ist klar, das dies im Grunde ein Widerspruch in sich ist.

- Welche Masse hat ein C70 Molekuel im ueberlagerten Zustand ?

Genau, ich denke wenn dieser Zustand als physikalisch existent anzusehen ist, so muss dieser auch Masse besitzen und möglicherweise gravitativ sein.

- Ist Masse eine abstrakte Eigenschaft ?

Nach allem was ich weiß, was nicht wirklich viel ist, sehe ich Masse als elementarste Eigenschaft an. Alles was existiert besitzt auch Masse. Aber manches besitzt nicht viel mehr als Masse (Dunkle Materie/Energie), vielleicht verhält es sich mit dieser Wellenrealität ähnlich. Nur ne Spekulation ...


Freundlichst,
AMC

amc
04.10.11, 11:29
... wobei meinen "Gitterling" anscheinend niemand hier begeistert :-) Warum die LQG zwei duale Strukturen verwendet (Vornoi-Delauny) waere noch interessant zu erfahren.

Der Gitterling hat schon was faszinierendes an sich. Anmerkung, damit es gefunden werden kann: Voronoi und Delaunay muss es heißen.


Freundlichst,
AMC

Harti
04.10.11, 13:49
Die Erfahrung ist doch: man überwindet die Probleme am besten, in dem man sie erstmal am besten ignoriert.


Hallo Fossilium,
ohne diesen Satz hätte ich Dir ja voll zustimmen können. Ich würde insoweit beim Auftreten eines Problems (Widerspruchs, Unerklärlichkeit etc.) zunächst fragen: Wo genau liegt das Problem ? Ist es tatsächlich eins ?

Der Unterschied zwischen Denken (Vorstellung, Theorie) und Wirklichkeit wird häufig nicht gemacht und führt zu Widersprüchen. Zur Unschärferelation könnte man z.B. schon darauf hinweisen, dass ein 100% genauer Ort in der Realität nicht existiert, weil es einen nulldimensionalen Punkt nur in der Theorie (z.B. zwei Koordinaten in einem zweidimensionalen Koordinatensystem) gibt und ein solcher Ort aufgrund der Planck-Größen auch tatsächlich nicht feststellbar ist. Ein Ort ist deshalb nur in der Theorie genau anzugeben, in der Realität ist er eigentlich nur "verschmiert" vorstellbar.

MfG
Harti

Knut Hacker
04.10.11, 14:15
Was für Probleme präsentierst Du uns denn da eigentlich ? Lass uns mal darüber sprechen:




Hallo Fossilium,

ich scheine mich sehr missverständlich ausgedrückt zu haben.Ich habe mit der Quantenphysik keinerlei Probleme und habe davor gewarnt, philosophische Konsequenzen aus ihren Erkenntnissen zu ziehen, weil diese das Selbstverständnis der Naturwissenschaften außer Betracht lassen.

So habe ich doch erst kürzlich hier geschrieben:

Ein physikalischer Vorgang ist nur dann und insoweit von Bedeutung für die Physik als Erfahrungswissenschaft,als er dem Bewusstsein durch Messung zugänglich gemacht wird. Niels Bohr:" Es ist falsch anzunehmen, die Aufgabe der Physik bestehe darin, herauszufinden, wie die Natur beschaffen ist. Die Physik beschäftigt sich vielmehr damit, was wir über die Natur aussagen können."

Ich sehe also überhaupt keine Differenzen zwischen unseren Ausführungen.

richy
04.10.11, 15:46
Hi Marco
Die stammten dann wohl aus Savoyen? Ja genau. Torre Pellice und Pinasca bzw Perosa Argentina. In Pinache (D) bin ich den Stammbaum uebrigends durch und nun vor 1699, also Savoyen gelandet. Die sind 1699 uebrigends von Basel aus den Rhein hochgeschippert bis Schroeck. Das heisst heute Leopoldshafen und das Kernforschungzentrum dort ist recht bekannt. Und dort gibt es einen alten abgelegenen Altrhein-Hafen. Den kenn ich sehr gut.
Dort sind die angekommen. Kann man sich kaum mehr vorstellen.
Dank einer Datenbank kann ich hier sogar Einzelschicksale rekonstruieren. Sehr interessante Sache. Der Hammer ist, dass sich seit 1200 auch in Italien die Nachkommen nicht von diesen Taelern, nichtmal den Ortschaften entfernt haben. Ausser nach Argentinien. Man kann das an der besonderen Schreibweise der italienischen Form des Namens statistisch recht sicher bestimmen. Einige findet man sogar bei facebook. Die sind alle aus Savoyen und beherrschen sogar noch okziantianisch .
Gruesse

Knut Hacker
04.10.11, 17:03
Zur Unschärferelation könnte man z.B. schon darauf hinweisen, dass ein 100% genauer Ort in der Realität nicht existiert, weil es einen nulldimensionalen Punkt nur in der Theorie (z.B. zwei Koordinaten in einem zweidimensionalen Koordinatensystem) gibt und ein solcher Ort aufgrund der Planck-Größen auch tatsächlich nicht feststellbar ist. Ein Ort ist deshalb nur in der Theorie genau anzugeben, in der Realität ist er eigentlich nur "verschmiert" vorstellbar.


Hallo Harti,

ich möchte noch ergänzen, dass dies auch für Zeitpunkte gilt, insbesondere für die Gegenwart.

Diese Nulldimensionaliät ist möglicherweise der Schlüssel dafür, dass die Unschärferelation nur im Quantenbereich "gilt" Im makrokosmischen Bereich ist sie einfach vernachlässigbar und klassisch durch das Wesen des Kontinuums (Raum-Zeit-Kontinuum) erklärbar.Schon Aristoteles hatte das Kontinuum dahin definiert, dass es unbegrenzt in Teilbares geteilt werden kann:"λέγω δὲ συνεχὲς τὸ διαιρετὸν εἰς αἰεὶ διαιρετά " (Physik, Z.2, 232b24-25)

Knut Hacker
04.10.11, 17:14
Aber doch nicht hier Knut,

was haben wir denn mit dem AC-Forum zu tun?
Antwort: NIX!

Gruß EMI

Ich meinte mit "Gegendarstellung" doch lediglich meine nun von mir wieder gelöschte Erwiderung auf deinen Vorwurf.
(Eine Gegendarstellung auf meine Betitelung durch die AC - Administation als "......." wäre schon rechtlich nicht möglich, da es sich dabei um eine sogenannte Formalbeleidigung, nicht um eine Tatsachenbehauptung handelt)

Können wir nun dieses Thema begraben?

Knut Hacker
05.10.11, 14:21
Die Nichtlokalität ist eine intrinsische Naturtatsache, die sich im Quantenprozess zeigt. Die Quantenmechanik ist nicht inhärent nichtlokal. Aber sie zeigt nichtlokales Verhalten, wenn sie aus klassischer Sicht untersucht wird.


Sehr richtig! Carl Friedrich von Weizsäcker schreibt dazu in "Der Garten des Menschlichen" im Kapitel "Heidegger und die Naturwissenschaft":

"Die allgemeine oder abstrakte Quantentheorie kann man als eine nichtklassische Wahrscheinlichkeitsrechnung beschreiben.Das Wort "nichtklassisch" bezeichnet hier formal eine der Wahrscheinlichkeitsbewertung zugrundegelegte Mannigfaltigkeit möglicher Ereignisse, die von derjenigen abweicht, die man durch Anwendung der klassischen Aussagenlogik (Boolesche Algebra) herleiten würde. Man spricht gelegentlich von einer "Quantenlogik".Der Grund dieser nichtklassischen Logik liegt in einer, freilich von den Physikern nicht ausgearbeiteten, sondern nur verbal umschriebenen Abweichung von der klassischen Ontologie.Man kann in der Quantentheorie ein mögliches Ereignis nur noch in Bezug auf einen möglichen Beobachter definieren. Die Subjekt-Objekt-Beziehung wird hier, zum erstenmal in der neuzeitlichen Physik thematisch."

amc
05.10.11, 14:44
Hallo Fossilium,

hier würde ich gerne nochmal einhaken:

Wenn in unserem Atom m o d e l l gebundene Elektronen nur diskrete Energien haben, dann hat man natürlich ein Problem, wenn man daraus schliesst, die Natur per se würde unerklärliche Sprünge machen.

Ein Problem hatte man (und hat teilweise noch) vorallem mit den Modellen, die annehmen, die Natur würde sich im kleinen, genau wie im großen, stetig und kontinuierlich verhalten. Dann v e r k l u m p e n (dieses Wort ist zensiert :confused: darum die Abstände) nämlich die Atome wegen der EM-Kraft, UV-Licht wäre unendlich energiereich, Quantenfluktuationen kollabieren zu Schwarzen Löcher, auf einen Punkt konzentriert sich unendlich viel Materie/Energie ... gibt sicher noch viele andere Beispiele.

Wenn sich in einem unserer Modelle gemeinsame exakte Messwerte bei bestimmten Grössen ausschliessen, so hat man ein Problem, wenn man daraus schliesst, dass sich hier die Grenzen der menschlichen Erkenntnis wiederspiegeln.

Hier liegen nicht die Grenzen der menschlichen Erkenntnis, sondern die grenzen der Bestimmtheit prinzipiell. Es liegt nicht an dem möglicherweise unzureichend entwickelnten physikalischen Verständnis der Natur. Zumindest scheint dies mit großer Wahrscheinlichkeit so zu sein. Anders macht die Welt keinen Sinn, anders wäre sie nicht möglich.

Und wenn wir im Modell eine für Messvorhersagen erfolgreiche Gleichung benutzen, von der das Modell offen lässt, was diese Gleichung konkret beschreibt, so hat man natürlich ein Problem, wenn man daraus schliesst, die Natur existiere in geheimnisvollen Zuständen, oder spalte sich in 10 hoch 100 separate Universen auf.

Irendwas muss aber sein - entweder macht die Welt Riesensprünge, ohne Verbindung, ohne jedlichste Ursache, oder sie existiert in bisher unergründeten geheimnisvollen Zuständen, oder sie spaltet sich auf in separate Universen, oder die Natur reitet auf einer Führungswelle (de Broglie-Bohm), oder oder ... da gibts noch viele andere Interpretationen. Aber irgndwas davon muss sein!

Aber dann kriegen wir genau die Probleme, die Du hier in Deinem Beitrag aufführst.

Ich will hier auch mal eine Lanze brechen für die Quantenphysik. Sie hat doch mehr Probleme gelöst als geschaffen. Durch sie beginnen wir allmählich die Vorgänge in unserer Welt tatsächlich verstehen zu können. Mit auf Quantenphysik basierenden Waren wird heutzutage ein Drittel der Weltwirtschaft betrieben.

Probleme bekommen mit der Quantenphysik nur Menschen, die an alten Denkegebäuden festhalten. Wobei ich auch überzeugt bin, dass wir noch lange nicht am Ende sind beim Verständnis der Quantenwelt, da gibst noch ne Menge was nicht vertanden ist.

... die Probleme gibt es nur in unseren Modellen, nicht in der Wirklichkeit.

Hier natürlich uneingeschränkte Zustimmung, die Wirklichkeit kennt keine Probleme. :)


Freundlichst,
AMC

richy
05.10.11, 17:43
Hi Bauhof
Vielen Dank fuer die Zitate.
die Nichtlokalität der Quantenmechanik lässt sich meines Wissens nicht mit der Unbestimmtheitsrelation erklären.Ich meine auch, dass dies nicht direkt moeglich ist. Die Unbestimmtheitsrelation soll lediglich die dahinterstehenden Grundgedanken rechtfertigen. Wuerde ein Physiker mit diesem Konzept, diesen Grundgedanken, Philosophie die Verschraenkung konkret erklaeren, wuerde man ihn sicherlich nicht ernst nehmen. Und das ist gelinde ausgedrueckt. Wobei es fuer Zeilinger durchaus moeglich ware. Er muesste lediglich einen virtuellen, abstrakten, informativen (das kann man sich aussuchen) Konfigurationsraum einfuehren.
Ich habe jedenfalls darüber in der Literatur nichts gefunden, wo solches behauptet würde. (Und was ist mit dem letzten Zeilinger Zitat ?) In einem Vortrag wurde Lesch wie erwaeht dazu aufgefordert die Verschraenkung zu deuten. Wohl unvorbereitet. Er hat dann Heisenberg erklaert und anhand der youtube Kommentare kann man erkennen, dass einige Menschen damit nicht zufrieden waren. Weil dies nur eine Philosophie erklaert aber keine Verschraenkung.
Die Nichtlokalität ist eine intrinsische Naturtatsache, die sich im Quantenprozess zeigt.
Weil es eine Tatsache ist kann man diese auch nicht vom Tisch reden. Und das Wort "innewohnend" wird meist dann vewendet wenn man eigentlich nichts genaues weiss. So wohnt auch die Welle dem Teilchen intrinsisch inne und umgekehrt. Klingelt man an der Haustuer, macht stets das Teilchen auf :-)
Falls ich Claus Kiefer richtig verstanden habe, zeigt sich die Nichtlokalität der Quantentheorie durch die Verschränkung mit der Umgebung.
Ich hoffe du hast im Gegensatz zu Zippel verstanden, dass Kiefer den Begriff Verschraenkung fuer einen hochdimensionalen Raum annimmt. Ich meine man haette hier zwei verschiedene Begriffe einfuehren sollen. Aus der Sicht im Schuhkarton wird die Verschraenkung naemlich durch die Dekohaerenz aufgeloest. Man sollte daher immer betrachten wer denn hier eine Aussage trifft.

Kiefer studierte Physik und Astronomie an der Ruprecht-Karls-Universität Heidelberg und der Universität Wien. 1988 promovierte er bei Dieter Zeh in Heidelberg über den Zeitbegriff in der Quantengravitation

Kiefer vertritt somit einen Realismus, die viele Welten wie es auch aus seinem Zitat recht deutlich hervorgeht. Dekohaerenz ist nunmal ein realistisches Konzept.
Lokale klassische Eigenschaften werden also durch die Nichtlokalität der Quantentheorie (Verschränkung mit der Umgebung) bewirkt.Also ganz ohne Quantengeister. Zeilinger duerfte solche Aussagen nicht treffen und folgende noch weniger :
diese klassischen Eigenschaften wohnen dem Objekt nicht inne, sie werden erst durch die Wechselwirkung mit der Umgebung definiert.

wohnen dem Objekt inne = intrinsisch
Eine physikalische Wechselwirkung widerspricht der Kopenhagener Deutung und ist nur bei einem Viele Welten Modell zulaessig. Nur das ist neben Bohm physikalisch realistisch.
Wie du freundlicherweise dargestellt hast ist eine Deutung der Verschraenkung mittels einem realistischen Modell sehr wohl moeglich. Dazu voellig konsistent ohne Quantengeister. Die Erklaerung gibt es aber nicht umsonst. Und wer nicht bezahlt geht nunmal leer aus.

Es ist dann einfach sinnlos, von dem Zustand eines Teilsystems zu sprechen. Das bedeutet: Es ist sinnlos einen Schuhkarton anzunehmen.
Wir müssen jedem einzelnen Elektron ein Wellenpaket zuordnen. Zuordnen=Intrinsinisch. Im Grunde ein Unwort.
Und was beschreibt Zeilinger dann ? Heisenberg. Dass es Wellen gibt. Dass man bei einer Welle eine Frequenz angeben kann ODER eine Periodendauer.
Das Pinzip der Fouriertransformtion. Das schliesst aber weder die Existenz eine Gitarre oder eines Klavieres aus.

Alles, was wir vorher hatten, war das Wellenpaket, das lediglich vorschreibt, wo das Elektron mit welcher Wahrscheinlichkeit gefunden werden könnte, mehr nicht.
Und das ist nun mal gegenueber Kiefer bischen wenig nicht ? Dazu noch inkonsistent. Aber Zeilinger ist auf dem richtigen weg meine ich. Ein Jammer dass da so viele unnoetige Steine liegen.

Viele Gruesse

richy
05.10.11, 23:56
Hi fossilium

kann es sein, dass all die Probleme mit der Quantenmechanik (Unstetigkeiten, Unschärfe, Realitätsfrage, Nichtlokalität, Dualismus usw.) Scheinprobleme sind ? Das Problem der QM duerfte jenes sein, dass die Experimente vor allem zeigen welche Denkmodelle auf jeden Fall verworfen werden muessen. Zum Beispiel der einfache Realismus. Und das ist kein Scheinproblem. Ebensowenig der goedelsche Unvollstaendigkeitssatz.

Es gibt Scheinloesungen.
Wenn ich in der Bohmschen Mechanik z.B. den Konfigurationsraum einfach ignoriere, dann ist das eine Scheinloesung. Und damit ueberhaupt keine Loesung der eigentlichen Problematik. Ein sogar widerspruechliche Scheinloesung ist es eine Loesung abzulehnen, z.B. einen Realismus um sich dann auf ein Konzept genau dieser Loesung zu berufen, die Dekohaerenz.
Ich lehne etwas ab und gleichzeitig befuerworte ich es.

Stoeren mich vielen Welten, dann vertrete ich Kopenhagen.
Werde ich auf deren Quantenmystik aufmerksam gemacht, dann behaupte ich, ich bin Realist.
Soll ich etwas erklaeren benutze ich Viele Welten ohne sie zu erwaehnen.
Werde ich auf Quantenmystik und viele Welten angesprochen behaupte ich keine Erklaerung waere die Erklaerung.
Was soll das ?

Man gerät aber leicht in die Gefahr, die Grenzen zwischen Natur und Modell zu verwischen, Realität und Modell gar gleichzusetzen.
Ja, das kann vorkommen. Sogar gerade bei denjenigen, die sich tagtaeglich mit solchen Modellen praktisch am Schreibtisch befassen.
Wenn du einen E-Ing fraegst was eine EM-Welle ist, dann hat dieser die Maxwellgleichungen vor Augen, Wellengleichungen, irgendwelche abstrakten Gebilde, die er mit der Zeit immer mehr vergegenstaendlicht. Weil dies die Gegenstaende sind, mit denen er am Schreibtisch die Probleme loest. Ein Radartechniker wird eine andere Vorstellung einer EM Welle haben. Dennoch ist es keine grosse Kunst zwischen Modell und Beschriebenem zu unterscheiden. In manchen Faellen vielleicht nur noch intuitiv. Warum intuitiv ? Weil eine Beschreibung einer Beschreibung gar nicht so einfach ist.(siehe Knut) Aber anhand von Beispielen geht das immer.

Viele hier haben damit keinerlei Probleme. z.b: Hermes Gandalf .. Daher wundert es mich besonders, dass andere es einfach hinnehmen, wenn eine Interpretation verlangt, dass man die Realitaet einfach entfernt, so dass nur noch das Modell existieren soll. Gerade derjenige, der auf eine strikte Trennung achtet wird dies zunaechst in keinster Weise akzeptieren.

Nur wer diese Gleichsetzung bedenkenlos vollzieht, kriegt mit den quantenmechanischen Modellen Probleme:
Gerade derjenige bekommt die wenigsten (KI) Probleme. Denn der merkt gar nicht, dass man ihm die Realitaet entzogen hat, weil er denkt diese sei irgendwie in dem Modell (der SGL) noch enthalten. Zur Not rettet er sich in die Beschreibung nach der Messung. Da ist mit der Wellenkollapsvorstellung die SGL gar nicht mehr da, sondern entartet zu einer Delta (Dirac) Funktion. Der sieht gar nicht, dass die SGL nur ein Modell ist.
Und wenn wir im Modell eine für Messvorhersagen erfolgreiche Gleichung benutzen, von der das Modell offen lässt, was diese Gleichung konkret beschreibt ...
Ja ich hoffe doch das Modell beschreibt die Messvorhersagen, die Natur.:D
Selbst der Zusammenbruch der Realität, Kausalität oder Lokalität im Mikrokosmos kann man so beschwören ...
Das wird nicht beschworen, sondern ist Fact. Und die SGL ist Fact, denn sonst wuerde kein Geraet mit Halbleitern funktionieren. Ob die SGL vollstaendig ist, ist eine andere Sache. Es ist ja "nur" eine statistische Gleichung.

Und sind uns Dualismen und Komplementarilät nicht so gut bekannt, das wir sie gar nicht mehr wahrnehmen.
Beides ist uns so vertaut, dass wir dies gar nicht mehr wahrnehmen. Wenn wir ein Musikstueck anhoeren ist das ein Dualismus pur auf allen Skalen. Wir hoeren Spektren und Amplitudenverlaeufe, Huellkurven in allen Variationen. Zeit und Frequenzbereich nehmen wir dual wahr ! Und dazu denjenigen der diese erzeugt.
Was wundert uns so an der Nicht-Lokalität (schreckliches Wort) ?
Die spukhafte Fern"wirkung". Weil diese nicht ueber einen Dualismus erklaerbar ist. Sondern nur ueber Spuk oder eine Erweiterung des physikalischen Raumes. Oder eine dritte Moeglichkeit die uns vielleicht aus goettlicher Hand zufliegt wenn wir nochmals knapp 100 Jahre es uns im Kaminsessel gemuetlich machen.
So wie bei diesem Ehepaar, das vor einigen Wochen mehrere Tage auf eine goettliche Pannenhilfe gewartet hat :-)

Gruesse

richy
06.10.11, 18:01
Hi Knut

Ich habe deine Diskussion im AC Forum mitverfolgt. Um dir eventuelle Muehen zu ersparen schaust du mal hier :
http://www.quanten.de/forum/showthread.php5?t=1602&highlight=mediaon
Als Jurist wirst du schnell einsehen, dass man die gegebene Sachlage am besten einfach akzeptiert. Im Forum hier wird das Thema uebrigends nicht gerne gesehen.

Viele Gruesse

Knut Hacker
06.10.11, 18:17
Richy,
danke,
ich antworte dir per PN, da dies nicht Thema des Threads ist ist

richy
07.10.11, 17:48
Hi Knut
Ich habe mir nochmals die Loesungsansaetze bezuegliche der Existenzen und Nichtexistenzen von Existenz und Nichtexistenz angschaut :
http://www.quanten.de/forum/showthread.php5?t=1560&highlight=Russelsche
Der Loesungsansatz ueber Mengen und Klassen einer Typentheorie war gar nicht so ungeschickt :
http://www.quanten.de/forum/showthread.php5?t=1560&page=5&highlight=Russelsche
Allerdings anstrengend beim Lesen :-)
Ich denke die Antinomie spielt beim Goedelschen Unvollstaendigkeitssatz schon eine Rolle, bin mir aber nicht sicher. In dem Fall waere es dann vielleicht so, dass man das Mengenkonzept hier nicht auf die natuerlichen Zahlen uebertragen kann. Dass man hier nicht in Menge und Klasse unterteilen kann. Wobei mir als unterste Menge, die dann "ein festes Axiom darstellen wuerden" spontan die Primzahlen einfallen. Aber solch eine Behebbarkeit waere sicherlich jedem Mathematiker sofort aufgefallen. Am Goedelschen Unvollstandigkeitssatz gibt es keinerlei Zweifel.
Immerhin zeigt das Beispiel, dass der Satz nicht unbedingt auf alle formalen Systeme oder auch physikalischen Systeme uebertragbar ist. Es waere tatsaechlich interessant dies mal genauer zu Untersuchen. Wahrscheinlich gibt es darueber schon irgendwelche Arbeiten.

Kennst du diese Seiten hier ?
http://www.thur.de/philo/main.htm

Gruesse

Knut Hacker
07.10.11, 18:53
Ich denke die Antinomie spielt beim Goedelschen Unvollstaendigkeitssatz schon eine Rolle, bin mir aber nicht sicher.

Hallo richy,

Die beiden Gödelschen Unvollständigkeitssätze geraten spätestens dann in die Mühlen der Antinomien, wenn es um ihre eigene Geltung geht. Wie auch die antinomistishe Logik des Aristoteles – der Grundsatz vom ausgeschlossenen Dritten – können sich solche Sätze nicht selbst beweisen. Sie beruhen auf Axiomen und Axiome sind unbeweisbar.(Aristoteles hatte das selbst erkannt und geschrieben: „ Die Auffassung, dass die Axiome beweisbar sind, bedeutet einen Mangel an Schulung“), Hier stoßen wir auf das Problem von Letztbegründungen und damit auf das Münchhausen- Trilemma von Hans Albert.
Ich habe daher zum Beispiel etwas gegen Suchen nach einer „Weltformel“. Denn angenommen, man fände eine solche, könnte sie sich ja selbst nicht begründen, weil die Fragen offen blieben, warum es überhaupt eine solche gibt und warum gerade sie und keine andere.

Knut Hacker
08.10.11, 17:20
zeigt sich die Nichtlokalität der Quantentheorie durch die Verschränkung mit der Umgebung.


Ich möchte an dieser Stelle eine laienhafte – da mir auf dem Gebiet der physikalischen Erhaltungssätze die Kompetenz fehlt – Überlegung einer fachlichen Überprüfung aussetzen, bevor ich mit ihr philosophisch Schindluder treibe:

Letzte Elemente sind sich selbst überlassen in dem Sinne, dass sie Energie an das System, dessen Elemente sie sind, abgeben, aber nicht aufnehmen. Sie sind insoweit einem geschlossenen System vergleichbar. Für sie gilt daher der zweite Hauptsatz der Wärmedynamik von der Zunahme der Entropie. Als höchstmöglicher Zustand der Entropie kann dann die Nichtlokalität (Superposition/Verschränkung) verstanden werden.
Im Makrokosmos ist es anders. Hier herrscht die Tendenz zur Bildung komplexer dynamischer Strukturen, die ihre Energie aus dem Umfeld beziehen.Denn das Universum ist aufgrund seiner Expansion ein offenes System (der Satz von der Zunahme der Entropie gilt bekanntlich lediglich in geschlossenen Systemen).Die für die makrokosmische fortwährende Systembildung – zunehmende Strukturierung des Weltalls - benötigte Energie hinterlässt natürlich Entropie und ist an sich irgendwann aufgebraucht (Energieerhaltungssatz). Der Makrokosmos wäre dann vollständig durchkonstruiert. Aber die Expansion des Alls beschleunigt sich ja – für welche Erkenntnis jetzt der Nobelpreis verliehen worden ist. Sollte dies entsprechend der herrschenden Hypothese auf eine „dunkle Energie“ zurückzuführen sein, müsste der Energieerhaltungssatz überprüft werden.

Ich bitte, es mir nachzusehen, wenn diese Überlegungen indiskutabel erscheinen.

Bauhof
09.10.11, 08:51
Aber die Expansion des Alls beschleunigt sich ja – für welche Erkenntnis jetzt der Nobelpreis verliehen worden ist. Sollte dies entsprechend der herrschenden Hypothese auf eine „dunkle Energie“ zurückzuführen sein, müsste der Energieerhaltungssatz überprüft werden.
Hallo Knut Hacker,

mir ist es völlig unverständlich, was diese Ausführungen mit der Nichtlokalität der Quantenmechanik zu tun hat. Eröffne doch einen neuen Thread für Themen, die nicht zu einem bestehenden Thema gehören.

Auch dann bitte, wenn manche Philosophen glauben, alles gehöre zusammen.

M.f.G. Eugen Bauhof

Hawkwind
09.10.11, 09:09
Sollte dies entsprechend der herrschenden Hypothese auf eine „dunkle Energie“ zurückzuführen sein, müsste der Energieerhaltungssatz überprüft werden.



Der braucht nicht überprüft zu werden; Energieerhaltung gilt in der Allgemeinen Relativität nur "lokal", aber keinesfalls auf kosmischen Skalen. Durch die kosmische Expansion verliert das Universum permanent Energie:
Das wurde hier schon einge Male diskutiert, z.B.
http://www.quanten.de/forum/showthread.php5?t=1737

Knut Hacker
09.10.11, 17:26
Hallo Knut Hacker,

mir ist es völlig unverständlich, was diese Ausführungen mit der Nichtlokalität der Quantenmechanik zu tun hat.



Ich hatte aber doch geschrieben:

Als höchstmöglicher Zustand der Entropie kann dann die Nichtlokalität (Superposition/Verschränkung) verstanden werden.


Entschuldige bitte, wenn ich so danebenlag!

Knut Hacker
09.10.11, 17:42
Energieerhaltung gilt in der Allgemeinen Relativität nur "lokal", aber keinesfalls auf kosmischen Skalen. Durch die kosmische Expansion verliert das Universum permanent Energie:


Vielen Dank! Aber ist dabei schon die ja erst in jüngster Zeit entdeckte Beschleunigung der Expansion berücksichtigt? Die Bezeichnung "dunkle Energie" und deren Heranziehung zur Erklärung des Phänomens der Beschleunigung des Expansion ist natürlich rein hypothetisch in der aktuellen wissenschaftlichen Diskussion.

Die von dir erwähnte Lokalität der Energieerhaltung - und wohl aller Erhaltungssätze, also auch des Entropieerhaltungssatzes- wird interessant, wenn man eine Quantelung der Raumzeit diskutiert. Dann liege ich gar nicht so schief mit meiner Zurückführung der Nichtlokalität auf diese Sätze, was mich selbst überrascht.

JoAx
10.10.11, 11:24
Hi Leute!

Ich habe die Sache mit Energieerhaltung in der ART ausgegliedert.
http://www.quanten.de/forum/showthread.php5?t=2065


Gruß, Johann

Knut Hacker
10.10.11, 12:10
Hi Leute!

Ich habe die Sache mit Energieerhaltung in der ART ausgegliedert.
http://www.quanten.de/forum/showthread.php5?t=2065


Gruß, Johann

Aber ich habe doch nicht über die ART diskutiert, sondern diese lediglich herangezogen, um zu erklären, warum meine "Schnapsidee", die mikrokosmische Nichtlokalität auf den Entropiesatz zurückzuführen ( höchstmöglicher entropischer Zustand elementarer Materieteilchen ), keine Entsprechung im Makrokosmos findet.

Aber ich will deine Entscheidung nicht kritisieren. Mir ist es egal, wo diskutiert wird, die Hauptsache ist, dass diskutiert wird.

JoAx
10.10.11, 12:54
Hallo Knut!


Mir ist es egal, wo diskutiert wird, die Hauptsache ist, dass diskutiert wird.

Es ist schon etwas mehr daraus geworden, als nur die Antwort von Hawkwind. Das erschien mir zu deinem Thema schon nicht mehr zu passen. Da du scheinbar nur dein eigenen Thread verfolgst, wollte ich mit meiner Bekanntmachung nur dafür sorgen, dass nichts aus dem Blickfeld verschwindet. ;)


Gruß, Johann

Knut Hacker
10.10.11, 13:36
Hallo Knut!



Es ist schon etwas mehr daraus geworden, als nur die Antwort von Hawkwind. Das erschien mir zu deinem Thema schon nicht mehr zu passen. Da du scheinbar nur dein eigenen Thread verfolgst, wollte ich mit meiner Bekanntmachung nur dafür sorgen, dass nichts aus dem Blickfeld verschwindet. ;)


Gruß, Johann

Kein Problem, njema problema!

Harti
11.10.11, 08:31
njema problema!

Hallo Knut Hacker,
welche Sprache ist das ? Meine Tochter sagt mir "njema" bedeutet in suaheli "gut" und das Wort "problema" gibt es in dieser Sprache nicht.
MfG
Harti

Knut Hacker
11.10.11, 16:26
welche Sprache ist das ?

Sollte serbokroatisch sein ( Phonetisch ).

richy
11.10.11, 17:23
Hi Knut

Das Universum ist zwar kein Luftballon, aber das Video zeigt, dass man einen Solchen nicht nur aufblasen kann :
http://www.youtube.com/watch?v=vZjZ2ufOeOM
und auch ein wenig denn Zusammenhang zwischen Entropie und Expansion des Universums. Die Entropie wird mit durch die Anzahl der moeglichen Zustaende betimmt, die besetzt werden koennen.
http://de.wikipedia.org/wiki/Entropie_%28Thermodynamik%29
http://upload.wikimedia.org/wikipedia/commons/thumb/9/9f/Entropie.png/300px-Entropie.png
Wie wuerde sich die Entropie Bild aendern, wenn der Raum des Universums quantisiert waere ? Wurden die Elementarzellen lediglich groesser werden und Materie aus diesen Elementarzellen gebildet, dann wuerde sich die Entropie dadurch wohl gar nicht aendern. Dazu muessten sich die Elemtarzellen schon "vermehren", wie bei der LQG. Das entspricht dort einer zunehmenden Vernetzung in der dualen Struktur, die in der LQG auch angenommen wird. Wenn solch eine Teilung, Vernetzung auch bei den Raumquanten vorkommen wuerde, die Materie bilden, wuerde dies der Materie eventuell nicht so gut bekommen. Vielleicht expandiert daher das Universum hier scheinbar langsamer.
Der Zeitpfeil wird sehr wahrscheinlich vom Gradienten der Entropie festgelegt. Dann bestuende auch hier ein Zusammenhang. Und die Dekohaerenz entspricht einer Realisierung. D.h. einer Ankopplung an eine globale Entropie. Global. D.h. diese oder deren Gradient kann als eine Dimension betrachtet werden auf der unsere Realiitaet genau einen Zahlenwert annimmt. Und bezeichnet man die Entropie als Information, dann scheint es plausibel wenn z.B. Prof Zeilinger meint alleine die Information (das hier nun ein Teilchen zu unserer Realitaet hinzugekommen ist) genuegt.
Also solche Zusammenhaenge kann man schon konstruieren und bestehen vielleicht in irgendeiner Form. Auch dass die Gravitation mit der Entropie verknuepft ist. Ich vermute Herr Zeilinger wird irgendwann auch der Information eine Gravitation zusprechen. (Wobei dies unserem bisherigen Bild, Definition widersprechen wuerde)

Hier gibt es noch einen Versuch dies mit der Wheeler de Witt, kosmologischen SGL zusammenzufassen :
http://www.rzuser.uni-heidelberg.de/~as3/WarumQK.pdf
Yoh, das ist dann schon heftiger Stoff :D

Gruesse

richy
11.10.11, 18:21
Als höchstmöglicher Zustand der Entropie kann dann die Nichtlokalität (Superposition/Verschränkung) verstanden werden.
Ich wuerde hier nicht den Begriff Nichtlokalitaet verwenden, denn der bezieht sich auf unsere Realitat. Superposition/Verschränkung ... einfach |Psi|^2 selbst.
Die Welle verkoerpert alle moeglichen Zustaende und damit waere das IMHO nicht die hoechstmoeglichste Entropie, sondern die geringste. Es gibt nur diesen einen Zusatand, der alle Moeglichkeiten abdeckt, belegt.
Und was folgt daraus noch. Dass es fuer Psi selber gar keine Entropieaenderung gibt. Und daher auch keinen Zeitpfeil. Daher keine klassische Bewegung. Kein d/dt* Und daher wundere ich mich auch warum denn jeder das d/dt in der SGL einfach so akzeptiert. Klar das t bezieht sich auf unsere Zeit und deren Ablauf. Aber nicht auf eine Dynamik von Psi selbst. Die kann es doch gar nicht geben.
Und deshalb ist die Wheeler de Witt Gleichung wohl zeitunabhaengig. Denn betrachtet man Psi ueber das ganze Multiuniversum, dann gibt es keine nichtbelegten Zustaende und insgesamt keine Entropieaenderung und damit kein d/dt. So stelle ich mir das vor. Und daher hat Prof Rauscher wohl auch eine Zweite t Koordinate eingefuehrt. Inzwischen scheint sie sich aber mit M Theorien zu beschaeftigen. Es gibt ein Buch von ihr dazu.

Knut Hacker
11.10.11, 19:02
Hallo richy,
vielen Dank für die vielen Denkanregungen zu meinem Problem
Demnach liege ich falsch mit meiner Überlegungen, dass es sich bei der Nichtlokalität um einen Zustand höchstmögliche Entropie von Quantensystemen handeln könnte, nämlich in dem Sinne, dass die Quantensysteme in ihrer Elementarität einem geschlossenen System gleichgestellt werden könnten., Entropie als zeitliche Erscheinung ist ja ein Makrozustand
( Dein Link: Die Entropie ist über die Wahrscheinlichkeiten der einzelnen reinen quantenmechanischen Zustände im Makrozustand gegeben durch - die Formeln erließen sich leider nicht übertragen in dieses Fenster -

wobei pi die Wahrscheinlichkeit ist, im i-ten Mikrozustand zu sein. kB ist die Boltzmann-Konstante. Die Wahrscheinlichkeiten pi können Werte zwischen 0 und 1 annehmen, somit ist und die Entropie positiv semidefinit.)
und könnte nach meinen Überlegungen nur analog auf einem Mikrozustand übertragen werden


Die Welle verkoerpert alle moeglichen Zustaende und damit waere das IMHO nicht die hoechstmoeglichste Entropie, sondern die geringste.

Dieses Argument ist schlagend.
Es bedarf daher meiner „Rechtfertigung“ nicht mehr, dass meine Analogie keine Konsequenzen für den Makrokosmos hätte.
Ein gewisses Problem bleibt für mich außerhalb der Fragestellung:
Nur ein konstantes oder rekontrahierendes Universum ist ja geschlossen, das expandierende ist offen. Nur so lässt sich die zunehmende Strukturbildung trotz des Satzes von der zunehmenden Entropie erklären. Die Entropie ist natürlich ohne die Funktion der Zeit gegenstandslos. Im Universum gibt es jedoch keine absolute Zeit. Demnach wäre die Strukturbildung beobachtungs- und bereits als solche, also nicht erst qualitativ und quantitativ, gravitationsabhängig (Gravitation im Sinne der AR verstanden).
Das ist aber vielleicht Thema des vom Moderator geöffneten Parallelthreads.

Muss für heute leider schließen.

Knut Hacker
12.10.11, 17:58
richy,
ich muss mich nach weiteren Studium doch revidieren und verweise auf meinen Beitrag im Parallelthread "Energie- und Impulserhaltung in der ART".